Wo unsere physikalische Welt nur Illusion ist

   


Objekt vs Modell (in der Physik)

   


Zum Begriff physikalischer Objekte

   


Physik: Fragen, Antworten, und der eine oder andere Verdacht

   


Zum Realitätsbegriff in der Physik

   





D i s k u s s i o n


 Beitrag 0-122
Wie sich die physikalische Größe » Spin « definiert

 
 

 
Zum » Spin « der Elementarteilchen

 
 
Gewisse Elementarteilchen — Elektronen etwa — verhalten sie wie kleine Magnete.
 
Wir denken uns die Lage dieses Magneten beschrieben durch einen imaginären Pfeil — genannt » Spin « — und stellen dann fest, dass der eine recht merk­würdige Eigenschaft hat:
 
Wählt man irgend eine, völlig beliebige Richtung und frägt das Teilchen dann, ob sein Spin in diese Richtung zeige, so bekommt man entwder die Antwort JA oder die Antwort NEIN, niemals aber irgend eine andere Antwort.
 
Statt JA oder das NEIN sagen die Physiker « Spin up « bzw. » Spin down «.
 
 
Das Merkwürdige also ist, dass es dem Spinpfeil irgendwie nicht möglich zu sein scheint, in eine Richtung zu zeigen, die nicht die ist, auf die sich unsere Frage bezog.
 
Noch merkwürdiger: Werden zwei Teilchen gleicher Art, die Spin haben, z.B. zwei Elektronen e1 und e2, durch ein und dasselbe atomare quantenphysikalische Ereignis erzeugt, so sind sie hinsichtlich Spin mit einander verschränkt.
 
Dies bedeutet: Fragen wir e1 nach seinem Spin, so wird es mit » up « oder » down « antworten. Dieselbe Frage dann auch dem e2 gestellt, wird die jeweils andere Antwort ergeben — und das selbst dann noch, wenn e2 inzwischen ans andere Ende der Milchstraße gebracht worden ist.
 
 
Note: Der Begriff der » Verschränkung « von Quanten wurde geprägt durch Erwin Schrödinger: Discussion of Probability Relations between seperated Systems, Proceedings of the Cambridge Philosophical Society 31, 555 (1935).

 

  Beitrag 2102-143
Definition: Sichtbares Licht

 
 
Horst in 2102-140:
Wir sollten uns darüber im Klaren sein, daß [ sichtbares ] Licht tatsächlich nichts anderes ist, als eine Bezeichnung für die subjektive Empfindung eines geringen Anteils der uns bekannten elektromagnetischen Strahlung im Frequenzbereich von 380 bis 780 Nanometer.

Auch das sichtbare Licht ist nur ein eine Definition dafür, wie wir Menschen eine elektromagnetische Strahlung wahrnehmen
Eigentlich müsste man sagen für den Menschen sichtbares Licht, denn bestimmte Tiere, wie etwa Insekten können auch noch infrarotes bzw UV-Licht "sehen"

Und was ist z.B. mit Haien, die auch andere elektromagnetische Strahlung wahrnehmen. Dadurch "sehen" sie z.B. Tiere, die sich im Sand eingegraben haben. Jeder Nervenimpuls sendet ein, wenn auch kleines, EM-Signal aus. Wie die Haie diese EM-Strahlung wahrnehmen ist unbekannt. Ob sie das EM-Bild "sehen" liegt daran, wie die Daten vom Hai-Gehirn interpretiert, bzw dem Tier ins Bewusstsein gerufen werden.
Setzte man Haie vor eine starke EM-Quelle, dann ist das so, als würde man vor einem Menschen eine Blendgranate zünden.

Ebenso könnte es sein, das ausserirdishes Leben, falls es denn existiert, eine ganz andere Wellenlänge als "Sichtbares Licht" bezeichnet.
Die Evolution schöpft immer die Möglichkeiten aus, die zur Verfügung stehen.
Leben diese Aliens etwa in der Nähe eines Pulsars, dessen EM-Felder stärker sind als das Licht zwischen 380nm - 780 nm so könnte es durchaus sein, das deren "Augen" für die Wellenlänge der Pulsar-Energie optimiert sind. Für unser sichtbares Licht sind sie womöglich absolut blind.

Wie wir unsere Welt wahrnehmen hängt nur von den Informationen ab, die uns unsere Sinne liefern. Aber auch Eigenschaften oder Ereignisse, die durch uns nicht beobachtbar sind, können existieren.

Daher dürfen wir Vorhandenes nicht ausschließlich nach dem beurteilen, was es zu sein scheint.

Wir können nie sicher sein, sein eigentlichen Wesen schon erfasst zu haben. 


Das Wesen beispielsweise, das wir Materie zuschreiben, variiert unerwartet stark, je nachdem, auf welcher Größenskala wir sie betrachten.
 

  Beitrag 2112-7
Definition: Licht

 
 
Horst in 2112-6:
 
Ps. Und der der Rest der elektromagnetischen Strahlung?


In der Physik wird unter "Licht" grundsätzlich alle elektromagnetische Strahlung verstanden (wer das nicht will, muss von "sichtbarem Licht" sprechen).



Siehe etwa Was ist Licht?. Harald Lesch sagt dort explizit, er spreche jetzt nur über sichtbares Licht.

 

  Beitrag 1948-1
Definition: Information

 
 

Über sieben verschiedene Informationsbegriffe

Zwischenergebnisse auf dem Weg hin zu einer Antwort auf die Frage: Was ist Information im Sinne der Natur?



Information wird heute — neben Materie und Energie — vielfach als eine dritte Grundgröße angesehen (Brockhaus, S. 657). Information, als Begriff, scheint nicht auf andere Größen zurückführbar zu sein. Dennoch ist der Informationsbegriff fundamentaler Baustein aller Kommunikationsprozesse.


1. Der semantische (zwischen-menschliche) Informationsbegriff

Hier versteht man unter Information korrektes Wissen, welches man sucht, findet oder mitgeteilt bekommt.
Zwischen dem semantischen und dem nachrichtentechnischen Informationsgehalt einer Nachricht besteht KEINE wie auch immer geartete Beziehung:


2. Der nachrichtentechnische Informationsbegriff

Der Informationsgehalt einer Nachricht ist die Zahl binärer Entscheidungen, die man benötigt, eben diese Nachricht von einer ebenso komplexen anderen zu unterscheiden.

Da jede Nachricht eine Folge von Zeichen ist, kann auch jedes Zeichen als so eine Nachricht aufgefasst werden. Da nicht alle Zeichen mit gleicher Wahrscheinlichkeit auftreten, liegt es nahe, ihren Informationsgehalt mit der Wahrscheinlichkeit ihres Auftretens zu gewichten. Werden diese Produkte aufsummiert, so erhält man eine Zahl, die man nach Shannon als mittleren Informationsgehalt, Informationsdichte oder auch als Entropie der Nachricht bezeichnet.


3. Der sprachwissenschaftliche Informationsbegriff

Er verfeinert den semantischen Informationsbegriff dahingehend, dass neben Syntax und Semantik einer Nachricht auch noch deren Pragmatik Gegenstand der Betrachtung sein kann: Es kann vorkommen, dass unterschiedliche Empfänger ein und derselben Nachricht ihr unterschiedliche Information entnehmen (Pragmatik = empfängerspezifische Sicht auf semntische Inhalte).


4. Der kybernetische Informationsbegriff

Zunehmende Ordnung bedeutet zunehmende Information: Information ist das, was den Unterschied ausmacht.
Wo kybernetischer Informationsgehalt zunimmt, reduzieren sich Entropie und nachrichtentechnischer Informationsgehalt (und umgekehrt).
Die Kybernetik (das Wort bedeutet eigentlich "Steuermannskunst") abstrahiert reale Systeme hinsichtlich gewisser Eigenschaften und Verhaltensweisen zu Modellen — die man dann kybernetische Systeme nennt — und untersucht deren Struktur und Verhalten.


5. Der naturwissenschaftliche Informationsbegriff

Er wurde wesentlich geprägt durch Carl-Friedrich von Weizsäcker (Physiker und Philosoph) und kennzeichnet sich so:

Ein Telegramm enthält Information. Ist die nun als etwas Materielles oder Bewußtseinsinhalt? Antwort: weder noch:
  • Die Druckerschwärze auf einem per Fax versandten Papier ist verschieden von der Druckerschwärze des beim Empfänger ankommenden Exemplars: "Information ist gerade das, was beiden Zetteln gemeinsam ist" (Weizsäcker 1974).
  • Ähnliches gilt für den (pragmatischen) Inhalt jeder Nachricht: Das, was der Absender gedacht hat, kann verschieden sein von dem, was der Empfänger denkt. Dennoch ist beiden etwas gemeinsam. Eben das ist Information.
Und daraus folgert Weizsäcker:

"Man beginnt sich daher heute daran zu gewöhnen, daß Information als eine dritte, von Materie und Bewußtsein verschiedene Sache aufgefaßt werden muß. Was man damit entdeckt hat, ist an neuem Ort eine alte Wahrheit. Es ist das platonische Eidos, die aristotelische Form, so eingekleidet, daß auch ein Mensch des 20. Jahrhunderts etwas von ihnen ahnen lernt." (Weizsäcker 1974)


6. Der biologische Informationsbegriff

Weizsäcker findet den Gebrauch des Informationsbegriffs in Zusammenhang z.B. mit dem Chromosomensatz "völlig legitim" (Weizsäcker 1974), obwohl hier niemand spricht oder einem anderen Menschen etwas mitteilt. Spannende Frage also:

Wie lässt sich Information jenseits der menschlichen Sprache verstehen?


Und wie lässt sich verstehen, dass die Natur ganz offensichtlich Information erzeugt, wo doch gilt:

» Information ist nicht etwas, was auf der Straße herumliegt so wie Kieselsteine, sondern
Information wird erzeugt; und sie wird erzeugt nur von denjenigen, welche imstande sind, in Begriffen zu denken. «
(Weizsäcker 1973)



7. Der physikalische Informationsbegriff

Wäre so einer schon erarbeitet, müsste er wohl mindestens den biologischen und den kybernetischen verallgemeinern (und gemeinsame Wurzel beider sein).
Nicht vergessen sollte man: Auch auf der Ebene physikalischer Systeme treten Phänomene der selbstorganisierten Strukturbildung auf, die weit mehr auf Freiheitsgrade zurückzuführen sind als auf ein Ursache-Wirkungs-Verhältnis.

Somit scheint Information die Grundlage aller Selbstorganisation zu sein.



Siehe auch: Capurro, wo u.A. berichtet wird, wie Weizsäcker den Begriff "Evolution" einordnet:

Die moderne Naturwissenschaft basiert nicht nur auf dem Begriff der Erfahrung, sondern auch auf dem der Evolution.


Was bedeutet Evolution?


Antwort: Evolution ist "Vermehrung der Menge an Form", oder anders formuliert: ein "Anwachsen der Information" (Weizsäcker 1973).

Und: "Information ist nur, was Information erzeugt." (Weizsäcker 1974).



Auch der großen Frage, wie Bewusstsein entsteht, ist Weizsäcker (allerdings ohne Ergebnis) nachgegangen:


» Das Bewußtsein taucht in der Evolution aus dem Meer des Unbewußtseins auf.
Ist also doch Form das Zugrundeliegende und Bewußtsein eine ihrer Ausprägungen? «

» Aber wie kann Form Bewußtsein erzeugen? Ist sie selbst geistig? Was könnte man damit meinen? «

(Weizsäcker 1992)



Gebhard Greiter (grtgrt)
 

Nachtrag (am 15.2.2013):

In seinem Buch "Die Evolution des Geistigen" weist Thomas Görnitz darauf hin, dass jedes physikalische Objekt — als Träger von Information —

  • zugängliche und auch
  • nicht zugängliche Information (Entropie)

trägt bzw. tragen kann. Wichtiger Teil seiner zugänglichen Information ist die Information darüber, an welcher Stelle im Universum es sich befindet.

Allgemeiner: Genau der Teil seines Zustandes, den die Natur uns (im Prinzip wenigstens) zugänglich macht, ist Träger seiner zugänglichen Information.


Wie Görnitz auf den Seiten 156-158 seines Buches zeigt, kann berechnet werden, wie groß die Entropie eines Objekts ist:

Zitat von Görnitz:
 
Wenn man sich ein Schwarzes Loch mit dem Materiegehalt unseres Universums denkt, so hätten beide gleiche Dichte und Ausdehnung. Ließe man nun noch ein Teilchen in das Schwarze Loch fallen, so würde des Teilchens zuvor zugängliche Information unzugänglich und damit zu berechenbarer Entropie.

Mit diesem Gedankenexperiment konnte ich zeigen, dass dann beispielsweise einem Planck-Black-Hole, dessen Entropie in unserem Kosmos 1 Bit ist, insgesamt etwa 1062 QuBits entsprechen (deren Menge zuvor als Menge unverborgener Information mit der Entropieformel nicht berechenbar war).

Für ein Proton [als Informationsträger ergibt sich so, aus den heutigen astronomischen Beobachtungsdaten, ein Wert von 1041 QuBits.

Das liegt sehr nahe am früher von C.F. v. Weizsäcker vorgeschlagenen Wert von 1042 QuBits.
 


Interessant ist auch, was er auf Seite 172 sagt:

Zitat von Görnitz:
 
Entropie ist Information, die unbekannt ist, entweder weil eine Kenntnisnahme zu aufwändig oder zu uninteressant wäre (wie beispielsweise das Schicksal eines einzigen Atoms in einem Gas).

Entropie ist — salopp gesagt — Informationsmüll, wie Akten nach dem Schreddern: alle Buchstaben sind noch da, aber man kann nichts damit anfangen. Man muss sie aber los werden, um Platz für Neues zu schaffen.

Problematisch an dieser seiner Aussage aber ist, dass sie nicht unterscheidet zwischen
  • Information, die man  i g n o r i e r t , und
  • Information, die uns prinzipiell  u n z u g ä n g l i c h  ist (wie etwa die in einem Schwarzen Loch oder die in Daten, zu denen die Natur uns noch keinen Decodierungsschlüssel zur Verfügung gestellt hat).

 

  Beitrag 1951-1
Definition: Thermodynamische Entropie

 
 

Was genau ist Entropie?


Ist S ein in sich abgeschlossenes System und Z(t,S) sein Zustand zum Zeitpunkt t, so versteht man unter der Entropie E(t,S) von S zum Zeitpunkt t die Länge der kürzesten Bitfolge, über die sich der Zustand Z(t,S) komplett beschreiben lässt.


Entropie ist ein Maß für die Komplexität von Systemzuständen.

Je ungeordneter ein Systemzustand, desto höher seine Entropie.


 

  Beitrag 1954-1
Definition: Well Defined Degrees of Freedom (WDDF)

 
 

WDDF — Well Defined Degrees of Freedom


Zum physikalischen Informationsbegriff:

Grtgrt aus 1948-1:
 
Wäre so einer schon erarbeitet, müsste er wohl mindestens den biologischen und den kybernetischen verallgemeinern (und gemeinsame Wurzel beider sein).
Nicht vergessen sollte man: Auch auf der Ebene physikalischer Systeme treten Phänomene der selbstorganisierten Strukturbildung auf, die weit mehr auf Frei­heitsgrade zurückzuführen sind als auf ein Ursache-Wirkungs-Verhältnis.

Somit scheint Information die Grundlage aller Selbstorganisation zu sein.

 


Wie in Beitrag 1948-34 schon erklärt wurde, stellen Ordnung und Unordnung die beiden Grundformen dar, in denen Information auftreten kann. Ihr Bezug zu Freiheitsgraden ist offensichtlich:
  • Ordnung setzt  G r e n z e n  für Freiheitsgrade,
  • Unordnung ist Symbol für die  A u s s c h ö p f u n g  von Freiheitsgraden.

Nicht zuletzt deswegen kommt mir bei der Suche nach einer Antwort auf die Frage, was denn nun eigentlich das Wesen eines Begriffs von Information ausmachen könnte, der tauglich wäre, Basis aller physikalischen Gesetze zu sein, der Verdacht, dass es sich hierbei um ein Naturgesetz handeln könnte,

welches Freiheitsgraden entspricht,

entlang derer sich jeder im Universum ablaufende Prozess entwickelt in dem Sinne, dass die Natur
  • ihn zwingt, sich NUR in ihrem Rahmen zu entwickeln
  • ihm ansonsten aber jede Freiheit lässt, den Rahmen, diese Freiheitsgrade also, VOLL auszuschöpfen.

Der durch die Freiheitsgrade gesetzte Rahmen in Kombination mit absolutem Zufall im Sinne der Quantenphysik könnte zur Folge haben, dass solche Prozesse selbstorganisierend sind und so Ordnung entsteht, die der Evolution fähig ist — einer Evolution, die gezielt die jeweils stabileren Strukturen begünstigt, weil die ja, eben
w e g e n  ihrer Stabilität, selbst wieder zu steuernden Faktoren werden in dem Sinne, dass sie den Zufall mehr und mehr kanalisieren:

Hinreichend stabile Strukturen verändern die Wahrscheinlichkeiten, mir der ansonsten gleich wahrscheinliche Ereignisse zufällig eintreten.


Gebhard Greiter (grtgrt)
 

  Beitrag 1954-5
-

 
 
Henry aus 1954-4:
"Ordnung" wird durch Gravitation geschaffen, "Unordnung" ist der systemimantente "Drang" nach völliger Gleichverteilung, nach Ausgleich zwischen Potentialen. Der antopozentrische Begriff der "Freiheitsgrad" hat hier keinerlei Bedeutung.

Hi Henry,

"Ordnung" wird keineswegs nur durch Gravitation geschaffen, sondern — um nur EIN Beispiel zu geben — auch durch die einem Quantensystem (einem Molekül etwa) zugeordnete Wellenfunktion ψ.

Auch meinen Begriff "Freiheitsgrade" interpretierst du viel zu eng:

Nimm z.B. ein Molekül und die ihm zugeordnete Wellenfunktion. Sie definiert sog. "Knotenflächen". Die wiederum stellen die Menge aller Raumpunkte dar, an denen sich kein einziges Elektron im Molekül aufhalten kann (kein Elektron wird sich dort zeigen, wenn man versucht, es zu beobachten).

Damit ist diese Wellenfunktion Beschreibung eines sehr komplizierten "Freiheitsgrades". Er gibt dem Orbitalmodell des Moleküls eine ganz bestimmte Struktur (siehe etwa diesen Artikel und die Bilder darin) und sagt den Elektronen, wo sie sich zeigen bzw. nicht zeigen dürfen.

Gruß,
grtgrt
 

  Beitrag 1926-38
Versuch einer Definition: Bewusstsein

 
 
Gregor Lämmer aus 1926-35:
 
Denken ist Bewusstsein.

Das zu behaupten, geht mir zu weit.

Denken ist zunächst mal nur Informationsverarbeitung. Erst wo Bewusstsein dazukommt, wird daraus mehr.

Vielleicht sollte man sagen:

Denken, das sich selbst zum Gegenstand haben kann, bezeichnet man als Bewusstsein.



Gregor Lämmer aus 1926-34:
 
Bewusstsein ist bewusstes Sein.

Es gibt einen Unterschied von "Wissen, dass man weiß" und "Nichtwissen, wie man weiß".

Beide Aussagen finde ich treffend und richtig.

Dennoch scheint mir die erste als  D e f i n i t i o n  von Bewusstsein ungeeignet, da man dann ja sofort fragen müsste, was es denn eigentlich bedeutet, "zu sein".

grtgrt


PS: Brauchbare Definitionen sind nur solche, die
  • entweder den zu definierenden Begriff auf andere, bereits wohldefinierte Begriffe zurückführen,
  • oder den neuen Begriff implizit definieren (sozusagen als Lösung einer logischen Gleichung). In dem Fall allerdings müsste man beweisen, dass jene Gleichung auch tatsächlich Lösungen hat.

 

  Beitrag 1963-1
Eine neue — sehr naheliegende — Deutung quantenphysikalischer Messergebnisse (und verschränkter Quanten)

 
 


Eine neue — wirklich naheliegende — Deutung quantenphysikalischer Messergebnisse


Gebhard Greiter (grtgrt)



Die ständig stattfindenden Dekohärenzprozesse (allgemeiner: die Neudefinition der Wellenfunktion eines Quantensystems in jedem Elementarereignis) wird gegenwärtig so interpretiert, dass hierdurch Realität produziert wird: ein konkreter Zustand, den man wohl am treffendsten als eine Art Schnappschuss begreift, der die Stelle, an der das Elementarereignis passiert, abphotographiert: Details dazu in den Beiträgen 1915-107, 1915-66 und 1915-86 (die man am besten versteht, wenn man sie in eben dieser Reihenfolge nachliest).

Zudem gibt es das heute viel diskutierte Phänomen der Quantenverschränkung, welches dadurch gekennzeichnet ist, dass Messungen der Eigenschaften zueinander verschränkter Quanten zueinander korrelliertes (in bestimmten Beispielen sogar absolut identisches) Ergebnis liefern, obgleich der Wert der jeweils beobachteten Größe auch da noch ein absolut zufällig eintretender ist.

Beispiel: Wenn ein Atom zwei zueinander verschränkte Photonen aussendet und man deren Polarisation misst, stellt man fest, dass beide stets gleich polarisiert sind. Andererseits kann der konkrete Wert der Polarisation für so ein Paar in keiner Weise vorausgesagt werden. Er ist ebenso wahrscheinlich wie der jeweils andere (wenn die Versuchsanordnung so ist, dass die eingesetzten Polarisationsfilter genau zwei Werte möglich machen).

Diese Beobachtung kann so interpretiert werden, dass die Messung uns nicht einen Schnappschuss jener Photonen zeigt, sondern nur einen Schnappschuss einer Projektion p dieser Photonen auf einen Teilraum unserer Welt, der weniger Dimensionen hat als diese Welt selbst und zudem noch so beschaffen ist, dass auf bestimmte Weise zueinander verschränkte Photonen unter dieser Projektion denselben "Schatten" werfen (sprich: auf ein und dasselbe Objekt abgebildet werden).

Eine Möglichkeit, sich jene Schattenwelt vorstellen, wäre, sich daran zu erinnern, dass der Stringtheorie entsprechend unser Universum über seine 4 uns sichtbaren Dimensionen hinaus noch bis zu 7 weitere, sog. aufgerollte Dimensionen hat. Es könnte also gut sein, dass jene Projektion p das Photon einfach nur in einen Teilraum projeziert, der durch einige dieser aufgerollten Dimensionen aufgespannt wird. Er ist dann auf jeden Fall nur endlich groß, aber ohne Anfang und Ende.


Meine Frage an Euch alle:

Kann mir jemand beweisen, dass diese Deutung KEINEN Sinn macht (bzw. falsch sein muss)?



grtgrt

PS: Man könnte noch weiter gehen, indem man sich frägt, ob unsere Welt nicht vielleicht zwei unterschiedliche Existenzformen hat: Eine, die das ist, was wir als unsere reale Welt W wahrnehmen, und eine andere, die ihr Bild unter p ist: p(W) also. Vielleicht also ist p gar keine Projektion, sondern eher ein Isomorphismus, der uns — im Beispiel oben — ein wirkliches Objekt einfach nur ausschnittsweise zeigt: eben so, dass wir denken, wir sähen zwei unterschiedliche Photonen?

 

  Beitrag 2075-193
-

 
 
Stueps in 2075-188:
 
Im Speziellen deutet die (für uns Laien sehr schwer nachzuvollziehende) Verletzung der Bellschen Ungleichung darauf hin, dass diese Art Zufälle echt sind, und nicht auf Parametern beruhen, die wir nicht erkannt haben und erkennen können.


Hallo Stueps,

meinem Verständnis nach beweist die Bellsche Ungleichung (in Kombination mit Aspects Experimenten) nicht das Vorliegen von absolutem Zufall, sondern stattdessen nur die sog. » spukhafte Fernwirkung « — die Tatsache also, dass miteinander verschränkte Quanten zueinander korrelliertes Verhalten aufweisen  o h n e  dass vorher schon bestimmt worden sein kann, wie sie auf welche Messfrage antworten werden.

Gruß, grtgrt
 

  Beitrag 1969-27
Grundsätzliches

 
 
U...2 aus 1969-7:
Guten Morgen Gregor,

ich danke Dir für Deine Antwort. Wir bewegen uns damit aber auf der Schiene des Glaubens und nicht der Wissenschaft.
Deshalb lasse ich das jetzt einfach mal so stehen ...

Nun gut, Du lässt das einfach mal so stehen.

Also antworte ich Dir einfach mal so dahin.

Ich habe es schon oft erlebt, dass Menschen nur wissenschaftliche Erkenntnisse akzeptieren und nicht im Geringsten bereit sind, sich auf metaphysischer Ebene zu bewegen. Mit so einer Haltung kann man sich gut durch's Leben bewegen und sich selbst eine "heile Welt" zusprechen. Doch diese "heile Welt" ist etwas Bruchstückhaftes, weil die Wissenschaften immer nur zu einer begrenzten (bruchstückhaften) Erkenntnis gelangen können. Es liegt in ihrer Natur. Mehr ist ihnen nicht "gegeben".

Die "Welt der Physik" ist eine phänomenale Welt. Das "Höchste", was ihr möglich ist, ist die Vermittlung von Sinneseindrücken (an die Menschen) in Form von Phänomenen. Sinneseindrücke gelangen in unser Bewusstsein, mit deren Hilfe wir von der Welt Kenntnis nehmen, uns also bewusst wird - im wahrsten Sinne des Wortes -, dass die Welt sich uns kund tut über Kausalketten, die zu Sinneseindrücken führen.

Die wenigsten Menschen machen sich jedoch bewusst, dass das Bewusstsein eine geistige Komponente ist, die sich wissenschaftlich nicht erklären lässt, weil die Wissenschaft hierzu gar keinen Zugang finden kann. Geist ist immer etwas Überweltliches, so wie auch Gott als außerhalb der Schöpfung gedacht werden muss.

Die Welt kann nicht aus sich heraus etwas generieren, was schon vorher zu ihrer Beherrschung und Entstehung vorhanden sein musste. Eine Welt, die ihren Gott schafft, ist so wenig vorstellbar wie der Geist eines Menschen, der das Ergebnis eines rein materiell deterministischen Vorgangs ist.

Gott hat die Welt geschaffen und den Menschen zur Beherrschung eingesetzt. Anders ausgedrückt: Der Geist herrscht über die Materie. Deshalb wird die Materie niemals Geist hervorbringen können, weil dieser immer zuerst da ist.

Die Neurowissenschaften sind schon sehr lange damit beschäftigt, Licht ins Dunkel zu bringen. Aber bis heute kann kein Neurowissenschaftler wissenschaftlich aufzeigen, was Bewusstsein ist. Dies darf nicht mit den Prozessen verwechselt werden, die im Gehirn bei Denkvorgängen ablaufen. Diese Abläufe sind notwendig, damit der Geist über die materiellen Vorgänge des Gehirns in die materiellen Vorgänge des Körpers eingreifen kann. Solche Vorgänge sind sehr gut erforscht und haben wesentlich zu den Erfolgen von Psychopharmaka beigetragen.

 

  Beitrag 1971-6
Um Glauben – an was auch immer – kommen wir nicht herum

 
 
U... aus 1971-5:
Naja, die Gläubigen werde sich schwer damit tun, auch nur den Hauch eines Beweises für die Existenz Ihres Gottes erbringen zu können.

Das gilt auch für Dich, U... Jeder Mensch glaubt an etwas, egal an was, ob das nun ein Gott ist oder irgendeine andere Instanz, die Gott ähnlichen Chrakter hat. Das ist unser Schicksal. Jeder Mensch muss glauben. Das mag überheblich klingen, ist aber sehr rational gemeint. Wir wissen zu wenig, um nicht glauben zu können.

Jeder Mensch ist in der Lage, zu denken. Aber kein Mensch weiß, was Denken ist, wie es funktioniert und was Bewusstsein ist. Wir können nur feststellen, dass es so ist, aber niemals, warum. Deshalb müssen wir an den Grund für alles glauben. Egal, was wir für diesen Grund halten, aber um den Glauben kommen wir nicht herum. Das kann man drehen und wenden, wie man will.

Und wer nicht an ein Leben nach dem Tod glaubt - oder wer an das Nichts glaubt - oder an den Zufall - oder was auch immer - es ist immer ein Glaube.
 

 Beitrag 0-9
Was Einstein dazu dachte

 
 

 
Einsteins recht differenzierende Meinung zu Gott

 
 
Wie aus einem Brief Einsteins hervorgeht, den er 1954 an den Religionsphilosophen Erich Gutkind schrieb, sah Einstein sich selbst
  • weder als Atheist,
  • noch als jemand, der an die Bibel glaubt.

 
Hier seine wesentlichen Aussagen:

Zitat von Einstein:
 
    Das Wort Gott ist für mich nichts als Ausdruck und Produkt menschlicher Schwächen, die Bibel eine Sammlung ehrwürdiger, aber doch reichlich primitiver Legenden. Keine noch so feinsinnige Auslegung kann etwas daran ändern.
    Ich glaube an Spinozas Gott, der sich in der gesetzlichen Harmonie des Seienden offenbart, nicht an einen Gott, der sich mit Schicksalen und Handlungen der Menschen abgibt.
    Was ich in der Natur erblicke, ist eine großartige Struktur, die wir nur bruchstückhaft verstehen können. Diese Struktur muss jedem denkenden Menschen ein Gefühl von Bescheidenheit vermitteln – ein authentisches religiöses Gefühl, das mit Mystizismus nichts zu tun hat.
     



 

  Beitrag 1924-1
Wie aus Platonischen Ideen (sämtliche) anfassbare Materie wird

 
 

Erst Platonische Ideen machen Energie zu anfassbarer Materie


In Anhang 14 seines Buches "Versteckte Wirklichkeit" erklärt Lothar Schäfer, wie es zum Pauli-Prinzip kommt:

Es ist Folge der Tatsache, dass Elementarteilchen gleichen Typs ununterscheidbar sind und ihre Wellenfunktion deswegen gewisse Symmetrie-Eigenschaften hat. [Es gibt für dieses Identitätsprinzip keinen Beweis, doch wird es als Axiom allgemein anerkannt.

Wie Schäfer durch recht einfache Argumente zeigt, folgt aus dem Identitätsprinzip, dass die Wellenfunktion eines Systems von N gleichen Teilchen beim Vertauschen von zwei Teilchenzuständen höchstens ihr Vorzeichen ändert, ansonsten aber gleich bleibt.

Je nachdem, ob Vertauschen zweier Teilchenzustände das Vorzeichen ändert oder nicht, nennt man die Wellenfunktion antisymmetrisch bzw. symmetrisch.
In der Natur beobachtet werden beide Fälle: Teilchen, deren Spin durch ganzzahlige Quantenzahlen bestimmt ist, haben symmetrische Wellenfunktion, alle anderen haben antisymmetrische.

Da nun aber jedes Quantum durch nur 4 Zahlen (n = Energie, l = Bahnelement, m = Richtung des Bahnelements, s = Richtung des Spinmoments) eindeutig charakterisiert ist, erkennt man: Jeder über so ein Quadrupel gegebene Zustand eines Elektrons kann durch ein Elektron besetzt sein oder nicht, kann aber nie mit mehr als nur einem besetzt sein.
    Beweis: Wenn nämlich zwei Elektronen denselben Zustand hätten, würde Zustandsvertauschung die Wellenfunktion nicht ändern im Gegensatz zur Tatsache, dass Elektronen (als Fermionen) antisymmetrische Wellenfunktion haben.

Aus verschiedenen Gründen, die Blochinzew in "Grundlagen der Quantenmechanik" (1963) erörtert hat, gilt diese Formulierung des Pauliprinzips nur näherungsweise: Der Begriff "Zustand eines einzelnen Elektrons in einem N-Elektronen-System" ist nämlich nur unscharf definiert, da ein Heraustrennen des Zustandes eines Elektrons ohne Änderung des Gesamtsystems unmöglich ist. Genauer formuliert sagt das Pauliprinzip: "Die Wahrscheinlichkeit, in einem System von Spin-1/2-Teilchen zwei zu finden, für die die Messergebnisse aller den Teilchenzustand charakterisierenden charakteristischen Größen gleich sind, ist null."

Für die Elektronen eines Atoms folgt daraus, dass sie nicht alle den energiemäßig vorteilhaftesten Zustand besetzen können, denn dieser Grundzustand (n,l,m) = (1,0,0) ist voll besetzt, sobald ihn zwei Elektronen mit unterschiedlichem Spinmoment eingenommen haben. Wenn ein Atom also mehr als nur 2 Elektronen hat, so müssen diese — wie auf einer Stufenleiter — energetisch immer höhere Zustände besetzen: (2,0,0), (2,1,0), (2,1,1), (3,0,0), usw.

Daraus resultiert eine elektronische Struktur, welche Grundlage des Periodensystems der Elemente ebenso wie aller chemischen Gesetze ist.

Wichtig ist nun (siehe H. Margenau in "The Miracle of Existence", 1963):

Die Vermeidung besetzter Zustände ist NICHT eine Folge elektrostatischer Abstoßung — wie man meinen könnte —
oder irgendeiner anderen mechanischen Eigenschaft,

sondern beruht lediglich auf der Antisymmetrie der Wellenfunktionen der Elektronen (!).


Wenn nun zwei Moleküle oder Gegenstände weit voneinander entfernt sind, kann man für alle praktischen Anwendungen davon ausgehen, dass ihre Wellenfunktionen ψA und ψB unabhängig voneinander sind.

Erst wenn diese Objekte einander sehr nahe kommen (ihr Abstand etwa die Größenordnung 10-10 m erreicht), kommt es zu nicht mehr vernachlässigbarer Interferenz beider Wahrscheinlichkeitswellen, so dass man dann nicht mehr von zwei getrennten Zuständen sprechen kann. Die durch diese neue Wellenfunktion definierten Zustände des Gesamtsystems können stabilisierende oder destabilisierende Wirkung haben. Auf jeden Fall entdecken die im System vorhandenen Elektronen sofort, wo sie sich auf Zustände höherer Energie zurückziehen müssen, weil vorteilhaftere schon besetzt sind.

Nach diesem Prinzip enstehen auch die abstoßenden Kräfte zwischen zwei Molekülen oder anderen Gegenständen, die sich hinreichend nahe kommen. Der einzig und allein aus der Symmetrie-Eigenschaften der Wellenfunktion kommende Zwang, besetzte Zustände zu meiden, ruft dann physische Kräfte hervor: Platonische Ideen — die Symmetriegesetze — erzeugen dann also fühlbare Kraft, und so entsteht aus etwas, das nur gedanklich existiert, in der Tat anfassbare Materie — z.B. der Stein, von dem in Beitrag 949-59 die Rede war.


Deswegen sage ich mir:

Die durch uns bisher übersehene, in der Raumzeit fehlende zusätzliche Dimension unserer Welt ist gar nicht wirklich transzendent — es ist eine rein geistige Dimension, die aus sämtlichen mathematischen Gesetzen besteht, deren einige man als Eigenschaften mathematischer Objekte gut kennt (z.B. die Symmetrie-Eigenschaften der Wellenfunktionen).

Neu ist das alles nicht, denn schon 1995 schrieb Hans-Peter Dürr, vormals Leiter des Max-Planck-Institutes für Physik und Astrophysik in München:

... An manchen Stellen verdickt sich der Geist, gerinnt, und wird zu dem, was wir die Materie nennen.
Materie ist geronnener Geist, ...


Nüchterner und genauer ausgedrückt:

Der physische Teil unserer Welt wird geschaffen, geformt, und regiert durch nur gedanklich Existierendes.


Gebhard Greiter (grtgrt)
einer Darstellung von Lothar Schäfer folgend

 

  Beitrag 1995-1
Inwiefern anfassbare Gegenstände nur Illusion sind

 
 

Wie es zur Illusion anfassbarer Gegenstände kommt


Jeder Gegenstand G, den man anfassen und fühlen kann, ist eine Konfiguration von Elementarteilchen.

Jedes dieser Teilchen existiert zunächst nur virtuell, d.h. noch nicht mal an einem genau definierten Ort: Es existiert nur als ein Energiepaket T, für das die Wellenfunktion ψ unseres Universums U zu jedem Punkt P der Raumzeit eine Wahrscheinlichkeit w( P,T ) dafür nennt, dass T in Punkt P als Teilchen beobachtbar wird. Wo solche Beobachtung dann tatsächlich stattfindet, bedeutet das, dass T mit wenigstens einem anderen Teilchen in dem Sinne kollidiert, dass beide miteinander verschmelzen und aus dieser Verschmelzung sehr oft neue Elementarteilchen entstehen.

Das Entstehen zweier Teilchen T aus dem Nichts ebenso wie die eben beschriebene Kollision von Elementarteilchen nennt man ein Elementarereignis E. Versteht man unter input(E) bzw. output(E) die Menge aller durch E vernichteten bzw. neu erzeugten Elementarteilchen, so kann maximal eine dieser beiden Mengen leer sein. Auf jeden Fall aber haben beide identischen Gesamtimpuls.

Der spontane, plötzliche Übergang von input(E) zu output(E) entspricht einer winzigen Äbänderung von G, die sich bemerkbar macht
  • einerseits durch aus G kommende Strahlung – Licht etwa –
  • und andererseits über eine leichte Abänderung der Kräfte, die zwischen den G darstellenden Elementarteilchen wirken in dem Sinne, dass Teilchen, die Ruhemasse haben, sich nicht beliebig nahe kommen können, dass es ihnen aber umgekehrt auch ziemlich schwer fällt, sich allzu weit voneinander zu entfernen.

Mit anderen Worten:

Ständig in G eindringende Strahlung (wenigstens die allgegenwärtige kosmische Hintergrundstrahlung) ist für die meisten in G stattfindenden Elementarereignisse E verantwortlich. Da für sehr viele der in G stattfindendes Elementarereignisse E die Menge output(E) Teilchen enthält, die aus G als Strahlung entweichen — sehr oft als sichtbares Licht —, wird G über sie durch unsere Sinne — entweder direkt oder über geeignet konstruierte Detektoren — beobachtbar.

Mehr noch: Die zwischen den G darstellenden Materieteilchen wirkenden Kräfte führen zu einem Kräftegleichgewicht, welches — da es ja durch jedes Elementarereignis nur ein klein wenig abgeändert wird — zur Folge hat, dass G seine Form i.A. nur langsam ändert (und dass Widerstand spürt, wer den Gegenstand G berührt oder gar versucht in wegzuschieben oder zusammenzudrücken).

Diese Sinneswahrnehmungen also sind es, die — aufsummiert durch unser Gehirn — zu dem führen, was wir als einen uns sichtbaren oder durch uns berührbaren Gegenstand begreifen.


Gebhard Greiter (grtgrt)
 

  Beitrag 1995-9
Leben wir als Teil einer nur errechneten (simulierten) Welt?

 
 
Zara.t. aus 1995-7:
Heute weiß man, dass die Leere des Vakuums alles andere als leer ist. Jeder Punkt der Raumzeit ist eine Überlagerung von möglichen Teilchen. Besser: von virtuellen Teilchen, die nicht direkt gemessen werden können und von Möglichkeiten "reale" Teilchen zu messen.

Hi Zara,

genau so hat es mein Beitrag 1995-1 ja auch beschrieben.


Zara.t. aus 1995-7:
 
Rutherford hat noch vollkommen klassisch gedacht. Dieses Atommodell konnte nicht funktionieren. Elektronen, die wie Planeten die Sonne, den Kern umkreisen müßten in den Kern stürzen. Auch aus diesem Problem heraus entwickelte sich die Quantenmechanik.


Natürlich: Rutherfords Atommodell war noch nicht so genau, wie wir es heute wissen. Dennoch erscheint mir sein Vergleich recht erhellend.

Deine Aussage aber, dass sich "aus diesem Problem" die Quantenmechanik entwickelt hätte, stimmt so nicht. Ursache ihres Entstehens war die Entdeckung des Wirkungsquantums durch Max Planck (1899, im Dez 1900 der Fachwelt verkündet).

Nebenbei: Anton Zeilinger vermutet, dass die Quantelung aller physikalischen Größen und Prozesse darauf zurückzuführen sein könnte, dass unsere ganze Welt letztlich nur aus Information besteht und die sich eben nur durch Bitfolgen kodieren lässt.

Hätte er recht (und in dem Fall wäre sogar Energie nur Illusion), wäre es gar nicht mehr so abwegig, auch die Möglichkeit in Betracht zu ziehen, dass unser ganzes Universum — und damit sogar wir selbst — vielleicht nur Teil einer gigantisch angelegten Computer­simulation sein könnten (so wie Brian Greene das, als nicht wirklich ausschließbare Möglichkeit, allen E...es in Kap. 10 seines Buches "The Hidden Reality" ja auch schon diskutiert und auf Plausibilität hin zu durchdenken versucht).


Gruß, grtgrt


PS: Anton Zeilinger und Andreas Mücklich gehen noch weiter, indem sie vermuten, dass die Natur jedem physikalischen Objekt nur endlich viele Bits zugesteht, sich selbst zu beschreiben. Dies würde begründen, warum Heisenbergs Unschärfe-Relation gilt: Je genauer eine Eigenschaft des Objekts beschrieben ist, desto mehr jener Bits sind hierfür verbraucht, und desto weniger stehen zur Verfügung, dazu komplementäre Eigenschaften des Objekts wirklich genau zu beschreiben. Wenn das Objekt dann, wie durch Zeilinger vermutet, nur aus seiner Beschreibung besteht, ist klar, dass die Unschärfe in Wirklichkeit  U n b e s t i m m t h e i t  ist: Wirklich scharf also kann kein Objekt existieren.

Wäre unsere Welt nur eine simulierte (d.h. errechnete), müsste man jene begrenzt langen Bitfolgen als die Variablen sehen, die im simulierenden Programm das physikalische Objekt darzustellen haben. Dies würde erklären, warum sie begrenzt sind.


Nach Zeilinger lässt sich das keineswegs nur für Elementarteilchen so sehen, sondern auch für beliebig komplexe Objekte — beispielsweise für ganze Messapparaturen.
Er zeigt das am Beispiel solcher, die einen Mach-Zehnder-Interferometer nutzen, mit dem man für Photonen zwei Arten von Information sammeln kann — niemals aber Information beider Art zugleich: Diese Arten von Information sind zueinander komplementär im Sinne der Unbestimmtheitsrelation (wie Zeilinger auf Seite 202 seines Buches Einsteins Schleier recht überzeugend darlegt).

 

  Beitrag 1995-16
-

 
 
Zara.t. aus 1995-14:
Grtgrt aus 1995-1:
Jeder Gegenstand G, den man anfassen und fühlen kann, ist eine Konfiguration von Elementarteilchen.

Dieses Postulat müßte untersucht werden. Es ist auch als Lego-Weltbild bekannt. Und eigentlich als unhaltbar erkannt. Aber noch läßt sich darüber streiten.
Mein Postulat:

Ein Gegenstand kann nicht aus Quantenobjekten bestehen. Ein Gegenstand ist keine Ansammlung von Elementarteilchen.

Hi Zara,

du hast insofern recht, als dass ein Gegenstand, der aus Lego-Bausteinen zusammengesetzt ist, keinen dieser Steine verändert (bzw. seiner Gestalt nach irgendwie aufweicht).

Bei einer Quantenkonfiguration ist das anders: Hier ergibt sich tatsächlich eine Aufweichung, die dazu führt, dass jedes Teilsystem einer solchen Konfiguration einen Teil seiner Selbständigkeit und rollenspezifischen Identität aufgibt. Gutes Beispiel sind Moleküle, die aus mehr als nur einem Atom bestehen: Es gibt darin stets mindestens ein Elektron — und damit auch mindestens ein Orbital — welches sich NICHT mehr nur einem einzigen der jener Atome zuordnen lässt.

Mathematisch ausgedrückt: Die Wellenfunktion einer Quantenkonfiguration ist eben NICHT einfach die Summe der Wellenfunktionen, die sich ergäben, wenn man jedes Quant einzeln betrachtet als isoliertes System sehen würde.

Beste Grüße,
grtgrt
 

  Beitrag 1995-18
-

 
 
Henry aus 1995-17:
Ich denke, das ein Gegenstand mehr ist als eine Sammlung von Quantenobjekten / Elementarteilchen.

Das sage ich ja: siehe 1995-16 ( zeitgleich mit deinem Einwand veröffentlicht ).


 

  Beitrag 1995-25
-

 
 
Hi E...,

danke für den Link hin zum Interview mit Anton Zeilinger.

Besonders zwei Definitionen daraus sollte man sich auch ihrer Formulierung nach gut merken:
  • Der quantenmechanische Zustand ist die Information, die wir über die Welt haben [so die Kopenhagener Interpretation
     

  Beitrag 1995-27
-

 
 
E... aus 1995-23:
... komplexe makroskopische Systeme sich in  k e i n e m  Quantenzustand befinden und deshalb auch  k e i n e  Illusion sein können.

Quelle: http://www.heise.de/tp/artikel/7/7550/1.html

Hi E...,

diese deine Aussage ist natürlich falsch, und das in gleich zweierlei Hinsicht. Hier mein Beweis:
  • Da Zeilinger im durch dich gefundenen Artikel sagt, der quantenmechanische Zustand eines Systems sei die Information ist, die wir über das System haben, ist jedes makroskopische, durch uns beobachtbare System doch ganz klar in einem solchen Zustand.
  • Als was uns das System erscheint (sein "Erscheinungszustand", die "Illusion" also) wird hervorgerufen durch uns erreichende Quanten, die — erzeugt durch sein ständiges dekohärent werden — von jenem Objekt ausgehen. Es sind Quanten, die die entsprechenden Elementarereignisse sozusagen "aus dem Objekt heraus­schlagen" (genauer: aus einer Verschmelzung des Objekts mit den Quanten seiner Umgebung, die es dekohärent machen und so auch mit seiner Umgebung verschränken können).

Gruß, grtgrt
 

  Beitrag 1995-58
-

 
 
Hi Stueps,

dass das, was ich eine Illusion nenne, tatsächlich eine ist, finde ich bestätigt im letzten Satz der folgenden Aussage von Thomas Görnitz (nachzulesen auf Seite 110 seines Buches "Die Evolution des Geistigen"):

Zitat von Görnitz:
 
Bei größeren Systemen, wenn sie sich nicht fast am absoluten Nullpunkt befinden, treten ständig Photonen einer Wärmestrahlung aus, die zu der betreffenden Temperatur gehört. Damit besteht ein ständiger Informationsverlust, sofern nicht beispielsweise durch ideale Spiegel die Information zurückgeschickt werden würde. Da größere Objekte ständig strahlen, wird ein makroskopisches Objekt uns meist wie etwas Faktisches  v o r k o m m e n .

Das Wort "vorkommen", auf das ich dich aufmerksam machen möchte, habe nur ich im Zitat optisch hervorgehoben. Es bedeutet nichts anderes, als dass unsere Sinne uns hier eine Illusion generieren. Es ist die (bzw. ein Teil der), die ich in meinem Beitrag 1995-1 meine (und die du mir nicht glauben willst).

Gruß, grtgrt
 

  Beitrag 1995-48
Klassische Existenz physikalischer Objekte ist eine Illusion

 
 
Stueps aus 1995-43:
 
Es ist meines Wissens nach unter Wissenschaftlern nicht die Frage, dass physikalische Systeme in klassischer Form existieren, sondern wann sie klassisch werden.

Hi Stueps,

physikalische Objekte existieren nicht in klassischer Form — sie  z e i g e n  sich nur in dieser Form (dann nämlich, wenn ein Elementarereignis passiert und zur Folge hat, dass aus dem Objekt Quanten in seine Umgebung entkommen. Sie sind es, was wir sehen bzw. registrieren, und so den Eindruck gewinnen, das Objekt selbst sei sichtbar und würde klassisch existieren).

Die Tatsache, dass pro Sekunde wahnsinnig viele solche Ereignisse passieren, suggeriert uns den Eindruck eines stehenden "Bildes" (eben ganz so, wie wir ja z.B. auch das durch eine gute Fernsehkamera erzeugte Bild eines sich gerade NICHT bewegenden Gegenstandes als stehend empfinden, obgleich doch in Wirklichkeit all die Pixel, aus denen es sich zusammensetzt, pro Sekunde öfters neu geschrieben werden als unsere Augen noch unterscheiden können).

Gruß, grtgrt
 

  Beitrag 2016-136
Welle oder Teilchen?

 
 
Henry aus 2016-132:
 
Es gibt keine Welle, die sich ausbreitet. Das Teilchen ist ein Teilchen, aber es zeigt bei entsprechenden Experimenten Auswirkungen, die eigentlich nur von Wellen kommen können, eben z. B. Interferenzen. Google einfach mal "Doppelspaltexperiment".


Diese Aussage scheint falsch. Richtig ist eher das Gegenteil, denn:

Feynman hat mal geschrieben (leider erinnere ich mich nicht mehr wo): Jedes Photon, das aus einer punktförmigen Lichtquelle kommt, ist eine Kugelwelle, die sich mit Lichtgeschwindigkeit ausbreitet. Erst wenn sie auf einem Detektor trifft, der sie registriert, suggeriert sie dem Beobachter, sie sei ein Teilchen.

Er schrieb weiter: Wenn im Raum um die Lichtquelle herum mehrere Detektoren aufgestellt werden, kann nicht vorausgesagt werden, mit welchem sie interagieren wird, d.h. an welcher Stelle das Photon sich als Teilchen zeigen wird.

Ein gewisser Heinz Heinzmann arbeitet gerade an einem Buch, in dem er der Welt klar zu machen versucht, dass die wahre Natur aller Quanten von der Art "stehende Welle" ist, und dass, was wir als "Teilchen" interpretieren, nur eine Stelle ist, an der sich so ein Wellenzustand umkonfiguriert. (Siehe etwa Seite 46 und 47 seines Papiers).

Heinzmanns Text finde ich nicht besonders klar, er hat mich aber sofort an Feynmans Aussage erinnert.

 

  Beitrag 2032-53
Teilchen erst durch Interaktion

 
 
Hans-m aus 2032-51:
Bauhof aus 2032-50:
 
Licht wird nicht als gleichförmiger Teilchenstrahl beschrieben, sondern als Schwingungsvorgang.

Licht kann sowohl als Welle als auch als Teilchen auftreten. es bewegt sich in beiden Erscheinungsformen gleichförmig mit 300 000 km/s
Das kann man messen.


Hallo Hans-m,

Licht bewegt sich keineswegs als Teilchen: Licht breitet sich stets nur als Welle aus.

Erst wo diese Welle mit anderen Wellen interagiert (und etwas entsteht, das eine Messapparatur aufzufangen in der Lage ist) stellt sich Licht auch als Teilchen dar.

Das einzusehen ist von fundamentaler Bedeutung, u.A. auch deswegen, weil so klar wird, dass sich Elementarteilchen (als Teilchen) nur mit gewisser Wahrscheinlichkeit an gewisser Stelle beobachten lassen.

Gruß, grtgrt

siehe auch Beitrag 2016-136.

 

  Beitrag 2016-140
-

 
 
Rockdee aus 2016-137:
Wenn das stimmen sollte grtgrt, dann habe ich mal ne Frage dazu:

Wenn eine Welle sich mit Lichtgeschwindigkeit in alle Richtungen ausbreitet, und man selbst wäre nun Beobachter, wie würde man die Welle wahrnemen?
Als Teilchen, denn Einsteins Theorie besagt doch, dass etwas das sich mit Lichtgeschwindigkeit bewegt verkürzt erscheint.
Wenn es sich nun in alle Richtungen mit so einer geschwindigkeit bewegt, erscheint es dann als Punkt/Kugel bzw Teilchen?

Falls das so wäre würde das doch erklären können, warum das Teilchen sich unbeobachtet wie eine Welle verhält, da es 'kein Bezugssystem' hat.
Sobald man es aber Betrachtet, ein 'Bezugssystem einfügt' muss es demzufolge anders erscheinen.
Da der Beobachter außerhalb des Systems der Welle ist.


Warum sollte man eine Welle als Teilchen wahrnehmen?

Was man als Teilchen (oder Materie) wahrnimmt ist stets nur Interaktion der Welle mit physikalischen Objekten (Kraftwirkung oder in Folge solcher Interaktion abgestrahlte elektromagnetische Wellen: Licht, Gammastrahlung, etc.).

Materie, und wohl auch alles, was man als "Teilchen" zu beobachten glaubt, ist nur Wirkung von Kraft (siehe dazu Beitrag 1995-1 und Beitrag 1995-68).


Zum Bezugssystem: Alles, was sich hinsichtlich einem Bezugssystem mit Lichtgeschwindigkeit bewegt, hat eben dieselbe Geschwindigkeit auch hinsichtlich jedem anderen Bezugssystem. Zwei entsprechende Beobachter würden höchstens verschiedene Wellenlänge feststellen.

 

  Beitrag 2039-1
Prä-Materie und Emergenz — Hans-Peter Dürrs Erkenntnisse (und sein Weltbild)

 
 
Wie es zur Illusion anfassbarer Objekte kommt, und wo der Anfang von Leben zu finden sein könnte, wurde hier im Forum schon mehrfach gefragt und zu beantworten versucht.

Eben aber finde ich in Hans-Peter Dürrs Buch "Geist, Kosmos und Physik" einige ganz besonders interessante Feststellungen dazu. Sie sind Kern seines modernen, holistischen Weltbildes, welches sich auf die Erkenntnisse der Quantenphysik gründet:


Zitat von Dürr (S. 36-37, etwas gekürzt):
 
Die neue Weltsicht ist im Grunde holistisch, nicht atomistisch: Es existiert eigentlich nur das Eine, das Ungetrennte, das Untrennbare. ...

Das untrennbare Eine ist Prozesshaftes, Potentialität, aber nicht nur Möglichkeit, sondern auch das Vermögen zur Schaffung von Realität und von greifbar Seiendem [anfassbaren Objekten.

Die zeitliche Evolution besteht in einem fortschreitenden Prozess der Differenzierung dieses Untrennbaren durch Errichtung von Grenzzäunen (physikalisch: auslöschende Überlagerung von Potentialwellen).

Man ist an Zellteilung erinnert, wo sich eine Zelle ja auch vermehrt durch Neubildung von Zellwänden.

Dies imitiert die Entstehung unabhängiger Subsysteme, die als Teile des Gesamtsystems fungieren und aus denen dieses Gesamtsystem "zusammengesetzt" erscheint. Dies ist aber nie der Fall, weil der Zusammenhang viel tiefer geht, so wie etwa die sichtbar getrennten weißen Schaumkronen auf stürmischer See ja auch nicht die Betrachtung rechtfertigen, das Meer sei aus Wellen und Schaumkronen zusammengesetzt.

Das Sinnstiftende im Zusammenwirken der Als-ob-Teile entsteht immer aus dem Ganzen, das sie einschließt. Dieses Ganze, Eine, ist immer da.

Auch wir, die wir alle hier im Raum leben, sollten uns nicht vorstellen, dass wir wirklich getrennte Teile dieser Wirklichkeit sind, lose zusammengehalten durch einige Licht-, Laut- und andere von der Physik identifizierbaren Signale, die wir uns zur Verständigung wechselseitig zuwerfen. Wir sind alle Teile dieses selben Einen, derselben Potentialität, und spüren das auch: Wie sonst nämlich könnten ein paar hingeworfene Worte und Sätze mit ihrem dürftigen, abzählbaren Informationsgehalt sich in unserem jeweiligen Bewusstsein so reich entfalten.
 


Hier wird ganz klar deutlich, dass alles materiell Existierende seiner wahren Natur nach nur Wellenpaket ist.

Und so schreibt Dürr denn auch:


Zitat von Dürr, S. 44:
Ich habe als Physiker 50 Jahre lang — mein ganzes Forscherleben — damit verbracht zu fragen, was eigentlich hinter der Materie steckt. Des Endergebnis ist ganz einfach:

Es gibt keine Materie!

 


Interessant ist ferner wie sich Dürr vorstellt,
  • dass es zu Leben kam,
  • dass materielle Objekte, Lebewesen und Anderes, nur endlich langes Leben haben und
  • wie sich Emergenz erklärt (er benutzt dieses Wort nicht, erklärt ihr Zustandekommen aber wenigstens ansatzweise):


Zitat von Dürr, S. 39-42, einiger Kürzungen wegen nicht ganz wörtlich:
 
Unsere Mesowelt ist eine statistisch ausgemittelte Mikrowelt (vergleichbar einem Ameisenhaufen, der von Ferne wie ein statischer Hügel aussieht, der beim genauen Hinsehen aber ungeheuere Beweglichkeit zeigt: Dass sich dies Gewimmel nicht auch im Großen ausprägt, liegt daran, dass für jede Ameise, die in einer Richtung läuft, es immer auch eine andere gibt, die das Umgekehrte macht, weshalb dann im Durchschnitt keine Bewegung des Ganzen sichtbar ist).

Dass diese Ausmittelung so vollständig gelingt, liegt wesentlich am 2. Hauptsatz der Thermodynamik, welcher besagt, dass in einem sich selbst überlassenen System jede Besonderheit, jedes Ausgezeichnetsein, im Laufe der Zeit zerstört wird (man denke an einen Schreibtisch, der, wenn wir nicht aufräumen, immer unordentlicher wird).

Deshalb verstehen wir nicht, wie es in der Natur mit ihrem starken Hang zur Unordnung überhaupt dazu kommt, dass sich bei der Evolution hochdifferenzierter Systeme (wie uns Menschen etwa) Unordnung über lange Zeit hinweg hinweg nicht durchsetzen kann.

Was also ist da passiert? Hat die Natur für ihren lebendigen Teil nicht vielleicht doch bei einer höheren Instanz eine Ausnahmeregelung den Zweiten Hauptsatz betreffend erwirkt?

Nach heutiger Einsicht scheint es keine solche Ausnahmeregelung zu geben. Die unbelebte wie die belebte Natur basieren auf derselben Art von Prä-Materie, die im Grunde eigentlich keine Materie ist. Sie kann sich auf verschiedene Weise organisieren:
  • Einmal ungeordnet und unkorreliert. Dann wird das resultierende Gesamtsystem stumpf, langweilig, apathisch (und wir nennen es unbelebte Materie).
  • Prä-Materie kann sich aber auch auf differenziertere, raffiniertere Weise formieren. Es entstehen dann Stukturen, in denen das im Grunde embryonal Lebendige selbst noch in der Mesowelt zum Ausdruck kommt und so lebendiger Organismus wird. Die eingeprägte Potentialität wird makroskopisch sichtbar. Das Gesamtsystem muss dazu weit weg von seinem Gleichgewichtszustand sein, um ein Ausmitteln seiner inneren Lebendigkeit zu vermeiden.
    Stellen Sie sich ein physikalisches Pendel vor (als herabhängenden, beweglichen Stab mit einem Gewicht unten). Es pendelt beim Anstoßen vorhersehbar und berechenbar um seine unter stabile Gleichgewichtslage. Dreht man aber Stab und Gewicht weit weg von unteren, stabilen Gleichgewicht nach ganz oben, so gibt es dort eine weitere Gleichgewichtslage. Sie ist instabil, und so wissen wir nicht, ob das Pendel auf die eine oder die andere Seite fallen wird. In diesem Instabilitätspunkt wird die inhärente Lebendigkeit des Systems sichtbar, weil es von winzig kleinen Unterschieden abhängt, ob der Pendel zum einen oder zum anderen Bewegungsablauf veranlasst wird. Die Naturwissenschaft kennt viele Systeme mit solch eingeprägten, dynamischen Instabilitäten. Sie führen zu, wie man sagt, "chaotischem" Bewegungsverhalten: Kleine Veränderungen in den Ursachen bewirken extrem große Unterschiede in den Folgen: Der Schlag eines Schmetterlings kann einen Taifun auslösen.

Leben — belebte makroskopische Oranismen — erfordern Strukturen in der Nähe inhärenter Instabilitäten. Aber Instabilitäten kippen. Um sie also lange in der Balance zu halten, müssen sie dauernd nachjustiert werden durch etwas, das sie neu austariert (intelligente Zuführung von Energie).

Diese Situation steht nicht im Widerspruch zum 2. Hauptsatz der Thermodynamik (d.h. zur allgegenwärtigen, dominanten Tendenz zur Unordnung). Denn es ist ja auch unsere ordnende Hand, die unseren Schreibtisch immer wieder in Ordnung bringen kann. Sie darf dabei aber nicht nur werkeln; sie muss darauf achten, was sie tut: Sie muss intelligent sein, den sonst beschleunigt sie nur den Prozess hin zur Unordnung.

Lebendige Systeme brauchen deswegen ... Intelligenz, eine geistige Führung, die prinzipiell im immateriellen Form-Grund verankert ist und sich in der Milliarden Jahre langen Evolution des Biosystems durch ein Plus-Summen-Spiel in komplexen Verästelungen immer höher entwickelt hat.
 

 

  Beitrag 2039-11
Ein Gleichnis

 
 
Den teilweise recht emotionsgeladenen Bemerkungen von Stueps, Henry und E... möchte ich  folgendes Beispiel  entgegensetzen:

Wenn jemand eine Karnevalssitzung oder einen Faschingsball besucht und sich dort als Neptun verkleidet zeigt, bedeutet das noch lange nicht
  • dass der Meeresgott tatsächlich existiert
  • oder dass jene Person auch überall dort, wo sie sich sonst noch lebt und mit anderen interagiert, als Neptun eingestuft sein möchte.

Bei einem Paket von Potentialwellen ist es ebenso:

Es existiert im gesamten Raum, hat aber nur in einer ganz kleinen Teilregion davon (als ortsabhängige Funktion betrachtet) so großen Absolutwert, dass das Plancksche Wirkungsquantum ihm gestattet sich dort — und NUR dort — im "Faschingskostüm Materie" zu zeigen.

Das Schienbein, von dem Stueps spricht, ist also nur dort sichtbar und anfassbar, es zeigt sich nur dort verkleidet als Materie, wo das Wirkungsquantum ihm gestattet, mit seiner Umgebung zu interagieren (zu "wirken"). Solche Wirkung erzeugt Lichtwellen, die uns das Schienbein sichtbar machen, und erzeugt Kraft, die uns glauben macht, wir könnten es fühlen und anfassen.

Das Schienbein als Materie ist also nicht identisch mit dem Schienbein als Wellenpaket.
  • als Wellenpaket existiert jenes Schienbein praktisch im gesamten Universum,
  • als Materie aber ist es nur Anschein: Es will uns suggerieren, Neptun (= Materie) zu sein, Neptun aber existiert nur in des Betrachters Phantasie.
 

  Beitrag 2039-2
Unsere Welt als Wahrnehmung und als Wirklichkeit

 
 

Unsere Welt als Wahrnehmung und als Wirklichkeit



Dürr macht uns klar, dass wir nicht mehr umhin können,

die  w a h r g e n o m m e n e n  Welten von der  w i r k l i c h e n  zu unterscheiden:


Zitat von Dürr:
 
Wenn wir von der Welt sprechen, in sie hineinsehen, dann vergleichen wir unsere Bilder miteinander und sind oft verschiedener Meinung. Wir machen den Fehler zu glauben, dass das, was jeder von uns in dieser Welt sieht, dasselbe sei, das auch der andere sieht. Aber es ist durch unsere spezielle Wahrnehmung gefiltert und deformiert. Denn dort, wo wir sehr empfindlich sind, nehmen wir mehr wahr, und dort, wo wir unempfindlich sind, nehmen wir überhaupt nichts wahr.

Die wahrgenommene Welt ist demnach eine ganz andere als die da draußen (die wirkliche).


Ein Hauptstreit, den wir untereinander führen, kommt daher, dass wir nie in Betracht ziehen, dass wir verschiedene Dinge sehen, weil wir auf verschiedene Weise sensibilisiert sind. Es ist uns gar nicht bewusst, was wir weglassen, uns dazu denken, oder wirklich echt sehen.

Keiner von uns merkt z.B. dass wir in unserem Auge, in unserem Gesichtsfeld, einen schwarzen Fleck haben. Wir sind gewöhnt daran, ihn ohne Nachzudenken mit Erinnerungen oder Schlussfolgerung zu übermalen, mit dem, was da gar nicht hingehört. Wer wirklich einen schwarzen Fleck sieht, gehe besser zum Augenarzt, denn da ist irgendetwas nicht in Ordnung. Wir alle haben diesen schwarzen Fleck, aber niemand spricht davon, weil wir alle unbewusst gelernt haben, den Mangel an Sehfähigkeit — diesen schwarzen Fleck — durch gespeicherte Information zu kompensieren.


Und so kommt es dass, wo fast jeder Materie sieht, die Quantenphysiker nur Potenzialschwankungen sehen und zunehmend nur noch in Gleichnissen sprechen können.
Die wirkliche Welt zu sehen bedeutet, einzusehen, dass Materie nur in der wahrgenommen Welt existiert, in Wirklichkeit aber (als Erscheinung) Ausmittelung sich ständig auf- und abbauender Kräfte ist: sich ständig neu gestaltende strukturierte Form.


Zitat von Dürr:
 
Diese Gestalt hat keinen Ort, an dem sie sich befindet; sie ist sozusagen über die ganze Welt ausgebreitet. Es gibt überhaupt keine Auflösung in Teile. Auch ist in der Physik die Wirklichkeit nicht Realität, sondern Potentialität. Sie ist nur Möglichkeit, die sich energetisch und materiell irgendwo manifestieren kann, sozusagen etwas noch nicht Entschiedenes, Schwebendes. Und diese Potentialität ist räumlich nicht lokalisiert.

Das führt dazu, dass die ganze Welt keine Ränder hat. Es gibt nur das Eine, und wir könnten sagen, das Ganze. Das "Ganze" aber ist auch nicht das richtige Wort. Es wäre dann ja etwas, dem kein Teil fehlt. Aber wenn es gar keine Teile gibt, dann können wir es auch nicht das Ganze nennen.


Dürrs Einsicht:

Unsere Welt — dieses Eine (mit uns darin) — lässt sich ebenso wenig in Einzelteile zerlegen,

wie die wogende See sich in einzelne Wellen und Schaumkronen zerlegen lässt.


Zitat von Dürr:
 
Das hat fantastische Konsequenzen und heißt beispielsweise, dass wir alle, die wir hier im Raum sitzen, zwar unterschiedlich und unterscheidbar, aber nicht getrennt sind. Wir befinden uns sozusagen in dieser Gemeinsamkeit, und das ist eine wesentliche Voraussetzung, dass wir überhaupt miteinander kommunizieren können.


In der Summe, so Dürr, sei festzuhalten:
  • Materie ist im Grunde nicht Materie: Was wirklich existiert, ist nur Form (strukturierte Gestalt).
  • Diese Gestalt ist nicht lokalisiert — sie ist gleichermaßen überall und macht die Welt unteilbar. Wo wir sie beschädigen, beschädigen wir uns selbst (!).
  • Die Zukunft ist offen, aber dennoch nicht völlig beliebig (da vom Vorausgegangenen beinflusst).
  • Unsere Welt (die wirkliche) kennt echte Kreativität, denn aus Nichts kann etwas entstehen.
  • Was entstanden ist, kann auch wieder ins Nichts verschwinden.

 

  Beitrag 1376-1
Was ist Zeit?

 
 
Auch wenn ich absolut fasziniert von dieser Seite bin ( ein Kompliment an die Verantwortlichen ), habe ich mir schon oft die Frage gestellt: "Was ist Zeit eigentlich?"
Mit den bisher veröffentlichen Theorien und wissenschaftlichen Meinungen kann ich aber leider nicht besonders viel anfangen.
Ich bin dann irgendwann zu meiner persönlichen Erkenntnis gekommen, daß es Zeit NICHT gibt, sondern daß sie nur eine Erfindung der Menschen ist.
Alles bleibt so wie es ist. Der Mensch hat die Zeit erfunden, um sichtbare Veränderungen zu beschreiben, um seinen Tagesablauf zu gestalten oder einfach nur um sein Dasein zu definieren.
 

  Beitrag 1376-28
Lebenszklus, Lebensbatterie, Entropie

 
 
Hi Stueps,

meine erste Antwort auf deine Anregung das Stichwort Entropie betreffend war falsch: Die Lebensbatterie eines Objektes lässt sich doch über Entropie definieren. Genauer: Sie definiert den — mit Hilfe der Entropiegesetze gut präzisierbaren — Freiraum, den ein Objekt hat, sich fortzuentwickeln, ohne durch diese Fortentwicklung seine Identität aufgeben zu müssen.

Hier die Argumentationskette, die mich zu dieser Meinung führt:

Zunächst muss festgestellt werden, dass ich von Objekten spreche, die Ansammlung nur endlich vieler Elementarteilchen sind, und dass ich zunächst davon ausgehe, dass deren Anzahl über die gesamte Lebensdauer des Objekts hinweg begrenzt ist. Dies anzunehmen ist keine Einschränkung, solange wir Objekte bestimmten Typs vorliegen haben (Atome, Moleküle, oder Objekte aus unserem täglichen Leben).

Sei also N eine positive ganze Zahl und sei X irgendein Objekt, welches während seiner ganzen Existenz — genau genommen ist das die Existenz seiner Identität — aus maximal N Elementartteilchen zusammengesetzt ist. Da N eine endliche Zahl, ist, gibt es auch nur endlich viele Zustände Z, in denen sich diese Elementarteilchen zusammenfinden können: Konfigurationen also, die durch ein kompliziertes Kräftegleichgewicht gebildet und zusammengehalten werden (es ergibt sich als Folge aller vier Grundkräfte).

Diese endlich große Zahl möglicher Konfigurationen ist nichts anderes als eine obere Grenze für die Entropie, die das Objekt maximal zu unterstützen in der Lage ist (das Maximum an Information also, die dieses Objekt irgendwann in seinem Leben darstellen kann).

Wo Objekte nicht gerade aus nur einem einzigen Elementarteilchen bestehen, kann man sie gut vergleichen mit Baumkronen, die einem ständigen Luftzug ausgesetzt sind — und hin und wieder sogar einem richtigen Sturm:
  • Die Blätter entsprächen den Elementarteilchen,
  • die Äste und Zweige den sie in bestimmter Konfiguration zusammenhaltenden 4 Grundkräften,
  • und der Luftzug, Wind oder Sturm entspricht den Scharen durchs All streunender Schwärme noch in gar keine Struktur eingebundener Elementarteilchen (Neutronen, Photonen, u.a.). Die meisten von ihnen durchqueren das Objekt so als wäre es gar nicht da, einige wenige aber kollidieren mit dem Objekt zugeordneten Elemenarteilchen und führen so zu einem nächsten Objektzustand.

So wie jedes Gleichgewicht ist auch ein Kräftegleichgewicht selbstheilend in dem Sinne, dass Störungen, wenn sie denn hinreichend klein sind, sich selbst beheben.

Dies hat zur Folge, dass, wo das Objekt aus einem Zustand Z1 in einen nächsten Zustand Z2 gestoßen wird, jener dem Z1 recht ähnlich sein wird. Dennoch wird er (so sagt uns der zweite Hauptsatz der Thermodynamik) mit an Sicherheit grenzender Wahrscheinlichkeit von höhere Entropie haben.

In den extrem wenigen Fällen, wo das NICHT so ist, spricht man von einer Mutation des Objekts. Sie kann den Wert des Objektes vergrößern oder verkleinern (das Objekt also, wenn es denn ein Lebewesen ist, krank machen oder es — ein ganz klein wenig wenigstens — zu einem höher entwickelten Lebewesen machen. In Verbindung mit Darwins Selektionsprinzip ist so erklärt, warum der Zusammenstoß von Elementarteilchen zwar in der Regel, aber eben nicht immer, zu höherer Ordnung führt, also zu weniger Entropie.


Zusammenfassend lässt sich feststellen:

Was ich in Beitrag 1376-15 die Lebensbatterie eines Objektes X im Zustand Z nenne, ist nicht anderes als das Paar

B = ( e(X,Z), e(N) )


worin e(N) eine obere Grenze für im Objekt enthaltene Entropie bezeichnet und e(X,Z) die Entropie von X im Zustand Z ist.

Die Differenz   1 – e(X,Z)/e(N)   ist dann zu deuten als die in der Lebensbatterie noch vorhandene Restladung (so normiert, dass 1 der voll geladenen Batterie entspricht).

Dem 2. Hauptsatz der Thermodynamik entsprechend wird sie mit an Sicherheit grenzender Wahrscheinlichkeit bei jedem Zustandsübergang kleiner, so dass klar ist: X wird irgendwann sterben, d.h. wird irgendwann so entstellt sein, dass es seine Identität verliert — eben ganz so, wie auch ein Mensch sich mit zunehmendem Alter mehr und mehr verändert, schließlich in einen Sarg gelegt wird, und dort weiter zerfällt, so dass man irgendwann nicht mehr sagen kann, was man da noch vorfindet sei ein Objekt vom Typ Mensch.

Nachdem man die kleinstmögliche Version der Zahl N nicht kennen kann, wird man auch die Restladung der Lebensbatterie stets nur mit gewisser Unschärfe kennen.


Also Stueps, danke, denn ohne deine zwei Anregungen wäre ich auf diese Erklärung so schnell nicht gekommen.

Beste Grüße,
grtgrt

PS: Ich habe oben so getan, als würden Zustandsübergänge sich stets ergeben durch Zusammenstoß des Objekts mit streunenden Elementarteilchen. Das ist nicht ganz richtig, denn es gibt in jedem Quantensystem ja auch spontane Zustandsübergänge, sogar solche, von denen Chemiker durchaus wissen, mit welcher Wahrscheinlichkeit in etwa sie eintreten. Am Gesamtergebnis meiner Betrachtung ändert sich dadurch aber rein gar nichts.

 

  Beitrag 1894-1
RZQ — die Raumzeit der Quanten: die Zeit als gerichteter Graph

 
 

Wie die Allgemeine Relativitätstheorie uns lehrt, entwickeln physikalische Objekte — sobald sie sich relativ zueinander mit unterschiedlicher Beschleunigung bewegen — einen unterschiedlichen Zeitbegriff. Siehe hier Details dazu.

Daraus, so denke ich, ergibt sich zwingend, dass man die Zeit als einen in die 4-dimensionale Raumzeit der Allgemeinen Relativitätstheorie eingebetteten gerichteten Graphen aufzufassen hat, dessen Knoten Ereignisse im Sinne der Quantenmechanik sind (sprich: Punkte der Raumzeit, an denen Elementarteilchen entstehen, zerfallen, oder sich neu aufteilen in Folge einer Kollision).

Genauer beschrieben findet sich diese Idee auf folgenden Seiten (man sollte sie in eben dieser Reihenfolge lesen):

(1)   Vergangenheit und Zukunft genauer definiert

(2)   RZQ: Die Raumzeit der Quanten (Teil 1)
(3)   RZQ: Die Raumzeit der Quanten (Teil 2)

Ich würde mich freuen, wenn Physiker diese meine Ansicht kommentieren könnten (insbesondere dann, wenn sie glauben, Argumente dagegen zu haben).

 

  Beitrag 1894-9
RZQ — die Raumzeit der Quanten: Definition des Zeitgraphen

 
 
Um Okotombroks Wunsch gerecht zu werden, diese Diskussion möglichst ohne Links nach außen zu führen, gebe ich hier eine kurze Zusammenfassung dessen, was ich als die Struktur unseres Universums sehe (recht viel mehr steht auch nicht auf meiner Homepage):

 
Nach allem, was die Theoretische Physik heute weiß, könnte man unser Universum auffassen als ein Paar (B,E), für das gilt:
  • B ist eine Menge elementarer Energiequanten (Branen im Sinne der Stringtheorie) und
  • E ist eine Menge atomarer Ereignisse, deren jedes sich auffassen lässt als ein Paar X(V,Z), derart dass V und Z Teilmengen von B sind.

Jedes Ereignis X = X(V,Z) kann als Tor aufgefasst werden, welches aus seiner direkten Vergangenheit V = X.V in seine direkte Zukunft Z = X.Z führt.

Man beachte: X.V ist Input von Ereignis X, wohingegen X.Z seinen Output darstellt.

Da die Menge aller Paare ( v, z ) mit v und z aus E derart, dass der Durchschnitt von v.Z und z.V nicht leer ist, eine zweistellige Relation auf E ist, kann man ihre transitive Hülle bilden. Sie ist ebenfalls zweistellige Relation auf E. Man nennt sie die Zeit (und kann sie sehen als einen gerichteten Graphen, dessen Knoten Ereignisse und dessen Kanten Branen sind).


Die Zeit als Pfeil zu sehen wäre nur dann gerechtfertigt, wenn die sie darstellende transitive Relation eine lineare Ordnung (im Sinne der Mathematik) wäre. Sie scheint aber lediglich Ordnung zu sein.


Frage an alle Physiker unter Euch: Wie kann der so definierte gerichtete Graph — man sollte ihn die Struktur der Zeit nennen — mathematisch mit Einsteins Gleichungen der Allgemeinen Relativitätstheorie verschmolzen (oder zu ihnen in Bezug gesetzt) werden?

 

  Beitrag 1823-146
RZQ — die Raumzeit der Quanten: Zum Motor der kosmischen Uhr

 
 
Okotombrok aus 1823-7:
Bernhard Kletzenbauer aus 1823-6:
Keine Prozesse, keine Vergleichsmöglichkeiten = keine Zeit
So klar hab' ich das für mich noch nicht.
Wenn wir das Gedankenexperiment weiterdenken, und annehemen, es bewegt sich wieder was, heißt das dann, die Erstarrung hätte gar nicht stattgefunden?

Ja, Zeit ist Veränderung. ABER: Der Fall, dass es keine Prozesse gibt, existiert gar nicht. Genauer:

Einstein sagt:

Zitat:
Die Zeit ist, was man von der Uhr abliest.

Dazu kommt, was ich gelernt zu haben glaube (siehe Zur Struktur der Zeit):

Zitat von grtgrt:
Die einzige umfassende Uhr ist der Kosmos selbst.


Jeder Tick dieser Uhr ist ein atomares Ereignis, in dem eine kleine Portion schwingender Energie entsteht, vergeht, oder mit anderen solcher Portionen kollidiert (was zu einer Verschmelzung und Neuaufteilung dieser Portionen noch im Ereignis selbst führt).

Dass diese kosmische Uhr niemals stehen bleibt, ergibt sich aus dem, was Lisa Randall (auf Seite 238 und 225 ihres Buches Warped Passages ...) schreibt:

Zitat:
Quantum contributions to physical processes arise from virtual particles that interact with real particles.

Virtual particles, a consequence of quantum mechanics, are strange, ghostly twins of actual particles. They pop in and out of existence, lasting only the bares moment. They borrow energy from the vacuum — the state of the universe without any particles.

Da virtuelle Partikel spontan entstehen (ohne beobachtbaren Grund also), ist nicht damit zu rechnen, dass das jemals endet. Sie also sind der Motor, der die kosmische Uhr treibt und die Zeit vergehen lässt.

Mfg, grtgrt
 

 Beitrag 0-58
Zum Begriff physikalischer Theorien und Objekte

 
 

 
Zum Begriff physikalischer Objekte

 
 
Die Wurzeln der Disziplin Theoretische Physik reichen zurück bis in die Zeit der Vorsokratiker (etwa 500 v.Chr.). Spätestens seit Isaac Newton bedient sich die Theoretische Physik zum Aufschreiben ihrer Ergebnisse zwingend der Mathematik. Der mathematische Zweig der Differentialrechnung etwa wurde durch Newton (und parallel dazu durch Leibniz) zu eigens diesem Zweck erfunden und entwickelt.
 
Dennoch sehen gerade die Vertreter der Theoretischen Physik sich häufig — und weit mehr als andere Wissenschaftler — dem Vorwurf ausgesetzt, was sie da vorbrächten sei einfach nur eine mehr oder weniger haltlose Vermutung und allzu weit hergeholt. Dem ist entgegenzuhalten, dass solche Vermutungen vor allem dem Zweck dienen, keinen Lösungsansatz zu übersehen, nur weil er so gar nicht naheliegt oder so gar nicht dem entspricht, was unsere Erfahrung uns nahelegt. Dass die meisten ihrer Denkansätze die Natur missverstehen, ist den theoretischen Physikern bewusst. Jede Idee dennoch zu prüfen und nach Argumenten zu suchen, sie zu widerlegen, ist dennoch ihre Pflicht, denn schon allzu oft hat ganz extrem Unwahrscheinliches sich später doch als wahr erwiesen.
 
 
Heute also gilt: Theoretische Physik ist kreative Anwendung hoch komplizierter mathematischer Konstrukte mit dem Hintergedanken, dass die Mathematik uns hilft, die Welt in der wir leben, zu entdecken, zu verstehen und ihrem Verhalten uns gegenüber vorhersagbar zu machen.
 
Aus mathematischer Sicht ist jedes physikalische Objekt ein
 
 
Gegenstand ( Eigenschaften, Theorie )

 
wobei
  • sich der Gegenstand — als gedankliches Bild einer Kategorie physikalischer Objekte gleicher Art — über die genannte Menge von Eigenschaften definiert,
     
  • jede dieser Eigenschaften durch eine Zahl quantifizierbar ist
     
  • und die so erhaltenen Zahlen (Eigenschaftswerte) der genannten Theorie gehorchen.
     
  • Die Theorie wiederum ist nichts anderes als ein Gleichungssystem, welches beschreibt, in welcher Abhängigkeit mögliche Werte der einzelnen Eigenschaften zueinander stehen: Jede Lösung des Gleichungssystems beschreibt — als Menge von Eigenschaftswerten — genau eine Objektinstanz.

 
Streng genommen also ist jede Lösung der Theorie — jedes konkrete physikalische Objekt —
 
eine Abbildung der Menge aller Eigenschaften in die Menge aller reellen Zahlen.

 
 
Da nun aber Objekte im Sinne der Physik so gut wie immer zeitabhängig betrachtet werden — und sich ihre Eigenschaftswerte mit der Zeit ändern — werden Physiker sehr oft erst dann von einer Lösung der Theorie sprechen, wenn sie eine ganze Schar mathematischer Lösungen meinen, deren Scharparameter der Zeitwert ist.
 
Noch allgemeiner: Wo z.B. die Stringtheoretiker behaupten, ihre Theorie hätte so etwa 10500 Lösungen (deren jede ein konkrete Instanz des Gegenstandes "Raumzeit mit allen ber die gesamte Zeit hinweg darin existierenden Dingen" beschreibt), ist damit eine Schar von Lösungen gemeint, die mindesten zwei Scharparameter hat: die Zeit einerseits und die sog. Kopplungskonstante andererseits.
 
 
Wer sich nun aber vor Augen führt, dass jedes Gleichungssystem mit mehr als zwei Unbekannten — nennen wir sie Z und K in Anlehnung ans eben gegebene Beispiel — sehr viele Lösungen haben kann, die aber alle in Z und K denselben Wert haben, so wird klar, dass jede Schar von Lösungen der Theorie selbst wieder Vereinigung kleinerer, aber immer noch beliebig großer Scharen sein kann.
 
    Diese Tatsache übringens führt nicht selten zur Entdeckung neuer, bis dahin völlig unbekannter Phänomene. Beispiel für ein auf diesem Wege entdecktes Phönomen ist Antimaterie. Paul Dirac konnte 1928 die Existenz eines » positiv geladenen Elektrons « (man nennt es heute das Proton) vorhersagen, da sie ihm auffiel, dass eine von ihm betrachtet quadratische Gleichung statt nur einer Lösung — die die Ladung des Elektrons beschrieb — gleich deren zwei hatte, die sich nur im Vorzeichen unterschieden.

 
Wir sehen: Die Aussage der Stringtheoretiker, ihre Theorie hätte wohl so etwa 10500 Lösungen, sagt rein gar nichts. Das einzige, was wir mit Sicherheit wissen, ist, dass die Stringtheorie — sollte ihr Gleichungssystem denn jemals endgültig formuliert sein — sicher noch weit mehr Lösungen haben wird als Einstein Feldgleichungen (d.h. die Allgemeine Relativitätstheorie). Schon die aber hat unendlich viele  g e s c h l o s s e n e  Lösungen — z.B. die, welche man das Gödel-Universum nennt.
 
Hinzu kommt: Die meisten Lösungen der Allgemeinen Relativitätstheorie sind überhaupt nicht in geschlossener Form angebbar (als Formel also), sondern nur mit Hilfe komplizierter Rechnung als Menge der sie darstellenden Eigenschaftswerte schrittweise (und stets nur näherungsweise) konstruierbar.
 
 
Nun ist allerdings zu beachten, dass Lösungen, die der Physiker sucht, fast immer Lösungen eines bestimmten Anfangswertproblems sein sollen.
 
Was das bedeutet, beschreibt Martin Bojowald wie folgt (unter der Voraussetzung, dass der betrachtete Gegenstand ein Modell unseres Universums sei):
 


Bojowald ( S. 309-311 seines Buches Zurück vor den Urknall ):
 

In der Kosmologie spielt die Eindeutigkeit der Lösung einer Theorie der Raumzeit eine wichtige Rolle — denn schließlich können wir ja nur ein einziges Universum beobachten: da, in dem wir leben. Leider erhält man Eindeutigkeit nie umsonst, sondern stets nur über zusätzliche (die Theorie einschränkenden) Annahmen, die mehr oder weniger natürlich erscheinen mögen. Sie treten in zwei prinzipiell unterschiedlichen Rollen auf:
  • als Annahmen, die zum Aufstellen einer Theorie nötig sind,
  • aber auch als Annahmen zur Auswahl von Lösungen der dann schließlich gegebenen Theorie.

Zu Annahmen der zweiten Art kommt man über eine standardisierte Operation, die eng mit der mathematischen Art der Gleichungen zusammenhängt, die in in physikalischen Gesetzen Verwendung finden: Es sind dies (meist partielle) Differential-, hin und wieder aber auch Differenzengleichungen, die zu bestimmen haben, wie sich eine Größe ändert, wenn man sich in Raum oder Zeit — oder im Sinne eines anderen abstrakten Parameters — bewegt.
 
Um die Lösung eindeutig zu machen, reicht es nun aber nicht, lediglich zu wissen, wie sich sich ändert, wenn man sich bewegt: Man benötigt zusätzlich einen Standpunkt, von dem aus solche Änderungen ausgehen. Er kann sein
  • eine Anfangsbedingung (d.h. man legt fest, welchen Wert die untersuchte Größe zu dem  Z e i t p u n k t  hat, von dem man ausgeht)
  • oder eine Randbedingung (was bedeutet, dass man festlegt, von welchem Wert der Größe man am Rande des jeweils untersuchten  R a u m g e b i e t e s  ausgeht)
Anfangs- und Randbedingung können als das theoretische Äquivalent der Entscheidung eines Experimentators zu Aufbau und Durchführung seines Experiments angesehen werden. Die Theorie selbst aber soll, mindestens näherungsweise, dem Verhalten der Natur entsprechen — so wie uns auferlegte Naturgesetze es erwarten lassen.
 
Ein Experiment ist dann immer eine spezielle Situation in der Natur, die durch den Experimentaufbau (z.B. ein Pendel) und die gewählte Ausgangsposition (z.B. die Position, von der aus man das Pendel frei schwingen lässt) spezifiziert ist.
 
Also: Die Theorie wird durch die Wahl eines bestimmten natürlichen Phänomens fixiert, aber erst seine spezifische Realisierung liefert die Randbedingungen, von denen ausgehend man unter Berücksichtigung seiner allgemeinen Möglichkeiten eine Lösung sucht.
 


Festzuhalten bleibt:
 
 
Wer von der Lösung einer Theorie spricht, meint damit i.d.R.
 
eine Lösung, die gewisse Randbedingungen respektiert,
 
die aber keineswegs aus der Theorie selbst kommen.
 
 
Lösungen solcher Art beschreiben das betrachtete physikalische Objekt dann meist auch nur lokal.

 
 
 
Schönes Beispiel hierfür ist die Schwarzschild-Metrik (als die Lösung von Einsteins Feldgleichungen, die 1916 als erste gefunden wurde): Karl Schwarzschild betrachtete den Außen­raum einer kugelsymmetrischen Massenverteilung. Hier verschwindet der Energie-Impuls-Tensor und die Metrik hängt nur von einer radialen Koordinate ab. Die resultierende Raum-Zeit-Struktur nennt man heute ein » Schwarzes Loch «. Sie ist durch einen sog. Ereignishorizont gekennzeichnet, der die zentrale Singularität abschirmt.
 
Noch weit exotischer ist eine von Kurt Gödel gefundene Schar von Lösungen: der sog. » Gödelsche Kosmos «. Er lässt geschlossene Zeitlinien zu und wurde von Einstein als eher nicht die wirkliche Welt beschreibend eingestuft (genauer müsste man eigentlich sagen "keine wirklich existierende  R e g i o n  unserer Welt").


 

  Beitrag 2049-10
Korrekte Einordnung der Viele-Welten-Theorie von Hugh Everett III

 
 
Henry aus 2049-8:
 
Die Viele-Welten-Theorie und Stringtheorie beschreiben beide in Bezugnahme auf die Quantenmechanik die Möglichkeit weiterer Universen, wobei die Viele-Welten-Theorie explizit genau deshalb entwickelt wurde, die Stringtheorie aber ganz sicher nicht.


Henry,
das ist so nicht richtig, denn:

Everetts Viele-Welten-Theorie ist nur eine gedankliche Krücke, ein Bild, dessen Qualität in etwa der Qualität der Bilder "Schrödingers Katze" und "Überlagerungszustand" entspricht.

Auch deine Aussage "Die Entwicklung der Viele-Welten-Theorie beruht auf der Annahme, es gäbe VERGORGENE VARIABLEN" halte ich für absolut falsch. Woher nimmst Du diese Vermutung?


Gruß, grtgrt
 

  Beitrag 2049-52
Everett zur Natur physikalischer Modelle

 
 
Grtgrt in 2049-10:
 
Everetts Viele-Welten-Theorie ist nur eine gedankliche Krücke, ein Bild, dessen Qualität in etwa der Qualität der Bilder "Schrödingers Katze" und "Überlagerungszustand" entspricht.


Everett selbst erklärt die Natur physikalischer Modelle wie folgt:

Zitat von Hugh Everett III, 1956:
 
Wenn wir einmal eingesehen haben, dass jede physikalische Theorie im wesentlichen nur  M o d e l l  für die Welt der Erfahrung ist, müssen wir alle Hoffnung aufgeben, so etwas wie die "richtige" Theorie finden zu können, denn:

Nichts hindert eine Reihe sehr unterschiedlicher Modelle daran, mit der Erfahrung übereinzustimmen (sie können also alle "richtig" sein),
und weil uns niemals die Gesamtheit aller Erfahrungen zugänglich ist, gibt es keine Möglichkeit, die vollständige Richtigkeit eines Modells zu bestätigen.
 

 

  Beitrag 2049-54
-

 
 
Hans-m in 2049-53:
Zitat:
Nichts hindert eine Reihe sehr unterschiedlicher Modelle daran, mit der Erfahrung übereinzustimmen (sie können also alle "richtig" sein),
und weil uns niemals die Gesamtheit aller Erfahrungen zugänglich ist, gibt es keine Möglichkeit, die vollständige Richtigkeit eines Modells zu bestätigen.
 
Auch wenn wir niemals ALLES wissen, sollen wir deshalb den (Bruch-)Teil, den wir wissen, einfach ignorieren?

Auch auf der Suche nach Wissen werden wir immer an eine Grenze stossen, aber alles, was für uns vor der Grenze liegt, dass können wir getrost nutzen.


Ja, Hans-m,

das ist völlig richtig, und auch Everett sah das so, denn er plädiert sogar ganz explizit für den Mut zur Lücke:

Zitat von Everett:
 
Wir glauben nicht, dass es der Hauptzweck der theoretischen Physik ist, "sichere" Theorien aufzustellen, die in der Anwendbarkeit ihrer Konzepte einen hohen Preis erfordern ... vielmehr soll sie nützliche Modelle aufstellen, die eine Zeit lang ihren Dienst tun, um dann, wenn sie verbraucht sind, [ durch bessere ersetzt zu werden.
 

Quelle: DeWitt und Graham (Hrsg): The Many Worlds Interpretation of Quantum Mechanics, 1973, p. 111
zitiert durch Peter Byrne: Viele Welten — Hugh Everett III – ein Familiendrama zwischen Kaltem Krieg und Quantenphysik, S. 179


 

  Beitrag 2049-55
-

 
 
Zitat von Everett:
vielmehr soll sie nützliche Modelle aufstellen, die eine Zeit lang ihren Dienst tun, um dann, wenn sie verbraucht sind, [ durch bessere ersetzt zu werden. 

Das hört sich so an, wie das "Wissen" : Die Erde ist eine Scheibe, und der Mittelpunkt des Universums
Dieses Wissen tat auch lange seinen Dienst und wurde irgendwann durch neues Wissen ersetzt.

falsches Wissen ist in jedem Fall gefährlicher als fehlendes Wissen.

Als man entdeckte, dass die Welt keine Scheibe ist, musste das gesamte Weltbild neu überdacht werden.

Als man endeckte, dass das Atom nicht unteilbar ist, musste man nur das entsprechende Denkmodell erweitern.
Wenn man zunächst das Atom als unteilbar hielt, entdeckte man die Protonen, Elektronen, Neutronen, dann die Quarks und mittlerweile ist man auf der Stringebene angekommen.
Dieses neue Wissen brachte eine Erweiterung des Wissensstandes aber keinen Anlass das bis dahin bekannte über Bord zu werfen.
Wichtig ist, dass man jeden Wissensstand nicht als endgültig festhällt, sondern sich ständig Raum für Optionen offen lässt.
 

  Beitrag 2039-62
--

 
 
E... in 2039-5:
 
die Aussage "Es gibt keine Materie!" ist ohne jeden Sinn und Inhalt ...


Da steht E... nicht nur im Widerspruch zu Hans-Peter Dürr, sondern auch im Widerspruch zu Everett, denn der schrieb:

Zitat von Hugh Everett III:
 
Die Konstrukte der klassischen Physik sind genau so Fiktionen unseres eigenen Geistes wie die jeder anderen Theorie:
Wir haben zu ihnen lediglich sehr viel mehr Vertrauen.
 

Quelle: DeWitt und Graham (Hrsg): The Many Worlds Interpretation of Quantum Mechanics, 1973, p. 111
zitiert durch Peter Byrne: Viele Welten — Hugh Everett III – ein Familiendrama zwischen Kaltem Krieg und Quantenphysik, S. 181


 

  Beitrag 2039-66
Quantenfeldtheorie mdash; die verbesserte Version der Quantenmechanik

 
 

Von der Quantenmechanik zur Quanten-Feldtheorie


Siehe auch, was Joachim Schulz schreibt



Zitat von Hans-Peter Dürr, S. 22-64 in Dürr und Österreicher: "Wir erleben mehr als wir begreifen" (2001):
 
In der alten Quantenmechanik spricht man nur von Dualität: Jedem Teilchen "entspricht" eine Welle. Ich habe also die Dualität Welle oder Teilchen.

Die neue Quantentheorie, die Quantenfeldtheorie, die sich auf eine unendliche Vielzahl von Teilchen bezieht, führt zu einer noch weiter gehenden Auflösung:

Das Teilchenbild geht immer mehr verloren. Was bleibt, ist nur noch Form.


... Quantenfeldtheorien sind viel allgemeiner, liegen begrifflich tiefer. Der Feldbegriff überlebt, der Teilchenbegriff löst sich ganz auf.

... Sich das vorzustellen ist schwierig, weil man hier den Begriff der Gestalt erweitern muss. Gestalt ist ja etwas, das wir uns normalerweise nur als Materieanordnung im Raum vorstellen. Im Raum. Aber in welchem Raum? ... verglichen mit den Räumen, wie sie z.B. in der Beschreibung der Quantentheorie vorkommen, ist der 3-dimensionale Raum nur ein ganz spezieller Raum. Wellen schwingen sozusagen noch in anderen Räumen, nur diese anderen Raumdimensionen nehmen wir nicht als vierte oder fünfte Raumdimension wahr, sondern wir sagen dann: Aha, das ist also ein Elektron oder ein Proton und so weiter. Das aber ist jeweils nur eine Verwirklichung in einer anderen Raumdimension.
 


Man wird, was Dürr hier sagt, vielleicht dann verstehen, wenn man sich vor Augen führt, dass ja auch in der Mathematik die Dimensionen eines Raumes zu gleicher Zeit unabhängig von einander nutzbare  F r e i h e i t s g r a d e  sind.

 

  Beitrag 1376-30
Zu Wesen und Aufgabe physikalischer Modelle

 
 
Henry aus 1376-22:
Zitat von Gebhard:
PS: Und noch was: Denkst du wirklich, meine Theorie sei auch nur annäherend so unwahrscheinlich wie — um nur EIN Beispiel zu nennen — die Viele-Welten-Theorie des Hugh Everett III

... Wir können uns aber besser über Theorien unterhalten, deren Bestätigung/Widerlegung im Bereich des Möglichen liegt, da ist mir deine lieber als irgendwelche Parallelwelten.

Hi Henry,

ich weiß nicht, ob dir bewusst ist, dass jede einzelne der vielen Welten des Hugh Everett nichts anderes ist als genau unser Universum zu einem Zeitpunkt, den genau ein Ereignis der Raumzeit als seine Gegenwart sieht. Sie existieren also wirklich (als je ein Schnitt durch die Raumzeit).


Man sollte zudem berücksichtigen, dass physikalische Modelle nicht den Anspruch erheben, die Struktur der Natur zu modellieren (das wäre — nach dem, was Niels Bohr uns sagt — ja sogar unmöglich). Sie sind einzig und allein dazu da, das Verhalten der Natur nachzubilden, sprich: Man verlangt lediglich, dass sie eine gedachte Maschinerie sind, deren Verhalten isomorph zum Verhalten der Natur ist.

Zitat von Niels Bohr:
Die Physik kann nicht ergründen, wie die Natur funktioniert.
Aufgabe der Physik ist lediglich, zu untersuchen, wie die Natur sich uns zeigt.


Beste Grüße,
grtgrt
 

  Beitrag 2085-264
-

 
 
H... in 2085-263:
 
Lass uns mal voraussetzen, dass der Mensch Wahrnehmung besitzt und damit widerspiegeln kann. Damit ist es einfach möglich, dieser Widerspiegelung den Namen MODELL zugeben und dieses Modell ins Verhältnis zur Wahrnehmung zu setzen. Das ist eigentlich alles, was passiert.

Nun zu streiten, ob dies REAL ist (oder surreal oder blau oder salzig...), kann man machen, bringt aber nicht wirklich viel. Denn für den Menschen besteht die einzige Interaktion in Wahrnehmung und Widerspiegelung.


Das scheint mir schön und recht treffend ausgedrückt ...

 

  Beitrag 1376-62
Beispiel 1

 
 
Stueps aus 1376-61:
Man könnte weiter fragen, ob es noch das selbe Photon ist, das z.B. ein Elektron anregt, und danach wieder emittiert wird?
Wenn man erste Frage mit ja beantworten kann, tendiere ich zu der Auffassung, dass auch in einem Medium für das Licht keine Zeit vergeht.

Aber es könnten sich aus dieser Sicht vielleicht WIdersprüche entwickeln: Falls es immer das selbe Photon ist, welches durch ein Medium marschiert, und dabei absorbiert und emittiert wird, wie "merkt" es etwas von diesen Wechselwirkungen?

Hi Stueps,

das Schöne am Modellieren ist, dass man da gewisse Freiheiten hat. Die ergeben sich daraus, dass ja durchaus mehrere Modelle geben kann, die sämtlich in dem Sinne gültig sind, dass jedes dieser Modelle ein Verhalten aufweist, welches analog dem an der Natur beobachteten ist.

In diesem Fall aber sollte man dann am besten mit dem einfachsten dieser Modelle arbeiten.

Im konkreten Fall ist das einfachste Modell das, in dem man annimmt, dass ein Photon durch Wechselwirkung mit einem anderen Elementarteilchen sein Leben beendet.


Grtgrt aus 1376-30:
 
Man sollte berücksichtigen, dass physikalische Modelle nicht den Anspruch erheben, die Struktur der Natur zu modellieren (das wäre — nach dem, was Niels Bohr uns sagt — ja sogar unmöglich). Sie sind einzig und allein dazu da, das Verhalten der Natur nachzubilden, sprich: Man verlangt lediglich, dass sie eine gedachte Maschinerie sind, deren Verhalten isomorph zum Verhalten der Natur ist.

Zitat von Niels Bohr:
Die Physik kann nicht ergründen, wie die Natur funktioniert.
Aufgabe der Physik ist lediglich, zu untersuchen, wie die Natur sich uns zeigt.

Beste Grüße,
grtgrt
 

  Beitrag 1376-63
Beispiel 2

 
 
Henry aus 1376-50:
 
Es gibt keine wie auch immer geartete "Lebensbatterie", ...

Hi Henry,

ich behaupte ja gar nicht, dass die Natur eine Lebensbatterie kennt, sie ist lediglich der Teil meines Modells der Natur, der diesem Modell die Fähigkeit verleiht, für jedes betrachtete materielle physikalische Objekt dessen Rest-Lebenserwartung zutreffend abzuschätzen:

Bitte beachte:

Grtgrt aus 1376-30:
 
Man sollte berücksichtigen, dass physikalische Modelle nicht den Anspruch erheben, die Struktur der Natur zu modellieren (das wäre — nach dem, was Niels Bohr uns sagt — ja sogar unmöglich). Sie sind einzig und allein dazu da, das Verhalten der Natur nachzubilden, sprich: Man verlangt lediglich, dass sie eine gedachte Maschinerie sind, deren Verhalten isomorph zum Verhalten der Natur ist.

Zitat von Niels Bohr:
Die Physik kann nicht ergründen, wie die Natur funktioniert.
Aufgabe der Physik ist lediglich, zu untersuchen, wie die Natur sich uns zeigt.

Beste Grüße,
grtgrt
 

  Beitrag 1910-1
Beispiel: Hugh Everett's Viele-Welten-Theorie

 
 
Hallo Henry,

du hast mir ja recht deutlich gesagt, dass es sich deiner Meinung nach nicht lohnt, über Hugh Everett’s Viele-Welten-Theorie weiter nachzudenken. Nun, es juckt mich jetzt doch, dir zu widersprechen, denn mein Standpunkt ist:


Eine physikalische Theorie sollte man erst dann vergessen, wenn sie widerlegt ist.


Für Everetts Theorie gibt es bisher auch nicht den Ansatz einer Widerlegung — und das, obgleich sich John Wheeler (sein Doktorvater) recht bemüht hat, einen solchen zu finden.

Niels Bohr — scheint so einen Ansatz auch nicht gefunden zu haben, denn er hat sich über Everetts Arbeit (und darüber, dass Wheeler sie ihm hat durchgehen lassen) mächtig geärgert, selbst nie ein Gegenargument vorgebracht, aber doch seine Schüler ermuntert, solche Gegenargumente zu suchen.

Interessant auch: Feynman hat Everetts Theorie ebenfalls als Unsinn abgetan, wohl auch gar nicht weiter darüber nachgedacht &mash, und das obgleich er selbst eine ebenso utopisch wirkende Theorie in die Welt gesetzt hat: die Theorie der vielen Wege, die Quanten – wie er denkt – gleichzeitig nehmen, wenn sie sich von A nach B bewegen.

Lustig ist ferner, dass gerade er seinen Studenten sagte:

Wenn ein Physiker dir erzählt, etwas sei unmöglich, so glaube ihm nicht.
Wenn er dir aber sagt, etwas könne VIELLEICHT möglich sein, spricht viel dafür anzunehmen, dass es eben DOCH möglich ist.


Dass er dieser Meinung war, wird so wirklich spannend, wenn man weiß, dass Peter Byrne (ein Wissenschaftsjournalist, der den erst kürzlich aufgefundenen Nachlass von Everett durchforstete) in 2008 schreibt:

"Before I started looking into Everett’s theory, I would have thought it was crazy.
Now I wouldn't be surprised if it's true."


Byrnes lesenswerte Würdigung von Everetts Arbeit zeigt uns übrigens auch, dass Everett nicht einfach eine Idee vom Himmel fallen ließ, sondern dass er sie — mit Mitteln der Mathematik und Informationstheorie — hergeleitet hat aus der Annahme, dass die Schrödinger-Gleichung ohne Einschränkung für jedes Quantensystem Gültigkeit habe. Da die Wellengleichung des gesamten Universums nicht wirklich hinschreibbar und daher auch nicht direkt untersuchbar ist, hat er versucht, ihr mit Mitteln der damals eben erst durch Shannon und Wiener entwickelten Informationstheorie beizukommen.

Zitat:
Charles W. Misner, der zur selben Zeit wie Everett bei John Wheeler promovierte, schreibt:
  • Hugh Everett proposed that we not search for remedies of the implausible «collapse of the wave function» by changing the mathematics of the Schrödinger equation (or its relativistic field theory upgrades), but just look hard what would be predicted if we let the equations show us how they think nature behaves.
  • Now, over 50 years later [2010 , there is a strong effort to do just that, but the broad picture is not yet clear. Thus my guess for the outcome ... is that some different «big picture» will arise, which is not «many worlds» but will still uphold Hugh Everett’s conviction that paranormal influences do not overrule the Schrödinger equation.


Leider steht Everetts Dissertation nur in der stark verstümmelten Fassung im Netz, in der John Wheeler ihn zwang, sie vorzulegen (Wheeler fürchtete, seines Schülers ur­sprüngliche Version könne Bohr zu sehr erzürnen und könne, da Everett sich einer sehr bildhaften Sprache bediente, von Bohr als zu wenig wissenschaftlich gebrandmarkt werden).


Siehst du Henry: Dem armen Everett ging es offensichtlich nicht anders als mir, der ich jetzt ja auch vor der Situation stehe, dass z.B. du das physikalische Modell, das ich in Beitrag 1376-8, 1376-15 und fortgeführt in 1376-28 präsentiere, als zu wenig wissenschaftlich empfindest – und das schon von meiner Begriffsbildung her (die ich ja ganz bewusst so wähle, um den Leser aufzurütteln, ihm meinen Denkansatz klar zu machen und ihm zu helfen, sich aus allzu eingefahrenen Denkwegen zu befreien):

Henry aus 1376-16:
Gebhard,
ich denke, du machst es dir zu einfach, Dinge einfach mit neuen Begriffen zu bedenken und dann (wie in deinem folgendem Beitrag) drauf los zu fabulieren!

Beste Grüße,
grtgrt
 

  Beitrag 2011-11
Exakte Modelle können keine (ganz) genauen sein

 
 
Thomas der Große aus 2011-10:
 
Die spezielle Relativitätstheorie beschreibt starre Objekte, das sind genau die, für die eine Länge bezüglich eines Bezugssystems scharf ist
und dort gibt es eine 0-Geschwindigkeit.

Für mikroskopische Objekte der Quantenmechanik gilt das, was C... aus der Unschärfe-Relation abgeleitet hat
und damit keine Nullgeschwindigkeit.

Wenn es keine 0-Geschwindigkeit gibt, dann ist insbesondere die spezielle Relativitätstherorie falsch.


Mir scheint, man muss es so sehen:

Die Relativitätstheorien (SRT und ART) sind klassische Theorien in dem Sinne, dass sie  e x a k t e s  Modell sein wollen, ein Modell also, das glatte Formeln anstrebt und deswegen Ungenauigkeiten, die hinreichend klein sind, einfach in Kauf nimmt.

Die Quantentheorie aber ist eine  g e n a u e  Theorie in dem Sinne, dass sie keinerlei Ungenauigkeit in Kauf zu nehmen bereit ist. Wo nichts Genaues ausgesagt werden kann, werden obere Grenzen für die unvermeidliche Ungenauigkeit mit zu einem Teil der Theorie.


Den Unterschied zwischen exakt und genau macht man sich am besten klar, wenn man bedenkt, dass
  • ein Kreis exakt ist, wenn gegeben im Sinne der Mathematik (als Paar, welches Mittelpunkt und Radius nennt),
  • ein punktweise beschriebener Kreis aber das darstellt, was man zeichnen kann: Eine Linie, die — wenn man ganz genau hinsieht — nur mit gewisser Unschärfe definiert ist.

Man erkennt daraus:


Eine exakte Theorie kann sehr gute Näherung einer genauen Theorie sein.

Wirklich ganz genau aber kann sie — allein schon der Unschärfe-Relation wegen — NIEMALS sein.



Tieferer Hintergrund der Unschärfe-Relation ist natürlich Plancks Wirkungsquantum: die Tatsache also, dass die Natur kein stetiges Verhalten (im Sinne der Mathematik) aufweist.

Falsch muss eine exakte Theorie deswegen aber keineswegs sein. Sie ist höchstens ungenau (so wie jede den Bruch 1/3 darstellende Dezimalzahl nur endlich vieler Stellen nicht falsch, sondern nur ungenau ist).

 

  Beitrag 949-49
Kann uns ein physikalisches Objekt auf den Kopf fallen?

 
 
E... aus 949-48:
... denn physikalische Objekte sind alles andere als nur Konzepte.

Hi E...,

du scheinst nicht verstanden zu haben, dass
  • Physik und Natur keinswegs dasselbe sind
  • und dass mir zwar ein physisches Objekt auf den Kopf fallen kann (der Hammer etwa), aber eben nicht ein physikalisches.

Beste Grüße,
grtgrt
 

  Beitrag 949-58
Physikalisches Objekt = eine Sicht der Physiker auf die Natur

 
 
Henry aus 949-57:
es reicht, darauf hinzuweisen, dass sich "physisch" auf den (materiellen) Körper bezieht und physikalisch auf die Wissenschaft, die sich mit Gesetzen und Zusammenhängen befasst, die wir durch Messung über die Welt erhalten können - die Physik.

Siehst du, Henry,

nach eben dieser deiner Definition ist ein physikalisches Objekt eben doch nur eine durch die Physik (als Wissenschaft) konstruierte  S i c h t  auf das Objekt.

In der Informatik nennt man so was einen "View" (und der ist i.A. weit weniger als das gesamte Objekt).

Beste Grüße,
grtgrt
 

  Beitrag 1376-21
Identität eines physikalischen Objekts

 
 
Hi Stueps,

du hast natürlich völlig recht, wenn du sagst, ein Objekt trage keine Energie sondern sei Energie.

Andererseits aber lässt sich ja wohl nicht leugnen, dass ich von einem Objekt erst dann sprechen kann, wenn die Energieportion, aus der es besteht, in einer Form vorliegt, die diesem Objekt einen Typ gibt (und die es irgendwie unterscheidbar macht im Meer aller Energie unseres Universums).

Wir kommen hier wieder auf die schon einmal diskutierte Frage, was genau (bzw. wie konkret genau) ein Objekt denn eigentlich sein muss, um als physikalisches Objekt bezeichnet werden zu können. Reicht eine konzeptuelle Abgrenzung, oder muss das Objekt sich in wirklich beobachtbarer Weise von seiner Umgebung abgrenzen? Wenn jene Abgrenzung aber wahrnehmbar ist, könnte man es dann nicht wirklich auch als Energieträger sehen? In dem Sinne also war meine sprachliche Wendung des "Tragens" von Energie gemeint.

Wie du ferner schon sagst: Energie altert nicht. Und wie der Energieerhaltungssatz uns lehrt, verbraucht sie sich auch nicht — sie kann sich höchstens umverteilen oder sich in unterschiedlicher Form zeigen.

Damit ist klar: Als Energie betrachtet, kann ein physikalisches Objekt (z.B. ein Mensch) auf keinen Fall altern. Altern — und sich mehr oder weniger plötzlich verändern — kann nur die Form, in der sich das Objekt beobachtbar macht, und die dazu führt, dass es sich von anderen physikalischen Objekten abgrenzt und so für uns überhaupt erst zu einem wohldefinierten Objekt wird.

Klar ist auch: Solche Alterung kann nur dann in vielen kleinen Schritten erfolgen, wenn das Objekt kein atomares ist (also kein Elementarteilchen).

Würdest du mir da zustimmen?

Beste Grüße,
grtgrt
 

  Beitrag 1701-16
Physikalische Objekte: Wie Okotombrok sie versteht.

 
 
Hallo Grtgrt,

Grtgrt aus 1701-15:
 
Licht ist ein physikalisches Objekt (und als solches unser Modell für einen Teil der Natur).

Mehr dazu auf Seite Zum Wesen physikalischer Aussagen.
 

auf deiner oben genannten Homepage schreibst du:
Zitat:
Physikalische Objekte sind nichts anderes als gedankliche Modelle, . . .
So weit kann ich nicht mitgehen. Mit physikalischen Objekten meint man direkt beobachtbare Dinge.
Tauscht man aber den Begriff "physikalisches Objekt" gegen den Begriff "physikalische Größe" aus, so kann ich nur zustimmen.

Weiter:
Zitat:
. . . die der Mensch sich macht, aus dem Wunsch heraus,
das Verhalten der Natur verstehbar und vorhersagbar zu machen.
Auf physikalische Größen bezogen stimme ich dir vollkommen zu.

mfg okotombrok
 

  Beitrag 1701-18
Physikalische Größen sind: quantifizierbare Eigenschaften physikalischer Objekte

 
 
Grtgrt aus 1701-17:
Hi Okotombrok,

ist ein Atom oder ein Elektron in deinen Augen eine physikalisches Objekt oder nur eine physikalische Größe?
Und ein ganzes Universum: Sollte man es wirklich nur als Größe bezeichnen?

Ich würde unter einer Größe eher eine Zahl verstehen (etwas ohne Struktur).

Gruß, grtgrt

Gebhard,

verstehst du nicht, was Okotombrok bemerkt? Falls physikalische Objekte unsere gedanklichen Modelle SIND, gibt es keine beobachtbare Außenwelt. Und ob du nun unter einer Größe eine Zahl verstehst oder nicht - in der Physik ist darunter etwas anderes und genau definiertes gemeint (Größen sind messbare Eigenschaften an Objekten). Wenn wir schon diskutieren, wollen wir uns doch bitte an die Begriffe so halten, wie sie allgemein anerkannt sind.
 

  Beitrag 1894-5
Zu physikalischen Objekten

 
 
Hi Henry,

es scheint mir nicht richtig, nur das, was sich schon als Materie zeigt, als physikalisches Objekt elementarster Art zu bezeichnen. Unteilbare Bausteine des Kosmos sind ganz sicher schwingende Energie-Portionen (nur einige davon zeigen sich zeitweise im Zustand Materie).

Nun zu Ereignissen: Als Position eines Ereignisses kann man nur die Summe aller Punkte verstehen, an denen sich die am Ereignis beteiligten Quanten dann gerade befinden. Es ist aber keineswegs so, dass die Position eines Quantums nur ungenau beobachtbar wäre: Sie ist wirklich auch ungenau definiert.

Warum das so ist, wird sehr schön erklärt auf Seite Teilchen sind Wellenpakete.

Damit ist klar: Unter der Position eines Ereignisses muss man sich (aus diesen beiden Gründen) eine kleine Region der Raumzeit vorstellen (eine Punktemenge, die Vereinigung von N Punktemengen ist, wenn am Ereignis N Quanten beteiligt sind) — also nicht einen einzigen Punkt im Sinne der Mathematik, sprich: im Sinne der die Raumzeit modellierenden differenzierbaren Mannigfaltigkeit.

Nun zur Frage, ob Wahrscheinlichkeitswellen wirklich existieren: Da Interferenz am Doppelspalt zeigt, dass sie interferieren, müssen sie ja wohl existieren. Eine ganz andere Frage ist, was es denn bedeuten mag, zu existieren. Hierzu hat Niels Bohr mal was sehr Treffendes gesagt (siehe meine Seite Zum Wesen physikalischer Aussagen und dem, was man die Realität nennt).

Zu Stringtheorie (ST) und Schleifen-Quanten-Gravitation (SQG): Zwischen beiden besteht insofern ein ganz gewaltiger Unterschied, als die ST eine hintergrund-abhängige Theorie ist, wohingegen die SQG (ebenso wie die ART) ohne Bezug zu irgend einem Hintergrund formuliert ist. RZQ ist hintergrund-unabhängig, also viel näher an SQG als an ST.

Ich sehe RZQ als eine Art vergröberte SQG, wobei mir aber nicht klar ist, ob die gröbere oder die feinere Sicht die natürlichere ist.

Das liegt daran, dass ich die Spin-Networks nicht wirklich verstehe. Als gut verständlich (und "anschaulich") würde ich sie nicht einzustufen. Hast du den Artikel, auf den ich oben verweise, gelesen?

Gruß, ggreiter
 

  Beitrag 1894-62
Physikalische Objekte sind Modelle, das Verhalten der Natur zu erklären

 
 
Hi Gerhard,

Physikalische Objekte (die Zeit etwa) sind gedankliche Modelle,
die der Mensch sich macht, aus dem Wunsch heraus,
das Verhalten der Natur verstehbar und vorhersagbar zu machen.


Man darf solche Modelle aber auf keinen Fall mit der Natur selbst verwechseln (und so kann es für jeden Teil der Natur — die Zeit ist da keine Ausnahme — mehr oder weniger genaue Modelle geben: sie sind dann natürlich auch mehr oder weniger einfach).

Dass auch grobe Modelle — wie etwa das von Aristoteles — sehr weit tragen können, ist unbestritten. Was aber, wenn man über eine Grenze der Anwendbarkeit eines einfachen Modells hinausgehen möchte? Eben dann braucht man ein genaueres, weniger einfaches Modell.

Mfg, grtgrt
 

  Beitrag 1057-149
Hat auch die Zeit ein sie erzeugendes "Higgs"-Teilchen?

 
 
Hi Thanninger,
  • Wenn ich Higgs richtig verstehe, glaubt er, dass Elementarteilchen gewisser Art irgendwie dadurch gebremst werden, dass sie sich ständig durch ein Meer sog. Higgs-Teilchen bewegen (und das eine Art Reibung versursacht, wodurch dann ihre Geschwindigkeit reduziert wird und sie somit etwas bekommen, das wir als ihre Ruhemasse deuten).
    Was ich dabei nicht verstehe: Auch das Higgs-Teilchen selbst hat Masse — man könnte also auf die Idee kommen, dass die Teilchen sich gegenseitig Masse geben.
    Frage an die Physiker also: Worin genau unterscheidet sich das Higgs-Teilchen — seiner Qualität nach — von anderen Teilchen, die ähnlich große Ruhemasse haben?
  • Was auch immer Teilchen gewisser Art verlangsamt, wenn sie sich durch den 3-dimensionalen Raum bewegen (das Higgsfeld etwa?), könnte mit dafür verantwortlich sein, dass auch die Zeit einem Teilchen (oder Objekt) umso langsamer vergeht, je mehr es sich durch den 3-dimensionalen Raum bewegt. Auf jeden Fall wäre das konform zu meiner Theorie, die ja sagt, dass die Zeit genau dort voranschreitet, wo Teilchen zusammenstoßen (also aufgehalten werden).
  • Wenn dem so wäre, könnte man die Zeit, die einem Objekt vergeht, interpretieren als analog zur Abnutzung, die es erfährt, allein dadurch, dass es mit anderen interagiert. Und tatsächlich: Auch das Altern eines Lebewesens ist ja nichts anderes als ein Abnutzungsprozess, der etwas verbraucht, was nicht wieder-erlangbar ist: Zukunft des Lebewesens.
  • Natürlich ist das bisher nicht mehr als eine Frage — es würde mich aber interessieren, ob ihr schon mal jemand nachgegangen ist.

Gruß, grtgrt

PS: Wie schon Okotombrok würde auch ich es vorziehen, wenn wir Fragen zum Wesen der Zeit in einem eigenen Thread unterbringen würden. Warum eröffnest du nicht einfach ein Thema Fragen zum Wesen der Zeit ?
 

  Beitrag 1057-187
Strahlungsdruck

 
 
E... aus 1057-182:
Das Photonen keine Ruhemasse haben erlebst Du täglich. Deine Netzhaut verarbeitet die Informationen die das sichtbare Licht im Sinne des Wortes an sie heran trägt. Dabei genügt die Energie des ankommenden Lichts um die Fotorezeptoren (rot-, grün- und blauempfindlich) so zu reizen das sie die optischen Reize ans Gehirn weiterleiten .....
.
...Wenn Du schon mal ein KFZ gesehen hast mit einem kräftigen Hagelschaden, dann weist Du was Deiner Netzhaut blühen würde, wenn Licht eine Ruhemasse hätte, die mit knapp 300.000 Kilometern pro Sekunde permanent in Deiner Netzhaut einschlägt. Die Netzhaut wäre binnen kurzem unbrauchbar und das Auge würde erblinden.

Das Beispiel hinkt.

leg dich mal mit nacktem Bauch in den Regen. jeder Tropfen erzeugt einen Nervenreiz, wenn er Deine Haut "trifft".
Du wirst davon jedoch nicht durchlöchert, als würdest Du von einer Schrotladung getroffen, obwohl der Regen eine Ruhemasse besitzt. Die Haut ist so "gebaut" dass sie dem Aufschalg des Regens standhält. Auch die Netzhaut hat sich im Laufe der Evolution so entwickelt, dass sie dem "Photonenhagel" ein Leben lang standhält.
Zudem hat das Photon beim Auftreffen ja noch seine Masse. Es verliert seine Masse genau in dem Augenblick, sobald es seine Existenz aufgibt, und seinen Energiepuls auf die Netzhaut übertragen hat.

siehe auch Wikipedia : Strahlungsdruck

Okotombrok aus 1057-180:
Außerdem hat noch ie jemand beobachtet, dass sich Ruhemassen mit c ausbreiten.

Du musst das anders definieren:
Licht bewegt sich immer mit c, wenn wir es messen. Darum können wir auch nur eine Aussage treffen, bei Licht =c
anders haben wir es nie gesehen, denn wäre es langsamer, dann wäre es nicht existent. und was nicht existiert, darüber kann man keine Aussage machen.

Harti sagte dazu bereits:
Harti aus 1057-161:
Das Photon hat keine Ruhmasse, weil es nicht ruhen kann.

 

  Beitrag 949-64
Wichtige Details zur Kopenhagener Interpretation

 
 
E... aus 949-62:
Wie soll ein makroskopisches System mit Bestandteilen funktionieren die quantenphysikalische Eigenschaften haben, nicht stabil, nicht fassbar, nicht berechenbar und zeitlos sind? Antworte bitte nicht auf die Frage ...
 

An alle,

es mag ja sein, dass E... die Antwort ganz genau kennt, ich aber weiß nur folgendes:

Zitat von Lothar Schäfer, Hochschullehrer:
 
Die Kopenhagener Interpretation betrachtet den Kollaps der Wellenfunktion in einem Messakt NICHT als plötzliche Änderung des Zustandes des Universums, sondern lediglich als plötzliche Änderung unseres Wissens um irgendein Phänomen der physikalischen Wirklichkeit.

Henry Stapp hat in seinem Buch "Mind, Matter, and Quantum Mechanics" (1993) diese Ansicht so charakterisiert: In der Kopenhagener Interpretation ist das wahre Wesen von ψ nicht ein Problem der Ontologie, sondern eines der Epistomologie.
 

Auf die Frage, wer Stapp denn nun eigentlich ist, bekommt man zur Antwort: "Stapp is a leading quantum physicist who has given particularly careful thought to the implications of the theory that lies at the heart of modern physics. In this book, which contains several of his key papers as well as new material, he focuses on the problem of consciousness and explains how quantum mechanics allows causally effective conscious thought to be combined in a natural way with the physical brain made of neurons and atoms."

Gruß, grtgrt

PS: Was ich an Schäfer so mag ist, dass seine Sprache ebenso klar ist, wie seine Gedanken.
 

  Beitrag 1701-20
Zum Begriff physikalischer Objekte: seine wohl beste Definition

 
 
H... aus 1701-19:
phys. Objekt: ganz simple ein Sachverhalt, der einer physikalischen Betrachtung unterzogen werden kann. Der Sachverhalt kann wahrnehmbarer oder rein theoretischer Natur sein (z.B. war das Higgs Boson viele Jahre ein rein theor. phys. Objekt,
seit einigen Wochen besteht Hoffnung, dass nun das Standardmodell komplett ist).

phys. Größe: eine Eigenschaft des Sachverhalts (=phys. Objekt)

Gruß H... R.

An alle:

Dieser Versuch, den Begriff physikalisches Objekt zu definieren, scheint mir der bisher beste.

Danke, grtgrt


Zur Info: Ein Teil dieser Diskussion wurde an anderer Stelle geführt und ist dort beendet worden mit Beitrag 1896-19 und Beitrag 1896-20.
 

  Beitrag 1701-15
Beispiel eines physikalischen Objekts: Das Licht

 
 
Licht ist ein physikalisches Objekt (und als solches unser Modell für einen Teil der Natur).

Mehr dazu auf Seite Zum Wesen physikalischer Aussagen.
 

  Beitrag 1875-107
Stringtheorie: Ein Beispiel geschickter Kombination zweier Denkwege

 
 
Hi E...,

es freut mich, dass mir mal jemand sagt, wie mein Stil auf ihn wirkt. Danke also.

Damit Du mich besser verstehst, sei gesagt: Ich bin kein Physiker, die Art meiner Argumentation ist aber sicher dadurch geprägt, dass ich mal Mathematiker war (mit Schwerpunkt auf Algebra und Zahlentheorie), dann aber — seit etwa 1980 — stets als Informatiker gearbeitet habe (als IT-Berater mit Schwerpunkt auf besonders methodischem Software-Engineering).

Mein Interesse an theoretischer Physik wurde geweckt durch die Beobachtung, dass theoretische Physiker sich von Science Fiction Autoren des Typs Isaac Asimov eigentlich nur dadurch unterscheiden, dass sie versuchen, ihre visionären Gedanken über die Natur und die Struktur unserer Welt nicht nur aufzuschreiben, sondern auch zu beweisen oder zu widerlegen.

Bester Weg hierfür, da gebe ich Dir recht, wäre die Durchführung geeigneter Experimente. Dieser Weg aber stößt schnell an Grenzen (an prinzipielle, weit vorher aber noch an finanzielle). Wo sie erreicht sind, ist Informationsverarbeitung der einzige Weg, weiter zu kommen. Sie muss erfolgen in geschickter Kombination mathematischer und ingenieurmäßiger Denkwege.

Wenn man nicht zufällig ein Albert Einstein ist, kann man sich Erfolge dennoch nur erhoffen, wenn man so viel wie möglich auf den Gedanken anderer aufbaut. Wie jeder Promovierte weiß, ist eigene Leistung i.A. ja nur ein nächster kleiner Schritt hin zum Ziel. Man geht ihn in der Hoffnung, dass andere ihn aufgreifen und so ihrerseits in die Lage versetzt werden, die Entfernung zum Ziel weiter zu verkleinern.

Versetzen wir uns jetzt in die Rolle dieser anderen. Die Wahrscheinlichkeit, dass sie einen noch unvollständigen Weg aufgreifen, wird umso größer sein, je wertvoller er ihnen erscheint (sprich: aus je mehr Schritten er schon besteht). Wenn ich also einen (N+1)-sten Schritt in die Welt setze, tue ich gut daran, nicht zu verbergen, dass er nicht der erste Schritt des langen Weges hin zur Erkenntnis ist, sondern nur ein weiterer, vielleicht aber wirklich notwendiger.

Das also ist der Grund, warum ich nur ungern isolierte Ideen in die Welt setzte. Mir ist viel wohler, wenn ich die Ideen anderer extrapoliere. Nicht zuletzt gibt das mir und denen, die mir folgen, ja auch eine gewisse Sicherheit, nicht völlig schief zu liegen — solche Sicherheit zu suchen sehe ich nicht als Schwäche sondern als Teil jeder professionellen Vorgehensweise.


Nun zu den Strings: Sie modellieren ganz klar Elementarteilchen (virtuelle und auch stabile, wie etwa die Elektronen und Positronen). Und die existieren ja nun wirklich. Und dass Elementarteilchen in Schwingung befindliche Energiequanten sind, gilt ja nun auch schon längst als gesichertes Wissen.

Das Fragezeichen, das man neben die Stringtheorie zu setzen hat, steht also weniger am Begriff der Strings (oder Branen), sondern steht vor allem an den Vorhersagen, die die Stringtheorie in Hinsicht auf Sysmmetrie und Vollständigkeit des Standardmodells der Elementarteilchenphysik macht. Es steht insbesondere an allen Aussagen, die unserem Universum weitere Dimensionen zugestehen wollen oder gar noch bestimmte Krümmung jener.

Jene Fragezeichen sind den Stringtheoretikern selbst sehr wohl bewusst: Greene etwa schreibt (auf Seite 485 seines Buches The Fabric of the Cosmos): "More and more, ... clues point toward the conclusion that the form of spacetime is an adorning detail that varies from one formulation of a physical theory to the next, rather than being a fundamental element of reality. Much as the number of letters, syllables, and vowels in the word cat differ from those in gato, its Spanish translation, the form of spacetime — its shape, its size, and even the number of its dimensions — also changes in translation."

Mir ist sehr wohl klar, dass es durchaus Fachleute gibt, die der Stringtheorie sehr skeptisch gegenüber stehen:
  • Mir ist bekannt, dass Peter Woit (mir scheint, er wird wohl nur seines provokativen Buches wegen so oft genannt) so weit geht, zu sagen, dass Fördergelder, die an Stringtheoretiker gegeben werden, Geldverschwendung seien. Nachvollziehbare Argumente hat er in meinen Augen aber nicht, und seine Behauptung, die Stringtheorie sei bisher nicht in der Lage gewesen irgendwelche Vorhersagen zu machen, wirkt nicht so recht glaubhaft, wenn man weiß, dass er sich mehr mit Mathematik als mit Physik beschäftigt (also vielleicht nicht wirklich Fachmann ist) und dass viele Physiker ihm widersprechen: In WikiBooks findet sich eine Liste von 12 Vorhersagen der Stringtheorie, die man durchaus als schon verifiziert ansieht).
  • Andere, wie etwa Roger Penrose, sind skeptisch, versuchen aber dennoch — wie es sich für Wissenschaftler gehört — objektiv zu bleiben und machen sich wenigstens die Mühe, den Stringtheoretikern ernsthaft zuzuhören.
  • Einer von ihnen ist sicher auch Lee Smolin. Der aber sieht vor allem deswegen wenig Sinn darin, Stringtheorie zu betreiben, da sie sich in Bereiche vorwagt, in denen die Experimentalphysik wohl niemals wird Aussagen machen können (siehe dieses Interview mit ihm). So zu argumentieren ist nachvollziehbar, zeugt aber vielleicht doch eher von fehlendem Mut, über bestehende Grenzen hinauszudenken.

Ich sage: Solange die Stringtheorie keine Aussage macht, die im Widerspruch zu mindestens einem anerkannten Ergebnis der Experimentalphysik steht, kann auf keinen Fall behauptet werden, sie sei ein Irrweg.

Peter Woit scheint diese Logik zu ignorieren (was mir seine Meinung recht suspekt macht). Smolins Buch dagegen sollte man gelesen haben (aber: Wichtige Erfolge der Stringtheorie, über die Greene 2010 berichtet, scheint Smolin 2006 nicht zu kennen. Ob das Uneinigkeit in der Beurteilung bedeutet oder darauf zurückzuführen ist, dass jene Ergebnisse damals vielleicht noch gar nicht erarbeitet waren, weiß ich nicht).

Beste Grüße,
grtgrt = ggreiter = Gebhard Greiter

Mein Motto: Die beste Praxis ist eine gute Theorie (nach E... Denert, einer der beiden Gründer von sd&m)
 

  Beitrag 1878-7
Was es bedeutet, in die Zukunft zu reisen

 
 
 
Wie man (prinzipiell wenigstens) in beliebig ferne Zukunft reisen kann

Auf Seite Verschieden schnell durch die Zeit reisen wird anhand von drei Beispielen gezeigt, wie man in die Zukunft reisen kann.

Speziell das dritte Beispiel dort ist interessant.

Wer es nachgerechnet hat, wird sein Ergebnis vorliegen haben in der Form

          S = F( A, Z )

wo F eine Formel ist, A der Abstand vom Schwarzen Loch, in dem der Astronaut eine gewisse Zeit Z wartet, bis er umdreht und zur Erde zurückkehrt,
und S die Zahl der Jahre, die auf der Erde vergangen sein sollen, wenn der Astronaut zurückkehrt.

Natürlich werden die in der Formel auftretende Koeffezienten davon abhängig sein, welches Schwarze Loch der Astronaut zu besuchen gedenkt, und wie heftig er plant, auf seiner Reise zu beschleunigen oder abzubremsen.

Interessant aber ist: Da Vergrößern von A den Sprung S in die Zukunft verkleinert, und da umgekehrt Vergrößern der Parkdauer Z den Sprung S vergrößert, lässt sich praktisch jeder Wert von S erreichen.

Das einzige Problem des Verfahrens ist, dass der Astronaut sterben kann, noch bevor er wieder die Erde erreicht. Man sollte ihn also besser durch einen robusten Roboter ersetzen (oder Raumschiff und Astronaut durch einen Forschungssatelliten).


PS: Reisen in die Vergangenheit sind grundsätzlich NICHT möglich (da Zeit eine Richtung hat, die von Ursache zu Wirkung führt).

 

  Beitrag 1896-53
Das Credo von Lothar Schäfer (einem Physiker)

 
 

Das Credo von Lothar Schäfer — einem Physiker:

  • An der Wurzel der physikalischen Wirklichkeit erweist sich die Natur der materiellen Dinge als nicht-materiell.
  • Örtliche Ordnung wird durch nicht-lokale, unverzögert fernwirksame Phänome beeinflusst.
  • Die Wirklichkeit der Raumzeit hängt möglicherweise von Prozessen ab, die außerhalb der Raumzeit verankert sind.

Er sagt ferner:

Transzendente Wirklichkeit ist ihrem Wesen nach unbeobachtbar.

Die moderne Physik ist in zunehmendem Maße bereit, sie zur Erklärung der Wirklichkeit dennoch zuzulassen.

Die Botschaft moderner Physik ist, dass die Wirklichkeit an ihren Grenzen nicht im Nichts verklingt, sondern im Bereich des Metaphysischen.


 

  Beitrag 1904-1
Leben wir in einer just in time errechneten Wahrnehmungswelt?

 
 
Wie real ist, was wir als real erachten?

 
Dieses Thema dient der Diskussion von Fragen rund um Thomas Campells Theory of Everything (TOE).

Frage an alle Physiker unter euch:
    Gibt es noch andere Deutungen?
  • Wer hat sein Buch "My-Big-TOE-Complete-Trilogy" gelesen (und hat welche Meinung dazu)?

 

  Beitrag 1906-1
Gravitationskraft, versteckte Raumdimensionen, Locally Localized Gravitation

 
 
Liasa Randall betont gerne, dass sie sich mehr als Modellkonstrukteur sieht denn als Stringtheoretiker(in). Der Unterschied ist für sie:
  • Stringtheoretiker arbeiten top down, indem sie versuchen, in einer riesigen Menge ihnen vor die Füße gefallener Modelle — man spricht von etwa 10500 — solche zu finden, die keiner durch uns derzeit als zuverlässig anerkannten Beobachtung, die Astronomen oder Experimentalphysiker je gemacht haben, widersprechen.
  • Modellkonstrukteure aber gehen so vor, dass sie — ausgehend von eben jenen Beobachtungen — Modelle bottom up zu konstruieren versuchen, und das ohne jede Rücksicht darauf, ob sie nun zur Stringtheorie passen oder nicht.

Zwei Beobachtungen scheinen mir interessant:
  • Lisa Randalls Approach ist kreativer, objektiver und weniger naiv als der der Stringtheoretiker.
  • Mindestens eines ihrer Arbeitsergebnisse — ein Modell, das sie Locally Localized Gravitation nennt — bricht sogar mit der gängigen Vorstellung, dass im gesamten Kosmos ein und dasselbe Gravitationsgesetz gelte.

Genauer:

In Kooperation mit Andreas Karch betrachtete sie eine 5-dimensionale Raumzeit und darin eine 4-dimensionale Brane flacher Geometrie (sie könnte die Welt sein, in der wir leben). Randall schreibt:

"[In the theory we developed,] space looks 4-dimensional on or near the brane, but most of the space far from the brane appears higher-dimensional. ... We named our scenario locally localized gravity because localization produces a graviton that communicates 4-dimensional gravitational interactions only in a local region."

Nachdem die beiden die Brane etwas modifiziert hatten (so dass sie nun etwas negative Energie trug und daher nicht mehr flach, sondern leicht gekrümmt war), ergab sich etwas noch weit Interessanteres:

" ... we decided to study this model solely because of its fascinating implications for dimensionality: ... [Assuming a] second brane sufficiently far away, we found that there were two different gravitons localized near each of the two branes. Each of the graviton probability functions peaked near one of the two branes, and decreased exponentially quickly as you left it.

Neither of these gravitons was responsible for 4-dimensional gravity over the entire (5-dimensional) space ... The gravities experienced on the different branes were different. They could even have very different strength. Objects on one brane didn’t interact gravitationally with objects on the other. ...

The appearance of two different particles that both look like the 4-dimensional graviton was a big surprise to us. General physical principles were supposed to ensure that there is only a single theory of gravity. And indeed, there is a single 5-dimensional theory of gravity, but 5-dimensional spacetime turns out to contain two distinct particles that each communicate a gravitational force that acts as if it is 4-dimensional, each in a distinct region of 5-dimensional space ...

If this model is correct, we would have to live on the brane to experience 4-dimensional gravity ...

Of course, we do not yet know whether locally localized gravity applies in the real world."


Source: Chapter 23 of Lisa Randall’s Book "Warped Passages unraveling the Mysteries of the Universe’s hidden dimensions"(2005).
 

  Beitrag 1906-3
Beispiele flacher und nicht flacher Geometrie

 
 
Harti in 1906-2:
Hallo Grtgrt,

was genau muss ich mir unter einer "flachen Geometrie" vorstellen. Kann man den Begriff genauer definieren ?

Um konkret zu werden, kann ich einen Regentropfen mit Hilfe eines flachen Geometriemodells beschreiben ?

Hi Harti,

im 2-Dimensionalen wäre eine Fläche flacher Geometrie vergleichbar mit einem — aufgerollten oder flach daliegenden — Stück Papier.

Stringtheoretiker sprechen gerne von "aufgerollten Dimensionen". Lisa Randall sagt dazu:

Zitat von Lisa Randall:
The curled-up space is still mathematically flat ... because you can unroll the dimension to something you would recognize as flat; that is NOT true for a sphere, for example.

Die Oberfläche eines Regentropfens hat gekrümmte Geometrie (KEINE flache also).

Flache Geometrie ist euklische Geometrie.

Gruß, grtgrt
 

  Beitrag 1911-1
Absoluter Zufall ist unentscheidbar

 
 

Einstein war der letzte große Physiker, der dachte, die Natur würde voll deterministisch funktionieren (Gott würfelt nicht).

Niels Bohr hielt dagegen und war der Meinung: Gott würfelt doch, denn Quantensprünge sind absolut zufällig.

Ich sage: Wir können es nicht wissen, denn Bohrs Argumentation ist widersprüchlich. Der Beweis dafür kommt aus Bohrs eigenem Credo:


Zitat von Niels Bohr:
Die Physik kann nicht ergründen, wie die Natur funktioniert.

Sie kann nur untersuchen, wie die Natur sich uns zeigt.



Bohr hätte erkennen müssen, dass — wenn dem so ist — es DREI Arten von Zufall gibt (statt nur deren zwei):
  • scheinbaren Zufall,
  • physikalischen Zufall und
  • absoluten, als solchen dann aber unentscheidbaren Zufall:

Wer eine Münze wirft, wird das Resultat als zufällig empfinden — das aber nur deswegen, weil er die Ursachen und Umstände, die dieses Resultat herbeiführen, zu wenig kennt: genaueste Modalitäten der Impilsübertragung, den Einfluss des Luftwiderstandes, kleinste Schwankungen der Luftdichte entlang der Flugbahn etc. Deswegen liegt hier nur scheinbarer Zufall vor.

Im Unterschied dazu sind die Resultate von Quantensprüngen unvorhersagbar. Und das deswegen, weil in der uns erfahrbaren Raumzeit – in dem also, was die Natur uns zeigt — keine Ursachen existieren.

Wenn aber Bohrs Meinung, die Physik könne nicht ergründen, wie die Natur funktioniert, zutrifft, dann könnte es außerhalb aller denkbaren physikalischen Modelle tatsächlich auch für jeden Quantensprung eine Ursache geben. Es wird uns dann aber nicht gelingen, zu entscheiden, ob dem wirklich so ist. FAZIT also:


Gott lässt sich nicht in die Karten schauen.


grtgrt
 

  Beitrag 1911-4
Überlagerungszustand

 
 
Bernhard Kletzenbauer aus 1911-3:
 
Ab und zu hörte, oder las, ich vom "echten" Zufall innerhalb der Quantenwelt. Was hat es damit auf sich?

Das ist so, Bernhard:

In der quantenmechanischen Charakterisierung eines physikalischen Systems ist der momentane Zustand des Systems ein mathematisches Objekt, welches uns
  • für jede am System mögliche (fehlerfreie) Messung und
  • für jedes dabei mögliche Messergebniss
die Wahrscheinlichkeit liefert, mit der eben dieses Messergebnis erhalten wird. Man nennt diesen Zustand deswegen einen Überlagerungszustand.

Wird nun so eine Messung tatsächlich vorgenommen, so wird man genau eines dieser möglichen Messergebnisse beobachten — welches aber, kann man nicht vorhersagen: Die Quantenmechanik geht davon aus, dass die Natur es absolut zufällig auswählt.

Gruß, grtgrt
 

  Beitrag 1911-9
Über den Zufall im Sinne der Quantenphysik (1)

 
 
Harti aus 1911-7:
...schon die Tatsache, dass die Quantenmechanik Wahrscheinlichkeitsvoraussagen machen kann, widerspricht der Annahme eines absoluten Zufalls hinsichtlich der Ursachen für ein Ereignis. Bei Annahme absoluten Zufalls könnte sie nämlich überhaupt keine Voraussagen machen.

Hallo Harti,

es ist genau umgekehrt, als du es hier formulierst. Es ist so:
Weil bei Quanten-Ereignissen der absolute Zufall vorliegt, deshalb kann die Quantenmechanik nur Wahrscheinlichkeitsvoraussagen machen.

Harti aus 1911-7:
Falls man eine Wahrscheinlichkeitsaussage treffen kann, kennt man die Ursachen für ein Ereignis eben nur ungenau.

Du unterstellst dabei stillschweigend, dass für jedes Auftreten eines einzelnen Quantenereignisses immer auch eine Ursache existiert. Dem ist nicht so. Denn wenn dem so wäre, könnte man mit immer präziseren Messmethoden diese Ursachen zutage treten lassen. Das haben bereits viele Physiker versucht, sind aber in den letzten 100 Jahren immer gescheitert. Die Grundannahmen der Quantenmechanik wurden bei allen Experimenten bis heute stets glänzend bestätigt.

Bitte lese noch mal den Beitrag 1911-4 von ggreiter, darin wird die Sache bereits zutreffend geschildert. Nur eine kleine Einschränkung möchte ich dazu machen: Die Natur wählt nicht aus, sondern das quantale Geschehen findet ganz einfach statt und zwar absolut zufällig. Aber das ist vermutlich nur Sprach-Kosmetik.

M.f.G. Eugen Bauhof
 

  Beitrag 1911-10
Über den Zufall im Sinne der Quantenphysik (2)

 
 
Bauhof aus 1911-9:
Harti aus 1911-7:
Falls man eine Wahrscheinlichkeitsaussage treffen kann, kennt man die Ursachen für ein Ereignis eben nur ungenau.

Du unterstellst dabei stillschweigend, dass für jedes Auftreten eines einzelnen Quantenereignisses immer auch eine Ursache existiert.

Hallo Eugen Bauhof,

das will ich eigentlich nicht unterstellen. Ich bin lediglich der Überzeugung, dass wir nicht mit absoluter Sicherheit wissen können, ob es für ein Ereignis eine oder keine Ursache gibt.
Sowohl die positive Aussage "es gibt immer eine Ursache für ein Ereignis" wie die negative Aussage "ein Ereignis hat absolut keine Ursache" sind uns aufgrund eingeschränkter Erkenntnisfähigkeit verwehrt. Die Planck- Größen sprechen m.E. dafür, dass wir absoluten Zufall weder bejahen noch ausschließen können.

Zitat:
Die Natur wählt nicht aus, sondern das quantale Geschehen findet ganz einfach statt und zwar absolut zufällig. Aber das ist vermutlich nur Sprach-Kosmetik.

Die Frage ist, ob mit dem Begriff "absolut zufällig" eine Aussage gemacht werden soll, die jeder wissenschaftlichen Überprüfung entzogen ist. Wenn dem so ist, handelt es sich um eine Glaubensfrage und die Wissenschaft wird zur Religion.

MfG
Harti
 

  Beitrag 1911-12
Über den Zufall im Sinne der Quantenphysik (3) - was Zeilinger sagt

 
 
Harti aus 1911-10:
... Sowohl die positive Aussage "es gibt immer eine Ursache für ein Ereignis" wie die negative Aussage "ein Ereignis hat absolut keine Ursache" sind uns aufgrund eingeschränkter Erkenntnisfähigkeit verwehrt. Die Planck- Größen sprechen m.E. dafür, dass wir absoluten Zufall weder bejahen noch ausschließen können.

Hallo Harti,

ich bezeichne den absoluten Zufall lieber als objektiven Zufall. Ich kenne deine Lernresidenz ja schon seit langer Zeit, aber ich gebe die Hoffnung noch nicht auf. Wenn du schon von mir und von ggreiter nichts annehmen willst, dann vielleicht von einem anerkannten Experten. Anton Zeilinger schreibt auf Seite 46 seines Buches [1 folgendes:

Zitat:
Diesen Überlegungen zufolge tritt der Zufall in der Quantenphysik nicht etwa deshalb auf, weil wir zu dumm sind, um die Ursache für das Einzelereignis zu kennen, sondern weil es einfach keine Ursache für das Einzelereignis gibt, weil das Teilchen einfach keine Information tragen kann, wo es auf dem Interferenzschirm auftreffen soll.

Der Zufall in der Quantenphysik ist also nicht ein subjektiver, er besteht nicht deshalb, weil wir zuwenig wissen, sondern er ist objektiv. Ganz im Sinne Heisenbergs ist es nicht unser Unwissen, von dem wir hier also sprechen, sondern die Natur selbst ist in solchen Situationen in keiner Weise festgelegt, ehe das einzelne Ereignis auftritt.

Mit freundlichen Grüßen
Eugen Bauhof

[1 Zeilinger, Anton
Einsteins Schleier. Die neue Welt der Quantenphysik.
München 2003. ISBN=3-406-50281-4
 

  Beitrag 1911-14
Über den Zufall im Sinne der Quantenphysik (4) - im Vergleich zum online Konto

 
 
Laevicula aus 1911-13:
Das ist aber doch ein Unterschied. Beim Bankkonto könnte man das theoretisch vorher wissen; da weiß man, dass der Kontostand eine Ursache hat.
Könnte!
Das heißt theoretisch könnte man - aber praktisch kann man nicht.

Wenn irgendeine Fehlbuchung zu meinen Gunsten oder zu meinem Nachteil vorgenommen wird, kann ich das nicht wissen, bevor es auf dem Kontoauszug erscheint. Ich kann erst nach dem Lesen des Ausdrucks der Sache nachgehen und vielleicht feststellen, daß ich es hätte wissen können, wenn ich den Brief der Lotteriegesellschaft nicht ungeöffnet weggeworfen hätte.
Meiner Meinung nach gilt in der Quantenwelt und auf dem Konto, daß ich erst dann etwas darüber erfahre, wenn ich nachsehe.
Das heißt aber nicht, daß in der Quantenwelt und auf dem Konto absoluter Stillstand herrscht, wenn ich nicht nachsehe -
und daß sich erst in dem Moment wenn ich nachsehe *hokuspokus* "zufällig" irgendetwas ohne Ursache ereignet.
In diesem Forum ereignen sich auch dann neue Einträge, wenn ich nicht online bin. Es ist nicht so, daß es innerhalb von 24 Stunden eine bestimmte Anzahl "n" Möglichkeiten gibt, von denen *schwuppdiwupp* "zufällig" erst dann eine Realität wird, sobald ich online gehe.
Von all den Möglichkeiten, die bestehen, wird diejenige Realität, die sich aus dem vorherigen Zustand und den wirksamen Naturgesetzen (-Kräften) entwickelt. Eine vom Tisch fallende, gefüllte Tasse kann unter bestimmten Umständen unzerbrochen am Boden ankommen, ohne daß etwas vom Inhalt verschüttet wird. Wenn diese Umstände aber nicht gegeben sind, wird die Tasse am Boden zerbrechen und der Inhalt wird verschüttet.
Der Haken bei der Quantenwelt ist aber, daß jedes Nachschauen die Vorgänge selbst verändert.

Gruß
Bernhard Kletzenbauer
 

  Beitrag 1911-16
Über den Zufall im Sinne der Quantenphysik (5) - Zufall kann weder subjektiv noch objektiv sein

 
 
Liebe Mitdenker,

es macht mich immer wieder traurig, beobachten zu müssen, wie wenig das Volk der Dichter und Denker heute darüber nachdenkt, welches Wort in welcher Situation am besten passt:
  • Das Attribut subjektiv signalisiert dem Leser, dass es um eine subjektspezifische Sicht geht. Wenn also irgendwo von subjektivem Zufall die Rede ist, so macht das nur dann Sinn, wenn klar ist, von genau welchem Subjekt man denn jetzt eigentlich spricht.
  • Da das Attribut objektiv ein Synonym für vorurteilslos ist, ergibt der Ausdruck objektiver Zufall in meinem Augen keinerlei Sinn (auch wer objektiv denkt und vorurteilslos beobachtet, hat noch lange nicht entschieden, ob das, was da Gegenstand seiner Beobachtung ist, nun scheinbarer, physikalischer oder absoluter Zufall ist).
  • Andererseits ist ein absolut zufälliges Geschehen, doch sicher eines, das wirklich keinerlei Ursache hat.
Mit besten Grüßen,
grtgrt
 

  Beitrag 1915-20
Quanten-Kollision

 
 
U...bus aus 1915-15:
 
jetzt komme ich wieder mit meinen querdenkenden Fragen: ....

Hi U...bus,

ich mag querdenkende Fragen: Sie sind die spannendensten und die, die am ehesten zu was Neuem anregen.


U...bus aus 1915-15:
Grtgrt aus 1915-3:
 
Ein Elementarereignis E kann eintreten

entweder spontan (so dass ohne jede erkennbare Ursache ein Paar virtueller Teilchen entsteht oder vergeht)

oder durch Kollision existierender Teilchen (Dekohärenz): Zusammenstoßende Quanten nehmen einander wahr und führen so zum Kollabieren ihrer Wahrscheinlichskeitswelle).

1) spontan = ohne erkennbare Ursache; was sagt das aus? Ohne Ursache oder technisch nicht nachweisbare Ursache?
Ohne Ursache würde das Kausalitätsprinzip infrage stellen, technisch nicht nachweisbare Ursache wäre die Beschränkung der Physik auf meßbare Wirkungen.

ANTWORT: Gemeint habe ich (grtgrt): "Ohne Ursache (im Sinne der Quantenphysik".

Nachdem — wie Niels Bohr uns sagt — die Physik nur zum Gegenstand haben kann, wie die Natur sich uns zeigt (aber nicht, wie sie wirklich ist), können wir nicht ausschließen, dass das, was im Sinne der Quantenphysik OHNE Ursache ist, nicht vielleicht doch eine (uns verborgene) Ursache hat.

Meine Formulierung war gewählt, dieser Tatsache Rechnung zu tragen.

Die Quantenphysik übringens hat das Kausalitätsprinzip nicht nur in Frage gestellt, sondern wirklich über Bord geworfen.
Einstein war der letze Spitzenphysiker, der an das Kausalitätsprinzip glaubte und es NICHT in Frage gestellt sehen wollte. Damit, so denkt man heute, hatte er wohl unrecht.


U...bus aus 1915-15:
 
2) Kollision existierender Teilchen: In was stoßen diese Teilchen zusammen? Die Dinger müssen sich ja irgendwo aufhalten, es stellt sich also wieder mal die Frage nach dem "Behälter Raumzeit".

Wenn jetzt das Universum aus Wellenfunktionen besteht und aus nichts anderem, dann müssen doch diese Wellen den Raum bilden. Und tun sie das, dann bilden diskrete Wellenfunktionspakete räumlich begrenzte Entitäten, sonst könnten sie ja nicht zusammenstoßen, ...

ANTWORT: Jede Wellenfunktion hat im ganzen Universum Werte, nenneswert von Null verschieden sind die aber nur an (und in einer kleinen Umgebung von) wenigen ausgezeichneten Stellen. Dort, so sagt man, sei das Quantum mit nenneswerter Wahrscheinlichkeit anzutreffen.

Urbildmenge so einer Wellenfunktionen ist grundsätzlich der gesamte Raum.

Die Werte jeder Wellenfunktion sind zudem zeitabhängig.


Unter der Kollision zweier Quanten verstehe ich, dass sich die Stellen, an denen ihre Wellenfunktionen nennenswerten Wert haben, sehr nahe kommen.
Aus Sicht des Betrachters
  • zeigt die Summe beider Funktionen zunächst an zwei Stellen nennenswerten Wert.
  • Diese beiden Stellen wandern aufeinander zu, werden also zu einer Stelle mit nenneswerten Wert (der aber exponentiell abfällt mit zunehmender Entfernung von dieser Stelle),
  • und diese eine Stelle teilt sich dann in mehrere auseinander wandernde Stellen, an denen sich nennenswert von Null verschiedene Werte zeigen.

Die Zahl der auseinander wandernden Stellen ist die Zahl der Quanten, in die sich die beiden kollidierenden Quanten neu aufgeteilt haben.

Gruß, grtgrt
 

  Beitrag 1915-36
Auf hinreichend kleiner Größenskala verklingt Materie im nur noch gedanklich Existierenden

 
 
U...bus aus 1915-22:
 
... ich versteh bis heute die Physik nicht, warum sich dort die Erfahrung der Makrophysik nicht auch in der Mikrophysik niederschlägt. Jeder, der sich in die Badewanne setzt, verdrängt das Badewasser, warum sollte das im Quantenbereich der Natur nicht genauso sein?

Gruß

Hi U...bus:

Jede Person P, die sich in die Badewanne setzt, ist — aus quantenphysikalischer Sicht — nichts weiter als ein System von Quanten.

Quanten aber sind — so schrieb Heisenberg — Objekte, die "als Potentia in einer seltsamen Wirklichkeit zwischen der Idee von einem Ding und einem wirklichen Ding existieren". Wie aber soll "Potentia" Wasser verdrängen?


Die Antwort auf deine Frage also ist:

Sobald man ein makroskopisches Objekt P in zunehmend kleinere Teile zerlegt, kommt man irgendwann in eine Größenordung, in der die so entstanden extrem kleinen Teilchen — einzeln betrachtet — mehr und mehr nur noch  g e d a n k l i c h  existieren.

Das klingt verrückt, scheint aber wirklich so zu sein.
Ich tröste mich da einfach mit Richard Feynman's Feststellung:


Die Quantenphysik kann man nicht verstehen.

Wer glaubt, er hätte sie verstanden, der hat sie überhaupt nicht verstanden.


Gruß,
grtgrt
 

  Beitrag 1915-43
Über Dekohärenz (1)

 
 
E... aus 1915-38:
Auf ein neues...

Jede Wechselwirkung, sei es mit anderer Materie oder mit Strahlung, lässt ein isoliertes Quantensystem sofort kollabieren. Dieser Vorgang ist nicht rückgängig zu machen, er ist irreversiebel. Das untersuchte Teilchen wird klassisch. Nicht weil wir es so sehen wollen... sondern weil es so geschieht. ...

Mir wirst Du wohl diesesmal auch keinen Glauben schenken. Deshalb anbei noch ein Beitrag aus der Reihe alpha-Centauri. Professor Harald Lesch Jahrgang 1960 Astrophysiker, Naturphilosoph und Professor für Physik an der LMU München fragt dort "Was ist Dekohärenz?" und erklärt anschaulich um was es dabei geht.

http://www.br.de/fernsehen/br-alpha/sendungen/alpha...

Hi E...,

zunächst mal danke, dass du dich jetzt doch entschlossen hast, mir zu sagen, woher dein Wissen stammt.

Ich muss zugeben: Alles, was ich bisher von dir zum Thema "Dekohärenz" gehört habe, gibt korrekt (und fast wörtlich) wieder, was Lesch verkündet.

Aber verstehen wir ihn richtig?


Lass uns dazu mal folgendes Beispiel betrachten:

Nimm ein Wasserstoff-Atom — ein Atom also, das nur ein oder zwei Elektronen hat. Lesch sagt ja nun zweierlei:
  • Erstens: Jedes sich irgendwo im Universum aufhaltende Elektron wechselwirkt ständig mit seiner Umgebung — mindestens mit den Energiequanten, die die allgegenwärtige kosmische Hintergrundstrahlung bilden.
  • Zweitens: Dekohärenz sei nicht rückgängig zu machen, habe also bleibenden Effekt.

Wenn beide Aussagen (in dieser einfachen Form wenigstens) richtig wären, müsste ja sogar JEDES Elementarteilchen schon kleinste Sekundenbruchteile nach seinem Entstehen klassisch werden. Damit wären sämtliche Atome wirklich wie Sonnensysteme, um deren Kern herum sich die Elektronen in Form klassischer Teilchen bewegen. Deren nicht klassischen Zustand hätten Menschen dann ja wohl gar nie mitbekommen können.

Wie also löst sich dieser Wiederspruch?

Dekohärenz ist wie ein Blitz, dessen Wirkung sofort wieder vergeht. Das Stück Kreide, das Lesch uns da zeigt, sehen wir aber dennoch, denn es gibt halt in ununter­brochener Reihenfolge sehr viele solcher Blitze. Mit anderen Worten: Quanten wechseln ständig vom virtuellen (Überlagerungs-) Zustand in einen klassischen und zurück in den virtuellen: Klassisch sind sie nur während einer extrem kurzen Zeitspanne, die mit einem Elementarereignis beginnt, an dem sie teilhaben.

Lesch hat sich in seiner Sendung wohl einfach nur ungenau ausgedrückt.

Meine eben dargelegte Interpretation sehe ich auch bestätigt durch eine Aussage, die ich in Lothar Schäfers Buch "Versteckte Wirklichkeit" fand. Er spricht dort vom Orbital­modell der Atome und erklärt, dass die sog. Knotenflächen — bestehend aus der Menge aller Punkte, an denen die Aufenthaltswahrscheinlichkeit für Elektronen zu Null wird — den Raum in getrennte, sich nirgendwo überlappende Bereiche zerlegen, in denen sich dann je ein Elektron finden bzw. nicht finden kann. Das Interessante für unser Problem ist nun, dass, wenn man ein Atom beobachtet, welches mehr solcher Kammern hat als Elektronen, diese Elektronen sich einmal in einer, dann aber in einer anderen Kammer zeigen. Auf ihrem Weg hin von einer zur anderen aber kann man sie NICHT beobachten.

Das nun sehe ich als einen unwiderlegbaren Beweis dafür, dass so ein Elektron eben NICHT ständig klassisch sein kann.


Beste Grüße,
grtgrt
 

  Beitrag 1915-57
-

 
 
Grtgrt aus 1915-51:
 
Henry aus 1915-44:
Noch was: Ich verfolge Lesch´s Sendungen schon seit Langem, ich kann dir versichern, er meint es genau so und hat sich keines Wegs ungenau ausgedrückt.

Auch ich schätze Lesch sehr. Dennoch bin ich nicht bereit zu glauben, dass nicht auch ein Meister mal irren kann (z.B. unbeabsichtigt wider besseres Wissen).

Hallo Henry,

in "Bedeutende Theorien des 20. Jahrhunderts - Relativitätstheorie, Kosmologie, Quantenmechanik und Chaostheorie", einem Buch von Werner Kinnebrock (Professor für Mathematik & Informatik an der FH Rheinland-Pfalz) findet sich auch ein Kapitel über Dekohärenz. Es schließt mit dem Satz:

"Daher erfolgt der Wellenkollaps permanent, Superpositionen entstehen, aber sie zerfallen sofort wieder."


Ist das nicht genau das, was auch ich sage (und als die einzige logisch konsistente Variante des Dekohärenz-Geschehens empfinde)?

Gruß, grtgrt


PS: Ganz allgemein bin ich der Meinung, dass man ein Theorem erst dann richtig versteht, wenn man auch seine Begründung kennt und logisch überzeugend findet.
Die Begründung einzusehen sollte uns als Beleg des Theorems wichtiger sein als irgend ein — vielleicht doch nicht genau genug formuliertes — Statement von Lesch oder sonstwem.

Würdest du mir da rechtgeben?

 

  Beitrag 1915-66
Über Dekohärenz (2)

 
 
Hi E...,

ich erkläre mir Dekohärenz (= Interaktion eines Quantums mit seiner Umgebung) wie folgt:
  • Wenn ein Quantum Q entsteht, existiert es in seinem Überlagerungszustand.
  • Interaktion mit seiner Umgebung bedeutet, dass das Q mit einem anderen Quantum Q2 verschmilzt, und dass das Ergebnis dieser Verschmelzung sich neu in Quanten Q' aufteilt.
  • So ein Q' kann dem Q mehr oder weniger ähnlich sein.

Mit anderen Worten:

Was wir (bzw. unsere Messgeräte) zu Gesicht bekommen ist keineswegs Q in einem konkret gewordenem (Einzel-) Zustand, sondern ist einfach nur der neue, makroskopisch beobachtbare Zustand eines Quantensystems QS, in dessen Zentrum Q und Q2 eben durch neue Quanten Q' ersetzt worden sind. Solche Ersetzung führt zu einer Abänderung der Wellenfunktion von QS, was wiederum chemische Reaktion zur Folge haben kann (z.B. Schwärzung einer Fotoplatte an einer ganz bestimmten Stelle).


Meine Interpretation beobachtbaren Quantenverhaltens ist demnach:

Jedes Quantum Q existiert grundsätzlich NUR in dem, was die Kopenhagener Deutung seinen "Überlagerungszustand" nennt.

In Extrapolation dessen, was Richard Feynman dachte (dass dieses Qantum dann nämlich mehr oder weniger überall gleichzeitig ist), sehe ich seinen Überlagerungs­zustand als das Schwingen einer Energieportion, welches örtlich konzentriert erscheint und mit zunehmender Enfernung von diesem nur grob definierten Punkt im Raum schnell an Intensität abnimmt (ebenso wie durch einen massereichen Körper erzeugte Gravitationswellen dies tun: Auch sie sind 3-dimensional, verklingen schnell mit zunehmender Entfernung von diesem Körper, werden aber dennoch nirgendwo im Universum wirklich komplett zu Null).

Was wir als die Bewegung eines Quantums durch den Raum interpretieren, ist die Verschiebung des Ortes, an dem dieses Quantum maximale Intensität hat.


Gruß, grtgrt
 

  Beitrag 1915-86
Über mikroskopische (virtuelle) und makroskopische (reale) Wirklichkeit.

 
 
Henry aus 1915-80:
 Der Messvorgang "enthüllt" den realen Zustand.

Hi Henry,

die überwiegende Zahl aller Quantenphysiker glaubt heute, dass der Messvorgang den realen (Quanten-) Zustand nicht enthüllt, sondern erst erzeugt.

Davon ausgehend bin ich der Auffassung, dass
  • mikroskopisch gesehen (d.h. auf der quantenphysikalischen Skala) die Wechselwirkung eines Quantensystems mit seiner Umgebung (Messung ist da nur ein Beispiel) aus Elementarereignissen E besteht, deren jedes das System von einem Zustand Z1 in einen Zustand Z2 überführt. Beide sind — so sehe ich das — virtuell, also Überlagerungszustände.
  • Auf makroskopischer Ebene aber (das sei unsere Beobachtungsebene) ergibt sich in Summe solcher Elementarereignisse eine durch uns registierbarer Effekt (der auch unabhängig von uns, dem Beobachter also, existiert). Nur er ist das, was wir als "eindeutig real" bezeichnen können.

Werner Kinnebrock schreibt:
    Die Realität der Wellenfunktion [er nennt sie die mikroskopische Realität, die wegen der Unschärferelation nicht weiter auflösbar ist "ist für uns nicht wahrnehmbar. Das, was wir wahrnahmen, ist eine übergeordnete Realität, die aus Beobachtungen und damit aus lauter Zusammenbrüchen der Wellenfunktion besteht. Wenn wir unter » Realität « nur die Form der Wahrnehmungen verstehen, müssen wir feststellen, dass Realität erst durch Beobachten entsteht. Und da wir die Konditionen der Wahrnehmung selbst bestimmen, gehen einige Physiker so weit, dass sie sagen, die Realität wird erst durch uns geschaffen."

All das fasse ich zusammen zur Feststellung:
  • Die durch unsere Sinne registrierbare Wirklichkeit [ makroskopische Wirklichkeit: unsere Erfahrungswelt existiert unabhängig vom Beobachter — sie ist vom Beobachter trennbar.
  • Im Mikroskopischen aber (dort, wo die Unschärfe-Relation regiert) gibt es nur virtuelle, durch Überlagerungszustände gegebene Wirklichkeit. Sie kennt keine Trennung zwischen Beobachter und Beobachtetem, da beide völlig symmetrisch miteinander interagieren und im Zuge solcher Interaktion als nicht trennbare Teile eines Gesamtsystem modifiziert werden.

Gruß, grtgrt

PS: Meine Theorie besteht lediglich in der Deutung des Zusammenbrechens der Wellenfunktion: Was aufhört zu existieren, ist die Wellenfunktion von Zustand Z1 (und das einfach deswegen, weil er durch ein Elementarereignis zerstört wird). Sie wird ersetzt durch die Wellenfunktion des Folgezustandes Z2. Beides sind Zustände des Gesamtsystems, zu dem auch der Beobachter selbst gehört. Die Wellenfunktion des beobachteten Teilchens existiert nur rein rechnerisch — was wirklich regiert ist stets die Wellenfunktion des Gesamtsystems (!).

Soweit ich erkennen kann, steht diese Deutung in keinerlei Widerspruch zur Dekohärenz-Theorie von Zeh.
Sie nämlich beweist uns ja gerade, dass kein echtes Teilsystem eines Quantensystems autonom sein kann.


 

  Beitrag 1915-89
Erst Messung (Quanteninteraktion) erzeugt den konkreten Zustand - und der existiert nur an genau einem Punkt der Raumzeit !!!

 
 
Henry aus 1915-80:
Der Überlagerungszustand (Superposition) ist die Beschreibung von Möglichkeiten und nicht physikalisch real (Schrödingers Katze ist entweder tot oder lebendig). Der Messvorgang "enthüllt" den realen Zustand.

Wenn "Messvorgang" äquivalent zu Reaktion beliebiger Teilchen miteinander ist – wovon ich ausgehe – also dekoharänt über den gesamten Kosmos betrachtet, so ist unser Kosmos physikalisch real. Wäre die Annahme richtig, der Überlagerungszustand wäre real, so befände sich der Kosmos – uns eingeschlossen – permanent im Zustand der Nichtrealität. Henry

Hallo Henry,

da kann ich zustimmen, außer der Bemerkung ’Der Messvorgang "enthüllt" den realen Zustand’.

Vor der Messung existiert überhaupt kein realer Zustand eines Quantenobjekts, sondern eben nur die Beschreibung von Möglichkeiten durch die Bornsche Wahrscheinlichkeitswelle. Erst nach der Messung wird das Quantenobjekt manifest, z.B. durch die Schwärzung einer lichtempfindlichen Platte. Ein vermeintlich vorher existierender realer Zustand kann nicht enthüllt werden, weil dieser vor der Messung nicht existiert.

Vermutlich weißt du das, du hast dich nur unglücklich ausgedrückt.

M.f.G. Eugen Bauhof
 

  Beitrag 1915-90
-

 
 
Bauhof aus 1915-89:
 
Erst nach der Messung wird das Quantenobjekt manifest, z.B. durch die Schwärzung einer lichtempfindlichen Platte. Ein vermeintlich vorher existierender realer Zustand kann nicht enthüllt werden, weil dieser vor der Messung nicht existiert.

Das Beispiel "Schwärzung einer lichtempfindlichen Platte" empfinde ich als besonders lehrreich, denn es ist Teil unserer (makroskopischen) Erfahrungswelt
im Sinne von Beitrag 1915-86.

grtgrt
 

  Beitrag 1915-107
-

 
 

Nochmals zur Funktionsweise von Dekohärenz:


In Ergänzung meiner Beiträge 1915-66 und 1915-86 sowie Eugen Bauhofs Aufforderung 1915-79 sei festgestellt, dass Andreas Mücklich (promovierter Physiker und Astronom) auf Seite 198 seines 2011 erschienenen Buches "Das verständliche Universum" die Kopenhagener Deutung — den Kollaps der Wellenfunktion betreffend — wörtlich wie folgt beschreibt:

    Vor der Messung [eines Elektrons am Doppelspalt ist sie [die Wellenfunktion des Elektrons ausgedehnt über beide Spalte verteilt, und danach ist sie fast punktförmig an dem Ort lokalisiert, an dem das Elektron gemessen wurde. Wegen dieses anschaulichen Bildes spricht man auch vom "Kollaps der Wellenfunktion", der bei einer Messung stattfindet. Die Wellenfunktion schnurrt dabei auf einen Bereich zusammen, der so groß ist wie die Genauigkeit der Ortsmessung. Ab diesem Zeitpunkt wird das Verhalten der neuen Wellenfunktion wieder durch die Schrödinger-Gleichung beschrieben. Lediglich während der Messung ist sie außer Kraft gesetzt.


Das, so denke ich, bestätigt meine in 1915-66 und 1915-86 dokumentierte Auffassung, dass


der Zusammenbruch der Wellenfunktion eben NICHT etwas Bleibendes ist,

sondern etwas sich ständig Wiederholendes.



Nebenbei: Mücklich verschweigt nicht, dass bis heute über die Kopenhagener Deutung hinaus weitere Kollaps-Theorien diskutiert werden, u.A.
  • Hugh Everetts Viele-Welten-Theorie und
  • ein Erklärungsvorschlag von Roger Penrose, in dem Gravitationskräfte eine wesentliche Rolle spielen.

Das letzte Wort also scheint den Physikern noch nicht gesprochen zu sein. Warum also sollten dann nicht auch wir hier weiter nach besonders logischen Erklärungen der ständig stattfindenden Dekohärenz-Prozesse suchen?

grtgrt

 

  Beitrag 1915-109
Der Film, der darstellt, was wir als die uns umgebende physikalische Realität empfinden

 
 
E... aus 1915-108:
 
5. Durch den Mechanismus der Dekohärenz zerfällt also ein beliebig präpariertes System ohne weiteres Zutun in einen Zustand, der in der Pointer-Basis diagonal ist. Einmal dort angelangt, verbleibt es in diesem "effektiv klassischen", nicht interferierenden Zustand.
Quelle: https://www.tu-braunschweig.de/Medien-DB/ifdn-physi...

Hi E...,

deine Aussage "Einmal dort angelangt, verbleibt es in diesem "effektiv klassischen", nicht interferierenden Zustand" ist eindeutig falsch.

Das ist keineswegs nur meine Meinung (zu der ich über rein logische Erwägungen kam), sondern auch die im Buch "Das verständliche Universum" (2010) von Andreas Mücklich dargestellte. Er kennt Zeh und seine Theorie sehr gut und hat in Heidelberg studiert: an der Hochschule also, an der Zeh lehrt.

Tatsache ist:

Wo ein Quantenobjekt Q mit einem Messgerät (oder mit irgend einem anderen Quantenobjekt) interagiert, modifiziert das die Wellenfunktion beider.
Eben diese Modifikation ist das, was die Kopenhagener Deutung als den "Kollaps der Wellenfunktion" von Q bezeichnet.

Dieser "Kollaps" aber ist gleichzeitig der Geburtsmoment einer neuen Version der Wellenfunktion, während dessen die miteinander zusammenstoßenden Quanten durch neue Quanten ersetzt werden (im Extremfall durch solche, die von einigen der vorher existenten gar nicht unterscheidbar sind).

Beispiel: Wo immer man das Elektron eines Wasserstoffatoms beobachtet, ersetzt diese Beobachtung das Elektron durch ein neues Exemplar (und so ist es kein Wunder, dass jenes sich dann, wenn es wieder beobachtet wird, oft in einem anderen Orbital zeigt).

Da alle Materie mindestens mit der kosmischen Hintergrundstrahlung ständig interagiert, passieren Vorgänge dieser Art tatsächlich am laufenden Band (bei normalem Luftdruck so etwa alle 10-31 sec).

Jede Quanteninteraktion produziert eine Art Schnappschuss der Situation beim "Kollaps", und all diese Schnappschüsse zusammen bilden eine Art Film, der uns das zeigt, was wir als die uns umgebende Wirklichkeit wahrnehmen.


Gruß, grtgrt
 

  Beitrag 1915-112
Ein SEHR treffender Vergleich

 
 
Irena aus 1915-111:
 
Grtgrt aus 1915-109:
 
Jede Quanteninteraktion produziert eine Art Schnappschuss der Situation beim "Kollaps", und all diese Schnappschüsse zusammen bilden eine Art Film, der uns das zeigt, was wir als die uns umgebende Wirklichkeit wahrnehmen.

Ich habe es unter Stichwort Fakten gelernt. Die Quantenobjekte erzeugen durch die Wechselwirkungen ständig die Fakten, die unsere Realität bilden. Der Fakt ist eine Äußerung eines Objektes, die in einer Wechselwirkung erzwungen wird. Der Quantenobjekt muss sich für eine der Möglichkeiten entscheiden. Es bildet somit ein Faktum.

Ob es beim Wellenfunktion ein Kollaps stattfindet ist dahintergestellt. Ich stelle mir jedenfalls eine Analogie mit dem Menschen, dessen Gedankenverlauf ist uns nicht zugänglich, solange er sich nicht äußert.


Hallo Irena,

das ist ein sehr treffender Vergleich — ich werde ihn mir gut merken.

Vielen Dank für diese schöne Idee!

Mit besten Grüßen,
Gebhard Greiter (grtgrt)

 

  Beitrag 1915-116
Quantensysteme und die Born-Oppenheimer-Näherung

 
 
E... aus 1915-114:
 
Noch ist kein natürlich entstandenes Quantensystem beobachtet worden.

Doch E..., es gibt jede Menge solcher Systeme,

denn wirklich JEDES Molekül ist ein Quantensystem, dessen Teile einzeln zu betrachten nicht immer Sinn macht (jeder Chemiker wird dir das bestätigen).

Wo man — weil eine Gesamtbetrachtung allzu schwierig und allzu teuer werden würde — das Molekül dennoch nur als Summe seiner Teile sieht, nutzt man meist die sog. Born-Oppenheimer-Näherung. Sie führt zu ganz passablen Ergebnissen für Moleküle im Grundzustand, insbesondere bei denen mit schweren Kernen. Für angeregte Moleküle und geladene Ionen allerdings führt sie zu sehr schlechten, oft kaum brauchbaren Ergebnissen, was man besonders für Anwendungen im Bereich der Photoelektronenspektroskopie zu beachten hat.

Mit anderen Worten: Die Photoelektronenspektroskopie beweist, dass ein Molekül eben doch  n i c h t  nur die Summe seiner Teile ist, sondern in der Tat ein wirkliches Quanten-SYSTEM.


Zitat von E... :
 
Ebenso ist bei allen Experimenten an und mit Quantensystemen nie beobachtet (oder gemessen) worden das die Präperate wieder in den Kohärenzzustand zurückfallen.

Als Ganzes tun sie das tatsächlich nicht, da ja ständig irgend ein Quantenobjekt, welches Teil des Systems ist — z.B. eines Moleküls — mit Quanten, die das System durchqueren wollen, kollidiert. Denk an die kosmische Hintergrundstrahlung!


Gruß, grtgrt
 

  Beitrag 1915-117
-

 
 
Wrentzsch aus 1915-115:
 
Also alles, was wir nicht messen, ist keine Tatsache!

Das ist falsch. Richtig wäre, zu sagen:

Was NIEMALS mit seiner Umgebung interagiert, ist keine Tatsache (kann also nur als Möglichkeit existieren).

grtgrt
 

  Beitrag 1915-120
Was Quantensysteme charakterisiert

 
 
Gregor Lämmer aus 1915-118:
 
Grtgrt aus 1915-1:
 
Meine These 1:
  • Das Universum ist ein Quantensystem definiert durch seine Wellenfunktion.
Meine These 2:
  • Es kann mir niemand beweisen, dass diese Sicht falsch ist.
 
Kannst Du beweisen, dass sie richtig ist?

Hi Gregor,

ob der Beweis möglich ist, hängt vor allem davon ab, wie du den Begriff "Quantensystem" definiert siehst.
Vorschlag also: Nenne mir deine Definition, so werde ich dann versuchen, den Beweis zu finden.

Nebenbei: Solange meine These nicht als falsch nachweisbar ist, kann sie nicht schlechter sein als irgend eine andere, deren Richtigkeit man auch nicht beweisen (sondern höchstens vermuten) kann.

Ich selbst verstehe unter einem "Quantensystem"


eine Gruppierung von Quanten,
die dicht genug ist, dass die Wahrscheinlichkeiten, mit denen sich mindestens eines jener Teilchen an einem bestimmten Punkt befindet,
durch seine eigene Wellenfunktion nicht mehr genau genug beschrieben sind.



Gruß, grtgrt
 

  Beitrag 1915-124
-

 
 
E... aus 1915-121:
 
Grtgrt vertritt die Auffassung, quantenphysikalische Eigenschaften der Materie würden sich permanent und ungeachtet der bestehenden Wechselwirkungen mit der Umgebung realisieren. Überall und jeder Zeit.

Da geht es nicht um Wechselwirkungen zwischen einem Präparat und dem Messinstrument oder der Laboreinrichtung.
 

An alle,

was E... mir hier — in Form der Aussagen, die ich im Zitat rot zeige, unterstellt, ist UNSINN und etwas, das ich NIE behauptet habe. Ganz im Gegenteil:

Wer meine Ausführungen oben (z.B. in 1915-109) gelesen hat, wird erkennen, dass das, was ich dort als Zusammenstoß von Quantenobjekten bezeichne, eben jene Wechselwirkung ist.


Ich weise explizit darauf hin, dass ich die beiden folgenden Aussagen für absolut zutreffend halte:
  • Der einfache Grundgedanke der Dekohärenz, daß ein System nicht isoliert von seiner natürlichen Umgebung betrachtet werden darf, hat sich als überraschend fruchtbar erwiesen.
  • Alle Systeme werden mit einer ständig wachsenden Umgebung verschränkt und können somit normalerweise keine reinen Quantenzustände für sich selber besitzen. Quantenzustände sind grundsätzlich nicht einfach nur lokale Zustände.

grtgrt
 

  Beitrag 1915-122
-

 
 
Gregor Lämmer aus 1915-118:
 
Kein Geringerer als Werner Heisenberg hat den Ursprung sehr treffend formuliert mit der Aussage:

"Der erste Trunk aus dem Becher der Naturwissenschaft macht atheistisch, aber auf dem Grund des Bechers wartet Gott".


Es geht mir keinesfalls um religiöse Propaganda. Das ist für mich einfach Logik pur.


Hierin, Gregor, sind wir beide einer Meinung.


Gruß, grtgrt
 

  Beitrag 1999-8
Was Quantensysteme von klassischen Systemen unterscheidet

 
 

Was Quantensysteme von sog. "klassischen" Systemen unterscheidet


Unter einem System versteht man in Bezug zueinander gesetzte Objekte. Man nennt das System klassisch, wenn das System allein unter Berücksichtigung der örtlichen Position seiner Teile untereinander – und natürlich der Teile selbst – ausreichend gut beschrieben ist.

Jedes System wird erst durch Abstraktion klassisch, macht in dieser Form aber nicht immer Sinn, insbesondere dann nicht, wenn zwischen seinen Teilen Kräfte wirken oder andere, noch kompliziertere Abhängigkeiten bestehen (wie etwa Quantenverschränkung). Abhängigkeiten nämlich haben eine Aufweichung der Identität der Teilobjekte des Systems zur Folge, die so weit gehen kann, dass einige der Teile ihre Identität völlig verlieren – jene also undefiniert wird. Zwei Beispiele machen das klar:

  • Ein Kochsalzkristall ist eine Gruppierung von Natrium- und Chlorionen, die so lose ist, dass der Kristall, wenn man ihn ins Wasser wirft, sich einfach auflöst in eine Menge eben genau solcher Ionen: Sie gehen auseinander mit derselben Identiät, die sie hatten, als sie zusammenfanden.
  • Ganz anders, wenn sich ein Elektron und ein Positron zu einem System mit der Gesamtladung Null zusammenfanden. Es kann sich zerlegen in zwei Gammaquanten, und man wird sich kaum etwas Verschiedeneres vorstellen können als ein Elektron und ein Positron bzw. zwei Photonen (die Gammaquanten): Ein Elektron besitzt elektrische Ladung, Ruhemasse und ein magnetisches Moment. Ein Photon dagegen hat keine Ruhemasse, bewegt sich stets mit Lichtgeschwindigkeit, und besitzt weder Ladung noch ein magnetisches Moment.

Wir sehen:

Gibt es zwischen Systemteilen hinreichend starke Abhängigkeiten (z.B. durch Kräfte gegebene), so bewirken die,
dass die Eigenexistenz der Teile ganz oder teilweise aufgehoben wird
und damit ein neues, qualitativ anderes Ganzes entsteht.


Interessant ist nun:

Ausgedehnte Ganzheiten — der menschliche Körper ist ein Beispiel, die Gesellschaft ein anderes — machen NUR Sinn, wenn man sie im eben definierten Sinn als Quantensysteme begreift: als Systeme, die gut vergleichbar sind mit Mengen zueinander verschränkter Quanten.

Quantensysteme sind also nicht notwendig mikroskopisch klein: Sie können jede nur denkbare Ausdehnung haben. Sebst Systeme verschränkter Photonen mit einer Ausdehnung von gut 100 km können heute gezielt erzeugt und kontrolliert werden.



Der wesentliche Unterschied zwischen klassischer Physik und Quantenphysik besteht darin, dass
  • die klassische Physik durch Abstraktion und Reduktion zu exakten (dann aber nur näherungsweise gültigen Lego-) Modellen kommt,
  • während die Quantenphysik genau sein möche und so sogar noch entdeckt, wo die Natur unbestimmt wird (und jene Unbestimmtheit zu untersuchen und zu quantifizieren gestattet).
    Dies führt zu Systemmodellen, die berücksichtigen, dass die Teile jeden Systems nur noch mehr oder weniger unscharf definierte Identität haben und diese Unschärfe Folge der Tatsache ist, dass zwischen ihnen Kräfte wirken, die — und deren Wirkung — man in Modellen, die genau sein wollen, nicht vernachlässigen darf.


Was » exakt « von » genau « unterscheidet:


Ein Kreis im Sinne der Mathematik ist  e x a k t , in der Natur real auftretende Kreise dagegen sind es nicht (schon allein der Unbestimmtheitsrelation wegen). Dass und in welchem Ausmaß sie NICHT exakt sind, wird erst klar, wenn man sie  g e n a u  betrachtet.

 
grtgrt (einer Argumentation von Thomas Görnitz folgend)

 

  Beitrag 1999-83
-

 
 
Grtgrt in 1999-8:
 

Was Quantensysteme von sog. "klassischen" Systemen unterscheidet


Unter einem System versteht man in Bezug zueinander gesetzte Objekte. Man nennt das System klassisch, wenn das System allein unter Berücksichtigung der örtlichen Position seiner Teile untereinander – und natürlich der Teile selbst – ausreichend gut beschrieben ist.

Jedes System wird erst durch Abstraktion klassisch, macht in dieser Form aber nicht immer Sinn, insbesondere dann nicht, wenn zwischen seinen Teilen Kräfte wirken oder andere, noch kompliziertere Abhängigkeiten bestehen (wie etwa Quantenverschränkung). Abhängigkeiten nämlich haben eine Aufweichung der Identität der Teilobjekte des Systems zur Folge, die so weit gehen kann, dass einige der Teile ihre Identität völlig verlieren – jene also undefiniert wird.
Wir sehen:

Gibt es zwischen Systemteilen hinreichend starke Abhängigkeiten (z.B. durch Kräfte gegebene), so bewirken die,
dass die Eigenexistenz der Teile ganz oder teilweise aufgehoben wird
und damit ein neues, qualitativ anderes Ganzes entsteht.



Hans Dieter Zeh drückt das so aus:

Zitat von H. D. Zeh (1993):
 
Die Kinematik der Quantentheorie erlaubt Quantenkorrelationen ("Verschränkungen") zwischen zwei beliebigen Systemen mit möglichen Zuständen φi und phi;k in der Form von Superpositionen der Art


psi;   =   Σ ci,k φi phi;k .


Diese  n i c h t l o k a l e  Kinematik beschreibt den wohl wichtigsten Unterschied zur klassischen Physik, indem


der Gesamtzustand eines zusammengesetzten Quantensystems im allgemeinen  k e i n e  Zustände der Teilsysteme mehr definiert.

Er kann dann also seinerseits auch nicht durch solche bestimmt sein.


Trotz der bewährten statistischen Aspekte der Quantenmechanik sind diese Korrelationen nicht als rein statistisch bedingt zu deuten. Sie bestimmen vielmehr auch eindeutig objektiv nachprüfbare ("reale") Eigenschaften des individuellen Gesamtzustands (wie z.B. einen Gesamtdrehimpuls).
 

 

  Beitrag 1963-3
Zum Effekt sog. » kalter Verschweißung «

 
 
E... aus 1963-2:
 
Wenn man zwei Stahlwürfel von zum Beispiel 50 mm Kantenlänge deren Oberflächen gehärtet (60 HRC nach Rockwell) und feinstgeschliffen sind mit sauberen und fettfreien Flächen aneinanderschiebt und so länger als 8 Stunden liegen lässt stellt man fest das die Flächen kalt verschweist sind.

Ansprengen nennen wir (die Metaller) das. Diese Verbindung kommt dem direkten Schweißen sehr nahe. Die atomaren Strukturen jedes Würfels sind dabei, bedingt durch die hohe Oberflächengüte, in der Lage sich mit den Atomen des jeweiligen gegenüberliegenden Würfels zu einem stabilen Gitter zu verbinden. Ein voneinander Lösen beider Teile ist dann ohne die Zerstörung der Oberflächen und deren Güte nicht mehr möglich.

Bei den von Dir behaupteten ständigen Dekohärenzprozessen die ja immer vorher Kohärenz erfordern wären solche Eigenschaften des Werkstoffes Stahl nicht möglich. Wie sollten sich Atomgitter vereinigen wenn sie permanent instabil sind?

Hi E...,
sei auch du von mir gegrüßt.

Der Effekt, den du hier beschreibst, ist interessant, darf aber nicht mit dem Dekohärent-Werden der Eisenatome (genauer: ihrer Teile) verwechselt werden.

Er ist vielmehr Folge der ständigen Störung jener Atome durch den immer wieder passierenden Zusammenstoß ihrer Quanten mit solchen, die die Eisenwürfel zu durchqueren suchen. Genauer:

Jeder Zusammenstoß führt zu einem leichtem Umbau des Quantensystems an der Stelle, an der so ein Zusammenstoß passiert. Da in Eisenatomen sehr starke Kräfte wirken, sind die in der Lage während der kurzen Zeit eines solchen Umbaus jede sich ihnen bietende Chance, die beiden Eisenwürfel dichter aneinander zu bringen, auch tatsächlich zu nutzen. In der Summe ergibt sich so jene kalte Verschweißung.

Danke für das schöne Beispiel,
mit besten Grüßen
grtgrt
 

  Beitrag 1915-141
Dekohärenz genau betrachtet

 
 
E... aus 1915-139:
 
Natürlich entfaltet sie [die Dekohärenz sich in sehr kurzer Zeit. Das heißt aber nicht, dass sie nur sehr kurze Zeit Bestand hat. Dann wäre sie n i c h t irreversibel.
Die Dekohärenz ist für ein Quantensystem entgültig und zwar solange bis man das Wellenpaket wieder von allen äußeren Einflüssen befreit, also völlig isoliert.

Hi E...,

könnte es sein, dass du dich hier einfach nur zu ungenau ausdrückst? Denn:

Was nicht sofort rückgängig gemacht wird, ist der neue Zustand des Quantensystems, welcher sich aufgrund seiner Kollision mit einem diesem System zu nahe gekommenen anderen System (dem "Messapparat") eingestellt hat. Das ist eine Sache.

Eine andere Sache aber ist, dass jener neue Gesamtzustand natürlich wieder nur ein virtueller ist — "zunächst bleibend" geändert hat sich nur die ihn beschreibende Wellenfunktion.

Was konkret wurde, war ein mit gewisser Wahrscheinlichkeit möglicher Zustand der zusammenstoßenden Quanten zum Zeitpunkt ihres Zusammenstoßes (weswegen ich den als "Schnappschuss" bezeichne). Sein Entstehen ist das, was die Kopenhagener Deutung den "Zusammenbruch" der Wellenfunktion nennt.

Beste Grüße,
grtgrt
 

  Beitrag 1915-143
Was wir als reale Wirklichkeit empfinden, ist einfach nur eine Menge von Daten

 
 
Stueps aus 1915-142:
Man kann Vorhersagen einer Theorie experimentell bestätigen, diese Bestätigungen erhärten die Richtigkeit dieser Theorie, beweisen sie jedoch niemals endgültig.

Damit, Stueps, hast du natürlich völlig recht.

Habe das in Bemerkung 1915-140 entsprechend richtig gestellt.


Zitat von Stueps:
Was mir viel wichtiger ist: Was meinst du zu Beitrag 1915-135?

Ich seh, worauf du hinaus willst, und dass man hier aufpassen muss, da nur kleine Ungenauigkeiten in der Formulierung des Sachverhalts ihr jeden Sinn nehmen.

Tatsache ist, dass wir hier gleich 3 solcher Formulierungen vorliegen haben:
  • 1: Jene von dir, die da in ihrer letzten (allzu ungenauen) Form lautet: » Ein Stein ist ein Stein «.
  • 2: Jene von Andreas Mücklich, der sagt » Wirklichkeit = Information über die Wirklichkeit «.
  • 3: Und schließlich jene, in der mich ich bemüht habe, möglichst genau zu sagen, was gemeint ist: » Ein Ding D(Q) existiert in genau dem Ausmaß, in dem — wie indirekt auch immer — Information darüber existiert. «

Aussage 1 ist — da Tautologie — eine Aussage ohne jeden Inhalt.


Aber sag mal ehrlich: Würdest du auch Aussage 3 als ohne jeden Inhalt bezeichnen?

Meiner Meinung nach kann Aussage 3 sogar in zweierlei Weise interpretiert werden:
  • Einmal aus Sicht eines Beobachters, d.h. als: "Jedes Ding D(Q) existiert für uns nur in dem Ausmaß, in dem wir Information darüber haben."
  • Oder absolut gesehen im Sinne von: "Jedes Ding D(Q) existiert nur in dem Ausmaß, in dem über D(Q) informierende Daten existieren (wer welchen Teil davon einzusehen in der Lage ist, wäre dann eine noch ganz andere Frage).
In Mücklichs Buch z.B. finden sich mehrere Argumente, die in meinen Augen nicht schlüssig sind, da sie diese beiden Interpretationsmöglichkeiten der Aussage 3 (und damit auch der Aussage 2 – nur sie findet sich in seinem Buch) nicht auseinander halten.


Bitte mach dir auch bewusst, dass Aussage 3 insofern sehr genau ist, als sie nicht einfach von einem "Ding" spricht, sondern von einem "Ding in Qualität Q".

Wäre das Ding z.B. eine Ziege, so könnte sie in einer Qualität aus Fleisch und Blut existieren (Q1), als Schatten an der Wand einer Höhle (Q2) – denk an Platons Höhlengleichnis – oder vielleicht nur in einem unserer Träume (Q3). Somit ist klar: Im Sinne von Aussage 3 ist eine Ziege nicht einfach eine Ziege.

Genau so wenig kann behauptet werden, Aussage 3 sei ebenso inhaltsleer wie Aussage 1.


Wie schon gesagt: Wir müssen hier sehr sorgfältig formulieren und sehr sorgfältig lesen, um nicht bei einer zyklischen Definition zu landen, die dann eben doch inhaltsleer wäre.

Aussage 3 – und etwas weniger offensichtlich auch Aussage 2 – sind aber keineswegs leer: Schließlich bedeuten sie nichts anderes als


Jedes Ding D(Q) ist eine Menge von Informationen.

Alles, was wir als reale Wirklichkeit empfinden, ist einfach nur eine Menge von Daten.



Beste Grüße,
grtgrt
 

  Beitrag 1916-1
Wo ist der unseren Verstand modellierende Teil der Physik?

 
 
Dieser Diskussionsfaden sei der Frage gewidmet, was alles Teil der Differenzmengen K-U oder U-RZ sein könnte, wo
  • K alles im Kosmos Existierende bezeichnet,
  • U alles, dessen Existenz an die Existenz unseres Universums geknüpft ist, und
  • RZ alles, was in der Raumzeit unseres Universums lebt (aber nicht an all ihren Punkten).

Meine These:
Wenigstens Teile von uns existieren in K-U (woraus folgt, dass diese Menge nicht leer sein kann).


Meine Frage:
Können wir entscheiden, ob auch die Menge U-RZ nicht leer ist?



Zunächst mal: Über Dinge nachzudenken, die nicht Teil unseres Universums sind, ist den Physikern inzwischen zur zweiten Natur geworden. Denken wir da nur an die Stringtheorie, die ja behauptet, dass jede der etwa 10500 Lösungen eines bestimmten Differentialgleichungssystems den Typ eines potentiell existierenden Universums darstellt.

Unter der Annahme, das sei richtig, folgt sofort:
  • Alle der Stringtheorie zugrundeliegenden mathematischen Gesetze (einschließlich aller logischen Gesetze und aller ganzen Zahlen) existieren ohne jede Bindung an unser Universum, sind also nicht Teile der Raumzeit unseres Universums.
  • Und sofort wird klar: Unser Verstand kann die Raumzeit, in der wir leben (unser Universum sogar), nach Belieben verlassen und betreten. Damit aber, so denke ich, können auch die Mechanismen, aus denen er besteht, nicht in vollem Umfang Teil unserer Raumzeit sein.
  • Letztlich sind demnach auch wir selbst nicht in vollem Umfang Teil der Raumzeit unseres Universums.
Was also ist die Physik des Teiles unserer selbst, der nicht Teil unserer Raumzeit ist?

 
Gebhard Greiter (grtgrt)
 

  Beitrag 1916-4
Was Mach sagte ...

 
 
H... aus 1916-2:
 
Lt. Mach z.B. verschwindet mit dem "Universum" alles, mithin ist K\U={∅}
In deiner Menge K steckt ja die implizite Vorstellung, dass es mind. einen Sachverhalt gibt, der nicht an die
Ex. des Univ. gebunden ist. Ausserdem haben wir es mit Prozessen zu tun, d.h. es gibt eine Abhängigkeit
der Konfiguration zum entsprechenden Punkt im RZ-Kontinuum. Das muss unbedingt Beachtung finden.

Geht das Ganze in Richtung Esoterik?
 


Hallo H...,

Mach lebte in einer Zeit, in der man sich noch nicht vorstellen konnte, dass es im Kosmos mehr als nur ein Universum geben könnte.
Es war deswegen für ihn K = U, und deswegen natürlich K-U = ∅.

Die Stringtheoretiker wenigstens sehen das heute das anders, aber wohl nicht nur sie. Denk da mal z.B. an die Theorie der Baby Universes und schau dir die Namen der Physiker an, die darüber schreiben.

Mit Esoterik will ich NICHTS, aber auch GAR NICHTS zu tun haben.

Beste Grüße,
grtgrt

Nebenbei: {∅} ist eine nicht-leere Menge. Ihr einziges Element ist die leere Menge.
 

  Beitrag 1916-5
Prämisse ist: Die (mathematische) Logik im Kosmos ist überall diesselbe.

 
 
E... aus 1916-3:
Grtgrt aus 1916-1:
(...)
Alle der Stringtheorie zugrundeliegenden mathematischen Gesetze (einschließlich aller logischen Gesetze und aller ganzen Zahlen) existieren ohne jede Bindung an unser Universum, sind also nicht Teile der Raumzeit unseres Universums.
 

Wahrscheinlich hast Du Belege oder Indizien für Deine "klaren" Postulate.
Ich bin gespannt.
 

Hi E...,

ich gehe da — wie alle Physiker, die sich Gedanken über Stringtheorie und damit verwandte Themen machen — davon aus, dass der Physik nichts anderes übrigbleibt, als vorauszusetzen, dass Logik und Mathematik im gesamten Kosmos Gültigkeit haben, also z.B. nicht nur lokal in einem bestimmten Universum (wenn es denn mehr als nur eines geben sollte).

Gruß,
grtgrt
 

  Beitrag 1916-11
Mehr als nur 9 Möglichkeiten, dass unser Universum nicht das einzige ist

 
 
Okotombrok aus 1916-8:
 
Grtgrt aus 1916-1:
Zunächst mal: Über Dinge nachzudenken, die nicht Teil unseres Universums sind, ist den Physikern inzwischen zur zweiten Natur geworden.

glaub' ich nicht, nenne doch 'mal welche und zitiere.

... Zwangsläufig muss etwas außerhalb unseres Universums für immer willkürliche Spekulation bleiben und ist somit weder ein Thema für die Physik, noch hat es irgendeine Bedeutung für uns.


Hi Okotombrok,

auf Anhieb fallen mir da mindestens zwei sehr bekannte Physiker ein: Brian Greene und Lisa Randall.

Nur wenig, von dem, über das ihre Bücher berichten, stammt von ihnen selbst. Man könnte also zahlreiche weitere Namen nennen.


In seinem Buch "The hidden Reality" (2011) schreibt Greene gleich im Vorwort:

Zitat von Greene:
 
What we have found has already required sweeping changes to our pictures of the cosmos. Through physical insight and mathematical rigour, guided and confirmed by experimentation and observation, we have established that space, time, matter, and energy engage a behavioral repertoire unlike anything any of us have ever directly witnessed. And now, penetrating analyses of these and related discoveries are leading us to what may be the next upheaval in understanding: the possibility that our universe is not the only universe.
 

Das gesamte Buch — gut 400 Seiten — diskutiert jene Möglichkeiten, und auf Seite 355 werden all diese Theorien — insgesamt 9 — in einer Tabelle nochmals aufgelistet und mit jeweils ein bis zwei Sätzen nochmals kurz angerissen. Diese Tabelle trägt den Titel "Summary of various Versions of Parallel Universes".

Mir ist völlig klar, dass sie alle bisher nur Theorien sind: Theorien aber, über die durchaus renommierte Vertreter der Theoretischen Physik ganz ernsthaft diskutieren — warum also sollten nicht auch wir das tun?


Insbesondere im letzten Drittel ihres Buches "Warped Passages unravelling the Mysteries of the Universe's Hidden Dimensions" (2005) diskutiert Lisa Randall mehrere Möglichkeiten sog. Brane Universes. Sie vergleicht sie mit Quallen, die in einem Ozean schwimmen, der (in diesem Vergleich) den Kosmos repräsentiert. Sie diskutiert aus der Stringtheorie kommende Möglichkeiten für Gravitons unterschiedlicher Typen sowie (mögliche) Elementarteilchen, die an die Qualle gebunden sind (in dem Sinne, dass ihre Wellenfunktion zwar nirgendwo im Kosmos zu Null wird, aber dennoch nur in und in unmittelbarer Nähe der "Qualle" einen Wert hat, der nicht vernachlässigt werden kann).

Für mich ergibt sich aus all dem, dass wir derzeit nicht ausschließen können, dass unser Universum mit einer dieser vielen "Quallen" im kosmischem "Ozean" vergleichbar sein könnte.


Das also ist in etwa der Kontext, in dem ich denke.

Beste Grüße,
grtgrt
 

  Beitrag 1916-16
Erste Überlegungen zur (möglichen) Interaktion unterschiedlicher Universen

 
 
H... aus 1916-14:
Hallo,

Theorie hin oder her: das ist doch einfach ein Bezug auf eine Domäne, mit einem Satz an Grundannahmen und
abgeleiteten Grundzusammenhängen (Gesetze). Die Theorie gewinnt an Vertrauen (also nicht: "wir beweisen die
Richtigkeit der Theorie"), wenn deren Vorhersagen mit der menschlichen Beobachtung (wir haben nur die) übereinstimmen. Sie gilt als widerlegt, wenn mind. eine Beobachtung der Theorie widerspricht.
That's it!

Für den interessierten Leser (aber nur für den, es steht ein bisschen Mathematik drin) mal eine Kostprobe zum Thema Viel-Universen:
multi-universe

Gruss
H...

Hi, H...!

Ich stimme dir zu, was die Verifizierung bzw. Falsifizierung von Theorien angeht. Aber ebenso richtig ist sicherlich, dass eine neue Theorie nicht wilde Spekulation sein sollte, wenn es eben Grundlagen gibt, auf denen andere Theorien bereits erfolgreich ihre Berechtigung erworben haben. Jedes Experiment ist die Verifizierung bzgl. einer Annahme die Theorie betreffend. Es sei jedem überlassen, über den Kosmos sich seine Gedanken zu machen und zu den abstrusesten Folgerungen zu gelangen. Eine Theorie, die ernst genommen werden will, wird aber die Grundlagen z. B. der ART bzw. der Quantenmechanik akzeptieren. Wenn Karl Popper also die Ansicht vertritt, dass es möglich und nützlich sei, eine Theorie völlig frei zu entwickeln, ist das nur eine Seite seiner Thesen. Die andere ist, dass sich Theorien "evolutionär" entwickeln, sie beruhen stets auf Vorangegangenem, soweit vorhanden.

Was nun die Multi-Versen angeht, so widerspricht das sicher nicht der Freiheit, die Popper für das Theoretisieren fordert – aber ich vermisse ganz eindeutig die Möglichkeit, diese Hypothese in irgendeiner Hinsicht auch nur durch ein Experiment zu bestätigen oder zu widerlegen, ja, sie entzieht sich nicht nur der Begutachtung, sondern einer grundsätzlichen deiner Forderungen: Wir können nichts beobachten, was mit Multi-Versen zu tun haben könnte. Nicht alles, was gedacht werden kann, muss auch wirklich sein können, und man muss Popper ja nicht in allem folgen.
 

  Beitrag 1916-20
Existieren z.B. Elektronen teilweise auch außerhalb der 4-dimensionalen Raumzeit?

 
 
Zurück zum eigentlichen Thema:

Existieren wir teilweise außerhalb der Raumzeit unseres Universums?


In einigen atomaren Zuständen (etwa im Zustand 2p des Wasserstoffatoms) zerlegt die Wellenfunktion den Raum in Regionen, die durch unendliche Flächen (sog. Knotenflächen) separiert sind.

Diese Flächen stellen Lösungen der Schrödunger-Gleichung dar: Flächen also, auf denen die Wahrscheinlichkeit, dass ein Elektron dort gefunden wird, Null ist. Solche Flächen also kann ein Elektron weder berühren noch durchdringen — es ist gezwungen, sie zu meiden.

Trotzdem kann ein Elektrom mal in der einen dieser Regionen gefunden werden und mal in der anderen. Lothar Schäfer schreibt:


Wie es von einem Raumteil in den anderen kommt, dazu sollte man schweigen, weil man es nicht beobachten kann.

Geometrisch jedenfalls, führt jeder Weg aus einem dieser Raumteile in einen anderen durch eine Knotenfläche.


Schäfer schließt daraus, dass diese Tatsache vermuten lasse, dass "die Wirklichkeit eines Quantenobjekts zwischen zwei Beobachtungen keine normale beobachtbare Wirklichkeit ist". Sie scheint nur über nicht-lokale Phänome erklärbar.

Er schreibt weiter: "Die Anormalität kommt dadurch zum Ausdruck, dass sich die Elektronen nicht an einen gewöhnliche Zwang von Raum oder Zeit zu halten scheinen, so als seien sie in der Lage, einem solchen Zwang in einem Zustand außerhalb der Raumzeit zu umgehen."

Die Existenz von Zuständen außerhalb der Raumzeit ist bislang nicht beweisbar, wurde aber dennoch auch von einigen anderen Physikern schon ins Gespräch gebracht, so etwa von
  • Stapp H.P. in: Are Superluminal Connections Necessary? (Nuovo Cimento 40B: 191-199, 1977)
  • Kafatos und Nadeau in: The Consious Universe (Springer Verlag, N.Y., 1990)
  • Goswami A., Reed und Goswami M. in: The Self-Aware Universe (Penguin Putman Inc., N.Y. 1993)
  • Nesteruk A.V. in: Is a Wave Function Collapse a Real Event in Physical Space and Time? (erschienen in: Duffy & Wegener, Hrsg.: Recent Advances in Relativity Theory 2: Material Interpretations, Hodronic Press, pages 169-170, 2000)
 

  Beitrag 1917-11
Die Grundlagen der materiellen Welt sind nicht-materiell.

 
 
Quante aus 1917-10:
Zu lesen steht geschrieben:
» Die Grundlagen der materiellen Welt sind nicht-materiell. «

Hi Quante,

diese Aussage zu erklären habe ich den Beitrag 1924-1 geschrieben.


Quante aus 1917-10:
... frage ich den Philosophen Grtgrt, was für ihn denn Grundlagen, im ganz konkreten, sind?

Grundlagen sind für mich die Naturgesetze — von denen die meisten, wenn nicht sogar alle, in ihrer ursprünglichsten Form wohl sämtlich auf mathematische Gesetzmäßigkeiten zurückführbar sind. Auf die Frage, woher die kommen, fällt auch mir nichts mehr ein.

Kurz also: Mathematische Gesetzmäßigkeiten sind Naturgesetz (das ist für mich offensichtlich) — und womöglich ist nichts sonst Naturgesetz.

Gruß,
grtgrt
 

  Beitrag 1917-12
Fast alle Dinge sind aufgebaut aus Dingen, die weit weniger konkret sind als sie selbst.

 
 
Quante aus 1917-10:
Die 2. "These" wird dann noch unheimlicher:

» Fast alle Dinge sind aufgebaut aus Dingen, die weit weniger konkret sind als sie selbst. «


Diese Formulierung muss man sich erst einmal auf der Zunge zergehen lassen, im Kopf angekommen wird’s dann nur noch ein grauer Brei, aber es ist in seiner Konsequenz, die Weiterführung ... der Grundlagenbedeutung, und, so vermute ich, auf einer höheren Ebene?

Hi Quante,

das ist folgendermaßen gemeint (und auf eine "höhere Ebene" muss man sich deswegen keineswegs bemühen):

Alle Dinge, die wir als Materie sehen, haben aus unserer Sicht eine recht handfeste Existenz (denk mal z.B. an Steine, dein Haus, dein Auto, dich selbst).
Dennoch bestehen sie alle aus Elementarteilchen. Die aber sind WEIT WENIGER KONKRET.

Nachdem du mir das, wie ich mal annehme, nicht glauben wirst, hier zwei Zitate eines Experten:


Zitat von Heisenberg, Seite 101:
Wenn man eine genaue Beschreibung des Elementarteilchens geben will — und hier liegt die Betonung auf "genau" — so ist das einzige, was als Beschreibung niedergeschrieben werden kann, die Wahrscheinlichkeitsfunktion. Aber daraus erkennt man, dass nicht einmal die Eigenschaft des "Seins" ... dem Elementarteilchen ohne Einschränkung zukommt. Es ist eine Möglichkeit oder eine Tendenz zum Sein.

Zitat von Heisenberg, Seite 162:
In den Experimenten über Atomvorgänge haben wir es mit Dingen und Tatsachen zu tun, mit Erscheinungen, die ebenso wirklich sind wie irgendwelche Erscheinungen im täglichen Leben. Aber die Atome und Elementarteilchen sich nicht ebenso wirklich. Sie bilden eher eine Welt von Tendenzen oder Möglichkeiten als eine von Dingen und Tatsachen.


Die beiden Zitate sind genommen aus Heisenbergs Buch "Physics and Philosophy" (1958). Ob die Seitenangaben sich auf die englische oder deutsche Ausgabe beziehen, weiß ich nicht.

Gruß,
grtgrt
 

  Beitrag 1923-1
Gibt es physikalisches Modelle, die beliebig weit ins Ultra-mikroskopische hinein Sinn machen?

 
 

Warum wir stets nur eine makroskopische Sicht auf die Natur haben können


Wo man die Energie eines Quantums (z.B. eines Elektrons oder eines Photons) zu messen sucht, muss man — so wird mir glaubhaft versichert — Energie und Zeit als zueinander konjugierte Größen im Sinne von Heisenbergs Unschärferelation betrachten. Damit gilt die Ungleichung

ΔE Δth/4π


wo h das Plancksche Wirkungsquantum bezeichnet (eine positive Zahl, die man als Naturkonstante kennt).

Nach gängiger Interpretation beschreibt diese Ungleichung nun aber keineswegs nur eine Unschärfe der Beobachtung, sondern vielmehr auch eine der Natur selbst innewohnende Unschärfe. Wer nun Δt zunehmend kleiner wählt, erkennt,

dass mindestens eine der folgenden Thesen wahr sein muss:

  • These 1: Es gibt kleinste Zeitspannen (die Zeit ist also gequantelt).
  • These 2: In jeder noch so kleinen Zeitspanne wächst die Energie des Quantums unendlich oft über jede nur denkbare Grenze.
  • These 3: In jeder noch so kleinen Zeitspanne hört das Quantum unendlich oft auf zu existieren.


Welche auch immer wahr sein sollte, man kann auf jeden Fall mit Sicherheit behaupten:


Es gibt eine (ultra-) mikroskopische Sicht auf die Natur, in der mindestens eine der Größen Zeit bzw. Energie keinen Sinn mehr macht.

Sie macht also nur Sinn aus makroskopischer Sicht.


Das aber lässt mich fragen:

Gibt es denn überhaupt ein physikalisches Modell, welches beliebig weit ins Ultra-mikroskopische hinein Sinn macht?



Letztlich führt das auch auf die Frage, in welchem Ausmaß Elementarteilchen denn überhaupt als "existierend" bezeichnet werden können.

Heisenberg selbst noch hat das klar erkannt, denn auf Seite 101 seines Buches "Physics and Philosophie" (1958) liest man:

Zitat von Heisenberg:
 
Wenn man eine genaue Beschreibung des Elementarteilchens geben will ... erkennt man, dass nicht einmal die Eigenschaft des "Seins"
— wenn man hier überhaupt von einer Eigenschaft reden will —
dem Elementarteilchen ohne Einschränkung zukommt.

 


Gebhard Greiter (grtgrt)
 

 

  Beitrag 1923-35
Die Wellenfunktion bricht nicht zusammen — sie repräsentiert Informationsstand

 
 
C... aus 1923-34:
 
Die offensichtliche Subjektivität dieser Wertung legt es m.E. nahe, den "Zusammenbruch" der Wellenfunktion (falls es diesen überhaupt gibt) nicht einem Zeitpunkt (Vergangenheit, Gegenwart, Zukunft) zuzuordnen, sondern ihn vielmehr an die Information zu koppeln, die ein Beobachter erhält, wenn er das jeweilige Ereignis beobachtet.
 

Hi C...,

mir scheint, deine Vorstellung den "Zusammenbruch" betreffend ist genau die richtige.

Alles, was ich bisher über Quanten-Experimente gelesen habe, deutet darauf hin, dass die Wellenfunktion nicht zusammenbricht, sondern dass sie sich automatisch jeder neuen Informationslage anpasst (sie ist sozusagen "Spiegelbild" der jeweils vorhandenen Informationslage).

Als "Zusammenbruch" erscheint das nur dem, der seine Betrachtung auf nur eine gemessene Größe (sprich: auf ein zu messendes Attribut) eines Quantums beschränkt. Schon wer mindestens zwei Quanten gleichzeitig betrachtet — wie man das etwa in einem von Leonard Mandel durchgeführten Experiment tut — sieht, dass z.B. das Unterbrechen eines der dort vorhandenen 4 Lichtwege zu mehr Information führt und sofort zu einer entsprechend modifizierten Wellenfunktion an zwei Detektoren, die Teile eines im Experiment in insgesamt 4 Teile zerlegten Photonstrahls aufzufangen da sind.

Dieses Experiment beschreibt Lothar Schäfer in seinem Buch in Anhang 17. Im WWW findet sich auch eine Beschreibung, die ist aber etwas gekürzt und daher weniger klar. (So auf die Schnelle habe ich jene Seite eben jetzt auch nicht wiedergefunden).

Beste Grüße,
grtgrt


PS: Eben erfahre ich, dass eine Gruppe um Gerhard Rempe (1998) folgendes Experiment in die Tat umgesetzt hat:

Rubidiumatome wurden durch einen "Doppelspalt" geschickt, der aus purem Licht bestand (also nicht aus Materie). Genauer:

In einer komplizierten Anordnung gelang es den Physikern, den Strahl aus Rubidium-Atomen an stehenden Lichtwellen so aufzuspalten, dass 4 Teilstrahlen entstanden:
Je zwei konnten sich gegenseitig überlagern und so Interferenz bilden.

Mit einem ganz besonderen Trick versuchten die Forscher nun, den Rubidium-Atomen die Information zu entlocken, auf welchem Weg jedes einzelne die Anordnung durchflogen hatte. Dazu muss man wissen: Rubidium-Atome besitzen in ihrer äußersten Schale ein Elektron, dessen Spin sich nach oben oder nach unten einstellen lässt. (Das Atom wird dadurch nicht verändert.)

Die Physiker richteten es so ein, dass das Elektron der Atome, die durch Spalt 1 kamen in eine, und das der Atome, die durch Spalt 2 kamen in die andere Richtung eingestellt wurde. An dieser Einstellung lies sich dann feststellen, durch welchen Spalt das Atom gekommen war.

Wie in ähnlichen Doppelspalt-Experimenten ergab sich auch hier:

Sobald diese Markierung gesetzt, also Weg-Information vorlag, verschwand die Interferenz zwischen den Atomen.


Das Besondere hier war, dass dieses Verschwinden allmählich herbeigeführt werden konnte:
Je mehr Atome man wie beschrieben normiert losschickte, desto mehr verschmierten sich die hellen und dunklen Streifen des Interferenzbildes:

Atome, deren Weg festgestellbar war,
bekamen hinter dem Spalt ganz offensichtlich andere Wellenfunktion als jene, deren Weg nicht feststellbar war.

Und das, obwohl sie sich von den anderen in rein GAR NICHTS unterschieden (!)


 

  Beitrag 1927-28
Zur genauen Definition der Begriffe » Geist « und » Materie «

 
 
Gregor Lämmer aus 1927-26:
... was bringt dann Materie hervor und plant diese?

Wer oder was Materie entstehen lässt, scheint mir überzeugend erklärt in Beitrag 1924-1.

Wer sie plant, ist weit schwieriger zu beantworten. Soweit man (als Physiker) sehen kann, sind das Naturgesetze, die in einer nur auf Geist basierenden zusätzlichen Dimension unserer Welt ihren Ursprung haben.


PS: Ich habe es bisher strikt vermieden von "Geist" zu sprechen und stattdessen den Ausdruck "nur gedanklich existierend" verwendet.

Da ich nun aber festgestellt habe, dass der Begriff "gedanklich" fast alle meine Geschrächspartner an aus Menschen kommende Gedanken erinnert, kann ich diesen Ausdruck nicht weiter verwenden.

Unter » Geist (in der Natur) « verstehe ich etwas,
  • auf dessen Existenz es zahlreiche indirekte Hinweise gibt,
  • das zweifelsfrei nachzuweisen der Mensch bisher aber keinerlei Geräte entwickelt hat.

John Archibald Wheeler war in der zweiten Hälfte seines Lebens mehr und mehr der Meinung, die Wurzel der Natur könnten rein nur aus Information bestehen.
Sollte er damit recht haben, wäre Geist (in meinem Sinne) der Mechanismus, der eben diese Information verarbeitet.

grtgrt
 

  Beitrag 1927-33
Wie realisiert die Natur erst Gedankliches, und sogar Geist?

 
 
Henry aus 1927-31:
Grtgrt aus 1927-28:

PS: Ich habe es bisher strikt vermieden von "Geist" zu sprechen und stattdessen den Ausdruck "nur gedanklich existierend" verwendet.

Da ich nun aber festgestellt habe, dass der Begriff "gedanklich" fast alle meine Geschrächspartner an aus Menschen kommende Gedanken erinnert, kann ich diesen Ausdruck nicht weiter verwenden.

Dein gesamte Fragestellung war nach meiner Ansicht nur vorgeblich ergenbnisoffen, du wolltest von Anfang an genau diese auch bestätigt sehen.

Nein, Henry,

mein Grund war, dass ich unter "Geist" eigentlich weit mehr verstehe als nur "Gedanken" oder das, was sie erzeugt und verwaltet.

Können wir denn tatsächlich sicher sein, dass Gedanken nicht auch von — was auch immer — gesteuert sein können (etwa so, wie man einen Dia-Projektor steuert und mit den Bildern füttert, die er an die Wand werfen soll)?

Gruß, grtgrt
 

  Beitrag 1927-47
Was steckt hinter nicht-lokalen Phänomenen?

 
 
C... aus 1927-43:
Hallo Grtgrt, Henry, Bauhof,

Grtgrt aus 1927-37:
Warum eigentlich denkt niemand an die Möglichkeit, dass wir Teil seiner selbst sein könnten?

Ich denke nicht nur, sondern glaube schon ziemlich lange an diese Möglichkeit.

Eine Analogie ist für mich die Eltern/ Kind-Beziehung. Betrachtet man die genetische Verwandschaft (vgl. auch Informationsbegriff in Beitrag 1948-1 unter Nr. 6) und die erzieherische Beeinflussung durch die Eltern, so lässt sich m.E. durchaus die Auffassung vertreten, ein Kind sei Teil seiner Eltern. Dennoch ist das Kind ein (in gewissen Grenzen) eigenständiges, also freies, Individuum.

Hi C...,

dein Argument finde ich überzeugend.

Ich selbst habe die Sache zunächst aber weniger abstrakt gesehen. Mein Gedanke ging eher in folgende Richtung:

Die Tatsache, dass es nicht-lokale Phänomene gibt (Qantenverschränkung ist vielleicht nur ein erstes Beispiel), legt in meinen Augen die Vermutung nahe, dass die Gesamtheit dessen, was wir beobachten können, nur einen Teil der Natur darstellt — dass da also vielleicht noch ein zweiter Teil ist, den wir bisher noch nicht mal seinem Wesen nach kennen.

Eine erste Vermutung war ja, es könne da "verborgene Variable" geben. Dass sich diese Vermutung als falsch erwies, schließt aber in meinen Augen nicht aus, dass da doch irgendwas sein könnte: Etwas, das uns verborgen ist ebenso wie 10/11 jeden Eisbergs unter der Wasseroberfläche existieren und daher zunächst unsichtbar sind. Das sichtbare 1/11 so eines Eisbergs — sein über Wasser existierender Teil also — kann gut die Form von Spitzen haben, die demjenigen, der nicht unter die Wasser­oberfläche sehen kann, anmuten wie eine ganze Reihe deutlich kleinerer, einzelner Eisberge.

Ebenso könnte alle Lebewesen in unserem Kosmos etwas verbinden, das — wenn wir es registrieren könnten — uns klar machen würde, dass alles, was Bewusstsein hat, Teil eines einzigen großen Wesens ist.

Mein "könnte ..." steht hier für "rein logisch gedacht, kann man nicht ganz ausschließen, dass ...".

Beste Grüße,
grtgrt
 

  Beitrag 1924-28
Was ist eine Platonische Idee?

 
 

Platonische Idee = etwas (als Idee) absolut genau Definiertes

Bitte beachten: Messungen können niemals absolut genau in diesem Sinne sein.


 

  Beitrag 1924-33
Nicht jedes Objekt ist ein physikalisches (oder gar materielles)

 
 
Grtgrt aus 1924-32:
U...bus aus 1924-31:
 
Was gibt es denn deiner Meinung nach für Elemente, die nicht physischer Natur wären?
Alles, was sich innerhalb des Universums befindet, ist physisches Objekt, oder glaubst du an Götter?

Nein, U...bus,

an Götter habe ich da nicht gedacht, wohl aber an Platonische Ideen (wie etwa die in Beitrag 1924-30 explizit definierten Mengen).

Gruß, grtgrt
 

Gebhard,

die Richtigkeit deiner These fällt und steht mit der Richtigkeit der These über die Existenz der Idee im platonischen Sinne. Den Beweis für die Richtigkeit dieser Behauptung hast du aber noch nicht erbracht (so wenig wie im Übrigen Plato selbst und seine große Anhängerschar).
 

  Beitrag 1924-2
Das Weltbild, das uns die moderne Physik nahelegt

 
 

Das Weltbild, das die moderne Physik uns nahelegt


Die zusätzliche — rein geistige — Dimension unser Welt, von der z.B. in Beitrag 1924-1 spricht, werde ich in Zukunft die Platonische nennen.

Sie ist die Heimat aller Naturgesetze. Jene existieren auf nur logischer Ebene, führen aber dennoch zum Entstehen des einen oder anderen Universums. Sie formen seine Raumzeit und erschaffen, prägen und steuern alles, was darin existiert (sei es Objekt oder Prozess).

Es gibt keinen Grund anzunehmen, dass unser Universum das einzige oder gar das einzig mögliche sei.

grtgrt
 

  Beitrag 1924-10
Zur Terminologie: Kosmos, Universum, unsere Welt

 
 
Okotombrok in 1924-6:
 
Wiki macht hier keine Unterscheidung zwischen Kosmos und Universum
Zitat von Wiki:
Als Universum (von lateinisch universus "gesamt", von unus und versus "in eins gekehrt") wird allgemein die Gesamtheit aller Dinge bezeichnet. Im Speziellen meint man damit den Weltraum (veraltet auch Weltenraum), auch Weltall oder Kosmos (von griechisch κόσμος kósmos "(Welt-)Ordnung", "Schmuck", "Anstand" das Gegenstück zum Chaos) und bezeichnet die Welt bzw. das Weltall sowohl als das sichtbare Universum, als auch als geordnetes, harmonisches Ganzes.
 

Hi Okotombrok,

die Definition in Wikipedia scheint mir veraltet. Sie war übrigens auch meine — allerdings nur, bis ich dann Brian Greenes Bücher gelesen hatte.

Bei Greene findet sich zwar keine explizite Definition für den Begriff » Kosmos «. Dennoch fällt mir auf, dass er die Worte Kosmos bzw. Universum genau dort benutzt, wo ich sie (meiner Definition entsprechend) auch wählen würde.

Ansonsten: Schon in Beitrag 1916-11 habe ich beschrieben, warum und in welcher Weise,

die Begriffe Kosmos und Universum einer Unterscheidung bedürfen:


... wir können derzeit nicht ausschließen,
dass unser Universum mit einer dieser vielen "Quallen" im kosmischem "Ozean" vergleichbar sein könnte.


Gruß,
grtgrt

PS: Wo ich hin und wieder von "unserer Welt" spreche, will ich offen lassen, ob dort unser Universum oder der Kosmos insgesamt gemeint ist bzw. gemeint sein sollte.

 

  Beitrag 1924-11
Ist das sich selbst bewusste Bewusstsein des (oder der) Menschen Teil eines kosmischen Bewusstseins?

 
 
E... aus 1924-8:
 
Grtgrt aus 1924-1:
Nüchterner und genauer ausgedrückt:
 
Der physische Teil unserer Welt wird geschaffen, geformt, und regiert durch nur gedanklich Existierendes.

Gebhard Greiter (grtgrt),
der hiervon überzeugt wurde durch Argumente von Lothar Schäfer

Was selbst bei der genaueren Formulierung immer noch fehlt, ist ein Hinweis darauf wer oder was da denkt.

Gedanklich existierendes setzt voraus das gedacht wird. In diesem Fall sollte man wissen wer oder was gedacht hat als z. B. unser heimisches Sonnensystem oder noch davor unser Universum entstand. Auch Schäfers Lothar macht dazu leider keine Angaben. Vielleicht hast Du einen Tip, der auch mich in die Lage versetzt, Lothar Schäfer zu folgen.
 

Hi E...,

hiermit sprichst du eine ganz besonders wichtige Frage an. Leider habe ich noch nicht einmal die Ahnung einer Antwort darauf.

Lothar Schäfer scheint " das sich selbst bewusste Bewusstsein " des Menschen wohl als so eine Art Zipfel eines versteckten " kosmischen Bewusstseins " zu sehen. Er verkennt aber keineswegs, dass das verdammt schwierige Fragen, unseren " freien Willen " betreffend, aufwirft. Hier scheint auch er (indirekt) zu sagen: "Leider habe ich noch nicht einmal die Ahnung einer Antwort darauf."

Beste Grüße,
grtgrt
 

  Beitrag 1924-19
Sich Platonische Ideen vor Augen zu führen bedeutet nicht, sie zu erschaffen

 
 
U...bus aus 1924-14:
 
"Gedanklich Existierendes" benötigt einen Denkenden, der es denkt. Also ist doch der physische Teil der Welt Voraussetzung für "gedanklich Existierendes" und nicht umgekehrt. Ohne Denkerbse kein Denken.

... ohne Denken keine Existenz, das versuche ich Physikern immer klarzuachen.
 

Hi U...bus,

was du hier sagst ist falsch, denn:
  • Erstens: Nicht alles, was nur gedanklich existiert, ist eine Platonische Idee. Bilder etwa, die wir träumen, oder Geschichten, die wir im Traum erzählt bekommen, existieren nur gedanklich, sind aber dennoch keine Platonischen Ideen.
  • Zweitens: Nirgendwo habe ich behauptet, dass erst unser Denken Platonische Ideen erschaffen würde. Ganz im Gegenteil: Die einzigen Beispiele Platonischer Ideen, von denen ich gesprochen habe, waren Folge mathematischer Gesetze, und die — darauf habe ich nun schon sehr oft hingewiesen — sind Naturgesetz.
  • Drittens: Das einzige Werkzeug, eine Platonische Idee in aller Klarheit wahrzunehmen, ist — für Menschen jedenfalls — der menschliche Verstand. Alle anderen Werkzeuge, die Werkzeuge der Experimentalphysik etwa, zeigen uns nur schattenähnliche Projektionen davon.


Meine Aussage war:

Der physische Teil unserer Welt wird geschaffen, geformt, und regiert durch nur gedanklich Existierendes.


Das Missverständnis, das ich jetzt in deinem Beitrag vorfinde, zeigt mir, dass ich mich genauer so hätte ausdrücken sollen:


Der physische Teil unserer Welt wird geschaffen, geformt, und regiert durch Naturgesetze, die Platonische Ideen sind.



Nochmals also: Die erste dieser Aussagen ist Folge der zweiten, genaueren Aussage. Es ist aber keineswegs so, dass erst unser Denken jene als Naturgesetz auf­tretenden Platonischen Ideen erschafft. Sie existieren völlig unabhängig von uns.

Beste Grüße,
grtgrt

PS: Natürlich gilt auch: Nicht alles, was jemand denkt, wurde durch ihn selbst erschaffen (also zum ersten Mal gedacht).
 

  Beitrag 1924-27
Warum Nicht-Stoffliches wohl doch existiert

 
 
Stueps aus 1924-23:
 
Die Dinge in unserer Welt gehorchen einer Ordnung, die sich empirisch untersuchen lässt. Daniel Kehlmann sagte in einem seiner Romane: "Und auf dem Grunde dieser Welt liegen die Zahlen." Dinge können erst in unserer Welt stabil existieren und wechselwirken, wenn sie dieser, na ich will mal sagen, "Überordnung" gehorchen.

Dies ist für mich ein Indiz, dass etwas Nichtstoffliches real existiert, dem Materie und Energie gehorchen. Wir kleiden diese Ordnung in Sprache, um sie begreifen und beschreiben zu können. Diese Sprache heißt bei uns Mathematik. Und ich verstehe Mathematik eben als das "Ordungs-Ding", nicht als menschliche Sprache allein. Die Beschränkung: "Mathematik ist ausschließlich etwas von Menschen erdachtes." ist mir persönlich wesentlich zu wenig. Ich erhebe keinesfalls Anspruch auf Richtigkeit dieser Meinung, das möchte ich betonen!
 

Ich sehe es ebenso,

grtgrt
 

  Beitrag 1929-19
Das wichtigste aller Naturgesetze: Es muss Raum bleiben für absoluten Zufall und freien Willen.

 
 
Stueps aus 1964-28:
 
Grtgrt aus 1964-21:
Es scheint mir eher so zu sein, dass
· das wichtigste aller Naturgesetze darin besteht, die Natur zu veranlassen, eben NICHT alles zu regeln.

Hallo Gebhard, hier drückst du dich m.E. sehr ungenau aus, was mich zum Raten zwingt, und einer Diskussion nicht unbedingt förderlich ist:

Welches Naturgesetz soll das sein? Die Heisenbergsche Unschärfe?


Hallo Stueps,

mit den Freiraum, auf dessen Existenz ich in Beitrag 1964-21 aufmerksam machen wollte, habe ich kein konkretes Naturgesetz gemeint, sondern vielmehr die Tatsache, dass offenbar die Summe ALLER Naturgesetze gar nicht beabsichtigt wirklich ALLES zu regeln.

Diese Tatsache also ist es, was ich dann "das wichtigste aller Naturgesetze" genannt habe.

Die Freiheitsgrade, von denen ich dort spreche (und die ich jetzt — weil sich der Begriff "Freiheitsgrad" als schon belegt herausgestellt hat — als "von den Naturgesetzen gelassenen Freiraum" bezeichne), betreffen dann also schlicht und einfach alles, was durch Zufall oder den "freien Willen" von was auch immer geregelt sein kann.

Als was solche "freier Wille" sich dann letztendlich herausstellen könnte, weiß ich natürlich auch nicht.

Beste Grüße,
grtgrt


PS: Wo ich mich nach deinem Dafürhalten zu ungenau ausdrücke, ist das keine Absicht, sondern vor allem Folge der Tatsache, dass ich es selber nicht so genau weiß. Schließlich und endlich ist sehr viel von dem, was ich hier schreibe, ja einfach nur lautes Nachdenken (!).

 

  Beitrag 1929-26
Evolution = Zufall + Selektion

 
 
Stueps aus 1929-14:
 
Grtgrt aus 1929-12:
Freiraum dieser Art existiert doch mindestens dort, wo absoluter Zufall regiert — etwa beim Kollabieren der Wellenfunktion, wo die Naturgesetze NICHT festlegen, in welch realen Zustand genau das betroffene Quantenobjekt denn nun zu kommen hat.

Interessant ist, dass du dann an anderer Stelle schon mein Argument, dass dieser absolute Zufall weder durch belebte noch durch unbelebte Materie nutzbar ist, ignoriert hast. Und an anderer Stelle wieder damit argumentierst.

Hi Stueps,

deine Meinung, dass der beim Zusammenbruch der Wellenfunktion waltende Zufall weder durch belebte noch durch unbelebte Materie "nutzbar" ist, teile ich nicht.

Begründung:

Evolution = Zufall + Selektion



Das Stichwort "Selektion" steht hier für die Tatsache, dass, wo immer ein Prozess zu einem nur durch Zufall bestimmten Ergebnis führt, dieses Ergebnis mehr oder weniger stabil sein kann als andere Ergebnisse, die statt seiner hätten eintreten können. Wählt der Zufall nun also ein ganz besonders stabiles Ergebnis aus, so kann das durchaus Konsequenzen haben, die auf Basis eines weniger stabilen, aber ebenso wahrscheinlichen Ergebnisses weniger wahrscheinlich oder sogar undenkbar gewesen wären.

Diese Überlegung gilt für sämtliche Prozesse mit zufälligem Ergebnis, also auch für Elementarereignisse.

Mit anderen Worten: Ich könnte deine Meinung nur dann als zutreffend einstufen, wenn wir wüssten, dass jeder Zusammenbruch einer Wellenfunktion nur zu Zuständen führen kann, die sämtlich exakt gleich stabile Nachfolgekonfiguration (des betroffenen Quantensystems) schaffen würden, wenn sie denn einträten.

Nichts spricht dafür, dass diese Bedingung tatsächlich für jedes nur denkbare Elementarereignis erfüllt ist.

Gruß, grtgrt
 

  Beitrag 1933-16
Warum die Zeit in der Physik ein unentbehrliches Konzept darstellt

 
 
Gregor Lämmer aus 1933-1:
wasistzeit.de fragt nach dem Charakter der Zeit.

Es ist aber auch interessant, zu fragen, warum es Zeit gibt.


Hi Gregor,

Einen Grund dafür, dass die Physik ohne den Begriff "Zeit" wohl gar nicht auskommen kann, sehe ich darin, dass die Zeit — der Zeitgraph also (siehe meine Theorie vom Zeitgraphen) — in nur EINER Richtung durchlaufen werden kann: hinein in die Zukunft.

Das ist so, da ihn rückwärts zu durchlaufen bedeuten würde, Elementarereignisse in eindeutig definierter Reihenfolge, deterministisch also, rückgängig zu machen.

Auf Quantenebene aber funktioniert rein gar nichts deterministisch.


Gruß, grtgrt
 

  Beitrag 1933-18
Warum ich nicht an ein » Blockuniversum « glaube

 
 
Hi Henry,

danke für den Hinweis auf den Begriff » Blockuniversum « (ein Stichwort, das ich noch nicht kannte).

Dennoch ändert auch die Tatsache, dass Einstein sich schließlich zum Inhalt dieses Begriffs bekannte, nichts an meinem Urteil.


Wenn ich recht habe (was wir natürlich nicht wissen), würde daraus automatisch folgen, dass auch die SCHEIDUNG zwischen Vergangenheit und Zukunft keine Illusion sein kann: Das einfach deswegen, weil dann ja ein Weg zurück in die Vergangenheit ganz andere Qualität hätte als ein Weg nach vorne in die Zukunft.

Auf jeden Fall gilt: Wer an das "Blockuniversum" glaubt — daran also, dass unsere Zukunft schon ebenso real existiert wie unsere Vergangenheit — käme zwangsläufig zum Schluss, dass der Mensch, auf seinem Weg hinein in die Zukunft KEINEN freien Willen hat. Das zu glauben, lehne ich ab.


Ich glaube, dass die Gegenwart ein 3-dimensionaler Teilraum der 4-dimensionalen Raumzeit ist,
der sich durch sie hindurch in Richtung Zukunft bewegt wie die Grenzlinie des Schattens eines Baumes,
der (sozusagen als "Vergangenheit") immer größer wird, wenn die Sonne hinter dem Baum zu versinken beginnt.


Natürlich ist dieser "die Gegenwart" genannte 3-dimensionale, sich bewegende Teilraum nicht glatt und durch einen einzigen Zeitwert gegeben (schließlich und endlich ist ja auch die 4-dimensionale Raumzeit KEIN 4-dimensionaler Vektorraum, sondern nur eine 4-dimensionale differenzierbare Mannigfaltigkeit).


Beste Grüße,
grtgrt

PS: Aus welcher (Online-) Quelle stammt dein Wissen übers Blockuniversum? Ist es diese (oder gibt es da noch bessere)?
 

  Beitrag 1938-26
Zwei noch völlig offene Fragen

 
 
Gregor Lämmer aus 1938-25:
... ich nehme an, dass es eine kleinste Dauer gibt, der man die Größe 1 zuschreiben kann. Dies hätte dann zur Folge, dass alle Dauern, die wir im Mesokosmos erleben, eine mehrfache Größenordnung dieser kleinsten Dauer darstellen.

Hi Gregor,

ob es diese kleinste Dauer (oder ob es unteilbare Zeitquanten) gibt, sind zwei noch völlig offene Fragen.

Siehe dazu Wikipedia übers Chronon, wo steht (Zitat):

In der aktuellen Quantenphysik wird die Existenz des Chronon sehr kontrovers diskutiert.


Gruß, grtgrt
 

  Beitrag 1938-28
Auch die Frage nach dem, was Zeit eigentlich ist, scheint völlig offen

 
 
Horst aus 1938-23:
Hallo Henry

Und mit welchem zeitlichen Begriff – der nicht von einer Bewegung abgeleitet wurde – erklärst du es?

Ich weiß zwar nicht wessen "vorher" und "nachher" du meinst, aber um von "vorher" und "nachher" sprechen zu können, bedarf es doch eines Objektes, da sich "vorher" an einem andern Ort im Raum befunden hat als "nachher".

Das heißt doch, man kann diese beiden Begriffe auch erklären, ohne irgendeinen zeitlichen Begriff zu verwenden, nämlich indem man den räumlichen Abstand bestimmt.

So, nun wäre es schon nett von dir auch deine Aussage zu erklären:

Zitat:
für mich bedeutet "etwas dauert", dass Zeit vergeht.

Insbesondere, wie du (und nicht Newton) feststellst, dass und wie "Zeit" vergeht!

Gruß Horst

Horst,

mir war nicht ganz ernsthaft zu Mute, weil das Thema ja nun wirklich nicht zum ersten Male durchkauen. Ich will mich deshalb auch nur noch einmal dazu äußern.

Also, mit deiner Ansicht, dass Zeit nicht real existiert, stehst du ganz sicher nicht allein, und auch gestandene Physiker sehen das so. Man kann es so sehen, es ist dann unser Verstand, der die Dinge im Raum und in der Zeit ordnet, weil wir sonst nur strukturloses Chaos vorfinden würden. Bliebe dann aber die Frage, wie er – unser Verstand – das fertig bringt. Falls es so wäre, dass Zeit nicht existiert, müsste alles jetzt und hier gemeinsam existieren.

Da liegt nämlich dein gedanklicher Fehler. Auch Physiker – und sonstige nette Leute -, die Zeit als vom Verstand geschaffen ansehen, bezeichnen das, was du rein der Bewegung zuschreibst, als zeitliche Abfolge. Die Bewegung von hier nach dort hat ein Vorher und Nachher, und völlig gleichgültig, ob ich die Zeit für real halte oder nicht, das ist es, was wir unter dem Vergehen von Zeit verstehen. Die Frage, ob Zeit – oder besser die Raumzeit – real ist, ist erst einmal davon gar nicht betroffen und ein anderes Thema. Die Frage, was Zeit tatsächlich ist, kann niemand beantworten, und das weißt du sehr genau, es ist also schon nervig von dir, ständig darauf rumzuhacken. Und was soll das bedeuten: Nicht auf Links im Internet berufen, sondern auf sich selbst? Willst du, dass wir hier ohne jede physikalische Grundlage diskutieren, alles beiseite lassen, was an Erkenntnis vorliegt?
 

  Beitrag 1938-40
Zeit in wenigstens 4 unterschiedliche Bedeutungen

 
 
Gregor Lämmer aus 1938-35:
Henry aus 1938-28:
 
Die Frage, was Zeit tatsächlich ist, kann niemand beantworten . . .

Zeit ist Dauer


Diese Antwort ist richtig, aber keineswegs vollständig, da der Begriff "Zeit" für uns ja mindestens 4 völlig unterschiedliche Bedeutungen hat:
  • Zeit als Dauer
  • Zeit als etwas Vergängliches
  • Zeit als etwas, das wir durchwandern (sprich: Zeit als transitive 2-stellige Relation auf der Menge aller Ereignisse)
  • Zeit als eine der 4 Dimensionen der RaumZeit der ART

 

  Beitrag 1938-42
-

 
 
H... aus 1938-41:
Diese Antwort ist richtig, aber keineswegs vollständig, da der Begriff "Zeit" für uns ja mindestens 4 völlig unterschiedliche Bedeutungen hat:
  • Zeit als Dauer
  • Zeit als etwas Vergängliches
  • Zeit als etwas, das wir durchwandern (sprich: Zeit als transitive 2-stellige Relation auf der Menge aller Ereignisse)
  • Zeit als eine der 4 Dimensionen der RaumZeit der ART

"Zeit ist Dauer" bringt physikalisch keinen Nährwert. Das ist so, als ob ich sage "Geschwindigkeit ist Schnelligkeit"..

Hallo H...,

Du hast sicherlich recht, dass der Zeitbegriff im allgemeinen Sprachgebrauch sehr diffus ist. Man könnte noch eine Menge anderer Begriffsinhalte für das Wort "Zeit" finden. Ich nehme mir jetzt die "Zeit" diese Zeilen zu schreiben, bedeutet z:B. sinngemäß nichts anderes als: "ich habe nichts Besseres vor".

Die Zeit als reine Dauer zu definieren, grenzt allerdings den Zeitbegriff schon mal ein, z.B. auch in dem Sinne, dass damit Zeitpunkte nicht unter diesen Zeitbegriff fallen.

Ich denke, dass genau dieser Zeitbegriff in der Physik verwendet wird, wobei die Dauer noch normiert wird, nämlich in Form von Sekunden, Minuten, Stunden etc., um die Dauer mathematisch beschreiben zu können. Für diese Art von Zeit = Dauer ist die Richtung eines Vorgangs in Form von kausalem Geschehen gleichgültig. Das Leben eines Menschen dauert von der Geburt bis zum Tod genauso lange, wie rückwärts betrachtet vom Tod bis zur Geburt. Ein Film dauert gleichlang, egal ob ich ihn vorwärts oder rückwärts laufen lasse.

Du hast auch recht, dass Dauer im Prinzip nur ein anderes Wort für für Zeit(dauer) ist, und damit noch nicht erklärt ist, wie unsere Vorstellung von dieser Dauer zustande kommt.

Ich denke, dass unser allgemeines "Gefühl" von der Dauer eines Vorgangs dadurch zustande kommt, dass wir aus Erfahrung wissen, dass während eines konkreten Vorgangs unendlich viele weitere Vorgänge stattfinden. Um die Vorstellung der Dauer eines Vorgangs zu bilden, müssen wir diesen Vorgang zu zumindest einem weiteren Vorgang in Beziehung setzen.

Zeit ist daher dem Wesen nach eine Beziehung (Vergleich) zwischen Vorgängen (Geschehensabläufen).

MfG
Harti
 

  Beitrag 1938-66
Die Plack-Einheiten — Grenze der Anwendbarkeit bekannter physikalischer Gesetze

 
 
Grtgrt aus 1938-65:
 
Wrentzsch aus 1938-63:
 
Wer hat eigentlich die Plankzeit entdeckt?
Wie kam man auf die Idee?
 

Hi Wrentsch,

in Wikipedia liest man:

"Die Planck-Einheiten, benannt nach Max Planck, bilden ein natürliches Einheitensystem. Sie werden aus drei Naturkonstanten hergeleitet, nämlich der Gravitationskonstanten G, der Lichtgeschwindigkeit c und dem planckschen Wirkungsquantum h, und markieren teilweise Grenzen der Anwendbarkeit der bekannten Naturgesetze (siehe Planck-Skala)."

Mit anderen Worten:
  • Die Planck-Zeit ist nicht mehr als eine Zeit-Einheit, die so gewählt ist, dass gewisse Formeln möglichst einfache Form annehmen.
  • Dies zu erreichen, wurde sie ale geeignetes Vielfaches des Planckschen Wirkungsquantums definiert (das in all jenen Gleichungen auftritt).
  • Dass es dieses kleinste Wirkungsquantum gibt, ist das eigentlich Erstaunliche. Seine Entdeckung — durch Max Planck — kann als Geburtsstunde der Quantenphysik betrachtet werden.

Gruß, grtgrt
 

Nur zur Ergänzung: Man setzt c, G, h und k als "1", das sind dann die sogenannten "Natürlichen" Konstanten und eine Umrechnung erübrigt sich, kommt ja immer "1" heraus (natürlich nicht in Verbindung mit anderen Einheiten).

Und das "Wirkungsquant", eigentlich Planck-Konstante, ist die Einheit Energie mal Zeit, wie der Name schon sagt, ebenfalls eine Konstante.
 

  Beitrag 1970-1
Verwirrendes

 
 
Da nach Einsteins berühmter Formel E = mc2 Energie und Masse zueinander äquivalent sind, war ich bisher immer der Meinung, dass natürlich auch Energie Gravitationskraft erzeugt.

Nun lese ich aber im Papier eines Physikers [genauer: bei Robert B. Laughlin in "Abschied von der Weltformel" Seite 189 oben:

Zitat:
Reales Licht unterscheidet sich wie realer quantenmechanischer Schall insofern von seinem newtonschen Gegenstück, als es selbst dann Energie enthält, wenn es eiskalt ist. Nach dem Relativitätsprinzip sollte diese Energie Masse erzeugt haben, was wiederum Gravitation hätte hervorrufen müssen.
Wir haben keine Idee, warum das so ist, und so behandeln wir das Problem, wie das vielleicht eine Regierung machen würde, und erklären einfach, der Raum würde keine Schwerkraft besitzen.


Wer bitte kann mir jetzt folgende 3 Fragen beantworten:
  • Erzeugt Energie wirklich keine Gravitationskraft?
  • Was versteht man unter eiskaltem Licht?
  • Und was genau ist quantenmechanischer Schall?

grtgrt
 

  Beitrag 1970-6
-

 
 
Grtgrt aus 1970-1:
Wer bitte kann mir jetzt folgende 3 Fragen beantworten:
  • Erzeugt Energie wirklich keine Gravitationskraft?
  • Was versteht man unter eiskaltem Licht?
  • Und was genau ist quantenmechanischer Schall?
Hi grt,

ich bin der gleichen Meinung wie oberen Schreiber. Der Sinn seiner Sicht ist, dass er Gravitation als emergente Eigenschaft der Entwicklung des Universums sehen will. Übrigens (bin ich stolz darüber) habe ich diese Gedanke schon vor der Lesung dieses Buchs. Sicher ist es nur seine Meinung, mindestens sehe ich bestätigt, dass sie physiklaisch gesehen nicht so abwegig ist.

Zu deinen Fragen:
- Was wäre wenn du uns hier deine Definition von Energie lieferst. Da bist du doch ein Weltmeister ;-)
Meines Erachtens ist die erste Frage nicht logisch korrekt formuliert. Energie erzeugt nicht die Kraft. Energie manifestiert sich durch die Kraftwirkung. Dadurch wird sie auch verändert.
- Eiskalte Licht ist nach mein Verständnis die s.g. Nullpunktenergie gemeint.
- Das Teilchen wird oft als Störung eines quantenmechanischen Feldes dargestellt (Welleneingenschaft). Daher kann man über den Schall sprechen.
Gruß, Irena
 

  Beitrag 1970-8
-

 
 
Grtgrt aus 1970-1:
 
Da nach Einsteins berühmter Formel E = mc2 Energie und Masse zueinander äquivalent sind, war ich bisher immer der Meinung, dass natürlich auch Energie Gravitationskraft erzeugt.

Nun lese ich aber im Papier eines Physikers:

Zitat:
Reales Licht unterscheidet sich wie realer quantenmechanischer Schall insofern von seinem newtonschen Gegenstück, als es selbst dann Energie enthält, wenn es eiskalt ist. Nach dem Relativitätsprinzip sollte diese Energie Masse erzeugt haben, was wiederum Gravitation hätte hervorrufen müssen.
Wir haben keine Idee, warum das so ist, und so behandeln wir das Problem, wie das vielleicht eine Regierung machen würde, und erklären einfach, der Raum würde keine Schwerkraft besitzen.


Wer bitte kann mir jetzt folgende 3 Fragen beantworten:
  • Erzeugt Energie wirklich keine Gravitationskraft?
  • Was versteht man unter eiskaltem Licht?
  • Und was genau ist quantenmechanischer Schall?

grtgrt
 

Man könnte definieren, dass Energie die Fähigkeit ist, zu wirken. Damit Energie wirken kann, muss es ein energetisches "Gefälle" geben, Energie (in diesem Sinne, als Wirkfähigkeit) an sich bewirkt gar nichts.

Physikalisch ist Energie der Masse äquivalent, und Masse ist es, die gravitativ wirkt (die Raumzeit krümmt). Die Krümmung der Raumzeit insgesamt wird immer durch die Gesamtmenge an Masse, also auch der Strahlungsenergie bestimmt. Ein Köper, der beschleunigt wird, erhöht seine Energie und damit seine Masse und krümmt damit stärker die Raumzeit.

Ich vermute, hinter dem "eiskalten Licht" des Autors steckt ein naives Bild von dem, was Energie ist. Wenn ich nämlich Energie im weiteren Sinne mit Wärmestrahlung gleichsetze, komme ich zur Frage, wie noch Wärme abgestrahlt werden kann, falls sich die Temperatur eines strahlenden Körpers dem absoluten Nullpunkt nähert. Wärme ist aber ein relativer Begriff zum einen und zum anderen kann die Temperatur den absoluten Nullpunkt (Heisenberg!) nicht erreichen, weshalb auf jeden Fall Wärme (Energie) abgestrahlt wird.

Korrektur: Obigen, letzten Absatz kann ich so nicht stehen lassen, weil ich hier Laughlin wohl ziemlich falsch verstanden habe, denn er sagt ja gerade, dass auch "eiskaltes Licht" noch Energie ist.
 

  Beitrag 1970-13
Emergenz: Wie die Natur sich selbst organisiert (und dann mehr ist als nur der Summe ihrer Teile)

 
 
Hallo Irena & Henry:

Vielen Dank für eure Antworten. Sie bestätigen mich in meinen bisherigen Ansichten.


Dass Laughlin es offenbar liebt, sich eher burschikos denn genau auszudrücken, finde ich schade. Er könnte anders größere Wirkung erzielen.

Ich mag seinen Stil nicht, teile aber dennoch seine Ansicht, dass Emergenz zur Kenntnis zu nehmen, wirklich wichtig ist.
Dass sie in den Mittelpunkt zu stellen Reduktionismus ersetzen kann, wie er zu glauben scheint, ist aber sicher falsch.


Ich für mich halte fest:

Emergenz
— verursacht durch eine Vielzahl zufällig eintretender Elementarereignisse, die den Drang nach Potentialabbau ständig stören —
ist dafür verantwortlich, dass komplexe Systeme mehr sind als nur die Summe ihrer kleinsten Teile.


Beste Grüße,
grtgrt
 

  Beitrag 1970-14
-

 
 
Vielleicht sollte ich vorausschicken, daß in meinem Denken die Welt nicht aus Teilen besteht. Primär ist das Ganze. Primär ist für mich das Phänomen. Für das reduktionistische Denken steht das Phänomen am Ende einer Kette von Wirkzusammenhängen die unten bei den Elementarteilchen oder noch früher (zB Strings) beginnt. Für das reduktionistische Denken ist das Phänomen (= Wahrnehmung = Bewußtsein...) noch nicht erklärbar. Ich bin mir ziemlich sicher, dass es reduktionistisch überhaupt nicht zu erklären ist. Ich gehe lieber den umgekehrten Weg und versuche die Teile vom Ganzen her zu beschreiben. Und dann landen wir beim Stichwort >Emergenz<.

Emergenz: Teile werden zusammengefügt und bilden eine neue Einheit. Dabei entsteht eine Qualität, die die Teile allein zuvor nicht zeigten und die auch aus der Kenntniss der Teile nicht ableitbar war. Eine neue Qualität. ZB: Nehmen wir an, wir wüßten alles, was es über die Atome Sauerstoff und Wasserstoff zu wissen gibt. Ich behaupte es ist unmöglich aus dieser Kenntnis das Reaktionsprodukt Wasser vorher zu sagen (zu berechnen).
Oder: Nur aus der Kenntnis einer Eizelle und einem Spermium ist es unmöglich vorherzusagen was für ein Tier daraus entstehen wird. Wenn uns also nur die Eizelle eines Alien gezeigt würde, könnten wir fast nichts über diese Wesen vorhersagen.

Ein bekanntes Buch trägt den Titel: "Am Anfang war der Wasserstoff." Sobald man diese Aussage ontologisch, reduktionistisch auffaßt hat man innerhalb der diesem Paradigma verhafteten Wissenschaften keine Chance mehr zu der Welt zu gelangen die wir kennen, weil wir in ihr leben.

Emergenz ist deshalb für mich kein von unten nach oben (kein reduktionistischer) gerichteter Prozess, sondern ein von oben nach unten gerichteter. Und oben steht für mich das Phänomen als solches.
 

  Beitrag 1999-54
Moleküle, die als solche nicht wirklich existieren (oder: Klassische Modelle sind zu ungenau)

 
 


Moleküle, die als solche nicht wirklich existieren



Thomas Görnitz (S. 189 aus "Die Evolution des Geistigen") ist der festen Meinung, dass die Biologen sich demnächst genötigt sehen werden, als Hintergrundtheorie nicht mehr die klassische Sicht der Physik zu verwenden, sondern die weit genauere quantentechnische. Er schreibt:

Zitat:
 
In den meisten biologischen Darstellungen fand man bisher als Hintergrundtheorie nur die klassische Physik. Chemische Verbindungen wurden wie eine Verklebung von klassischen Kügelchen geschildert, und die Formelschreibweise [der Chemiker] tut das ihrige dazu, diese Bilder zu verfestigen.

Dass beispielsweise Kochsalz
  • entweder als Ionenkristall, in dem jedes Ion eine gleichberechtigte Beziehung zu sämtlichen seiner Nachbarn hat,
  • oder als im Wasser gelöste Einzelionen,
  • aber nie als NaCl-Molekül
auftritt, wird nur selten erläutert.

Bei einem Molekül liegt eine recht dauerhafte Bindung von genau definierten Atomen vor. In einem Iopnenkristall hingegen hat man eine symmetrische Anordnung der Atome, bei der keinerlei spezielle Bindung zwischen ausgewählten Nachbarn vorliegt, sondern alle Nachbarn sind gleichberechtigt, und die Formel NaCl verdeutlicht lediglich, dass die Anzahl aller Natriumatome im Kristall genau so groß ist wie der der Chloratome.

Ebensowenig wird in der Regel verdeutlicht, dass Ladungstransport in Elektrolyten vor allem als Informationstransport verstanden werden muss. So werden die Ionen nicht einzeln durch den Elektrolyt geleitet. Stattdessen wird die Informationüber das Einbringen eines Ions an der Eingangselektrode an die Ausgangselektrode geleitet und dort ein identisches Ion abgegeben.

Auf die aus Sicht der Quanteninformation besonders interessante Rolle des Wassers soll hier nur kurz hingewiesen werden:
  • Wasser ist höchstens als Dampf » H2O « aber bereits in Wolken oder als Flüssigkeit NICHT mehr: Wassermoleküle verbinden sich zu Clustern von vielerlei Größe. [Diese Cluster] können sehr verschiedene räumliche und energetische Struktur besitzen und können auch verschieden auf Biomoleküle reagieren.   Hier erwarten wir noch wesentliche neue Erkenntnisse.


Im Rahmen der immer genauer werdenden Betrachtung biologischer Prozesse, so Görnitz, werde es immer wichtiger, statt der klassischen physikalischen Sicht die weit genauere quantentheoretische zugrunde zu legen.

Diese seine Meinung zu belegen zitiert er eine recht bezeichnende Aussage aus Engel, G. et al. "Evidence for wave-like Energy Transfer trough Quantum Coherence in photosynthetic Systems", Nature, Bd. 446 (2007) 782-786:

Zitat von Engel et al.:
 
The spectroscopic data clearly document the dependence of the dominant energy transport pathways ... of the excited-state wavefunctions of the wohle bacterio­chlorophyll complex. But the intricate dynamics of quantum coherence, which has no classical analogue, was largely neglected in the analysis — even though
  • electronic energy transfer involving oscillatory populations of donors and acceptors was first discussed more than 70 years ago,
  • and electronic quantum beats arising from quantum coherence in photosynthetic complexes have been predicted and indirectly observed.


Diese von Görnitz zusammengetragenen, sehr schönen Beispiele belegen recht deutlich, wie ungenau die klassische Sicht sein kann, und wie wenig diese alte Sicht der Tatsache Rechnung trägt, dass ein großes Ganzes eben doch weit mehr als nur die Summe seiner Teile ist (und diese Teile dort ihre eigene Identität sehr weitgehend verlieren können — was kein Wunder ist, wenn man bedenkt, dass sie ja sämtlich nur gedankliche Abstraktionen sind: unser Modell von etwas, das in Wirklichkeit Summe aus Produkten vieler einzelner Wellen ist).


 

  Beitrag 1057-142
Eine treffende Feststellung — sollte man sich merken

 
 
Harti aus 1057-136:
Grtgrt aus 1057-123:
Mir legt das den Verdacht nahe, dass die Natur die Begriffe unvorstellbar klein und unvorstellbar groß irgendwie als gleichwertig betrachtet.

Hallo Grtgrt,

ich bin der Meinung, dass der Gegensatz klein-groß eine Folge unserer Methode zur Erfassung der Natur ist und keine Eigenschaft von Objekten.

Hi Harti,

das finde ich sehr treffend - muss ich mir merken.

Danke, grtgrt
 

  Beitrag 1057-192
Mathematische Konstanten existieren in nur einer Version

 
 
Hallo H...,

H... aus 1057-191:
Ich glaube, die Größe π, die du hier ansprichst, ist nicht wesentlich. Bei einer anderen Mathematik wäre der Wert anders (hier gab es historisch viele Versuche...).

Pi ist ein reiner Quotient und keine Größe. Dieser Quotient ist unabhängig vom Zahlensystem immer der gleiche. Der Umfang ist immer etwa dreimal so groß wie der Durchmesser eines Kreises.
Was meinst du mit "einer anderen Mathematik" bei der der Umfang nicht ca. dreimal so groß sein sollte?

Du verwechselst reine Zahlenbeträge, die sicher abhängig sind von willkürlich gewählten physikalischen Einheiten, mit den Verhältnissen in der Natur.

Zitat von H...:
Es gibt theoretischen Überlegungen, wonach verschiedenen "Längenmessungen" existieren (und wir bisher immer nur eine davon anwenden) - und die sind völlig äquivalent. Das würde dazu führen, dass dein 2. Bsp. rein betragsmässig anders aussieht.

Ich weiß nicht, von welchen theoretischen Überlegungen du sprichst und vielleicht verrätst du es uns ja noch. Du wirst doch wohl nicht lediglich von Einheitensystemen reden? Derer gibt es mehrere, praktische und weniger praktische. Das wäre nun wirklich trivial und keiner theoretischen Überlegung wert. Das Wirkungsquantum ist davon wohl kaum abhängig, genausowenig wie von der Mathematik, ob unsere oder irgendeiner "anderen Mathematik", von der du sprichst.

mfg okotombrok
 

  Beitrag 1107-25
Beantworten wir Irenas Argumente

 
 
Hallo Irena,

da du mir so ausführlich geantwortet hast, will ich es auch tun. Also:

Irena aus 1107-20:
Hallo Doppel-grt,
ich denke, dass es gerade du muss über den Kopf gehen lassen, da das Problem der Selbstbezüglichkeit ist KEIN Denkfehler. Es ist objektive Gegebenheit. Du schreibst den Unvollständigkeit Satz zwar auf, dennoch ziehst von ihm keine Schlussfolgerung. Im Gegenteil, du läufst mit Flage, auf der "der Denkfehler" - also subjektive Gegebenheit - geschrieben ist.

Ich behaupte ja gar nicht, dass der Denkfehler in der Selbstbezüglichkeit liegt — er liegt einfach nur darin, dass manche Leute (du etwa) nicht einsehen wollen, dass z.B. die Aussage "Diese Aussage ist falsch." einem logischen Gleichungssystem entspricht, welches KEINE Lösung hat. Paradox ist daran gar nix, denn die Mathematik kennt jede Menge von Gleichungssystemen, die keine Lösung haben.


Irena aus 1107-20:
Zitat von grtgrt:
Das so zur Verlässlichkeit heute üblicher mathematischer Denkwerkzeuge!

So viel Emotionen!..

Ich sehe es überhaupt nicht so tragisch wie du. Mathematiker haben in Jahrtausenden ihre Gebäude gebildet und diese Gebäude sich als verlässlich aufweist. Deine Verallgemeinerung der Unzuverlässlichkeit der mathematischen Werkzeuge ist von dir stark übertrieben.

Diese meine Aussage bezieht sich einfach nur auf die Tatsache, dass Mathematiker (Algebraiker z.B.) heutzutage fast jeden ihrer Beweis mit Hilfe naiver Mengenlehre formulieren, obgleich sie doch wissen, dass Russell gezeigt hat, dass naive Mengenlehre zu Widersprüchen führen kann (also nicht wirklich zuverlässig ist).

WARNUNG also: Wer naive Mengenlehre gebraucht, muss das hinreichend vorsichtig tun, also sicherstellen, dass jede Definition, die er mit ihrer Hilfe formuliert, auch wirklich Sinn macht (was manchmal selbst schon einen eigenen, komplizierten Beweis erfordern kann).


Irena aus 1107-20:
Für mich die Unvollständigkeitsätze von Gödel nur eines bedeuten, eben das, was ich schon vorher geschrieben habe: die Mathematik ist ein Medium, das physikalische Realität widerspiegelt. Ähnlich stoßt auch Physik an ihre Grenzen: der Messungsgrenzen, der Grenzen der physikalischen Realität.

Du hast völlig recht: In der Mathematik (genauer: in mathematischen Gesetzmäßigkeiten) spiegelt sich physikalische Realität.

Mit irgendwelchen Messgrenzen aber hat das gar nichts zu tun (warum auch?).


Irena aus 1107-20:
Es ist ein schönes Satz von Andre Weil, der gesagt haben soll: "Gott existiert, weil Mathematik konsistent ist, und Teufel existiert, weil wir das nicht beweisen können".

Ich schätze Andre Weil sehr, da ich seine mathematischen Fähigkeiten wirklich bewundere.

Dennoch sollte er den Teufel besser nicht bemühen, denn Gödels Ergebnis ist letztlich nur Ausdruck der Tatsache, dass allein schon die natürlichen Zahlen uns in ganz unglaubliche mathematische Tiefen zu führen im Stande sind — in eine Welt, in die man ohne Ende immer tiefer gehen kann, ganz so wie man sich in eine Mandelbrotmenge ohne Ende immer tiefer hinein zoomen kann und dann dort eine schier unendliche Vielfalt von Formen und Farben entecken kann (siehe z.B. Abschnitt 1 bis 14 in Wikipedia). Die Menge aller Primzahlen z.B. ist ein Wunder für sich, und wir Menschen haben gerade erst begonnen, die allereinfachsten Strukturen dieser Menge zu verstehen ...

Leopold Kronecker (ein Zahlentheoretiker) hat mal voller Bewunderung gesagt:

Die ganzen Zahlen hat der liebe Gott gemacht, alles andere ist Menschenwerk.

Da ist viel Wahres dran.


Irena aus 1107-20:
Zu der subjektiven Grundlage der Mathematik. Da wir obendrauf ein objektives Gebäude haben, dann wird wohl das Subjektive an das Objektive anpassen müssen. Daher hat das Subjektive auch nicht volle Freiheit. Oder wenn Mathematiker eigenen subjektiven Empfindungen nachgeht, kann er vielleicht ein anderes Gebäude nebenan schaffen. Auf die Weise mathematische "Stadt" nur gewinnt. Daher bitte hier keinen Weltuntergang sehen.

Wenn du denkst, ich würde irgendeinen Weltuntergang sehen, dann versteht du mich völlig falsch.

Ich bin aber sehr wohl der Ansicht, dass die Informatik — was erfolgreiche Qualitätssicherung ihrer Ergebnisse betrifft — der Mathematik um etwa 2 Jahrtausende hinterher hinkt. Nur darauf wollte ich hinweisen.


Irena aus 1107-20:
Zitat:
Mathematische Beweise im Bereich nicht konstruktiver Mathematik sind aber nur Zerlegung großer Gedankensprünge in zunehmend kleinere bis hin zu einem Punkt, an dem der Zuhörer der Mei¬nung ist, jeder dieser kleinen Gedankensprünge bedürfe keines Beweises mehr, sondern behaupte offensichtlich Wahres. Wo dieser Punkt erreicht ist, muss subjektiv entschieden werden und hängt sehr von der Vorbildung des Zuhörers ab sowie von seiner Kritikfähigkeit.

Ich denke, dass "wo dieser Punkt erreicht wird" hängt nicht von der Subjektiven. Du übersiehst nämlich, dass Mathematik ist s. z. gemeinschaftliches Produkt. Wie in jedem Gemeinschaftsprodukt, muss das Erreichte von einem durch anderen geprüft werden.

Ich bleibe bei meiner Meinung.
... und wer Andre Weils Buch "Basic Number Theory" wirklich durchgearbeitet hat, wird wissen, was ich meine (denn was er als einen schon hinreichend kleinen, überzeugenden Gedankensprung da so alles stehen lässt, kann schon mal ein bis zwei Stunden Arbeit kosten, es einzusehen. Einen Fehler aber konnten wir, die wir sein Buch damals im Detail durchgearbeitet haben, ihm aber wirklich NIEMALS nachweisen. Wie gesagt: Ich bewundere ihn ...


Irena aus 1107-20:
Zitat:
Der allseits geachtete Physiker Steven Hawking scheint zu denken, jedes mathematische Modell sei auch ein physikalisches.

Passt auf wie dein Satz ändert, wenn du statt Begriff "ist" (was einer Äquivalenz zeigt) den Begriff "entspricht" nutzt. "Jedes mathematische Modell entspricht einem physikalischen". Andere Begriff zeigt auf den Bezug, nicht Äquivalenz.

Wobei auch hier ein Denkfehler, weil es ein Gegenteil richtig ist: jedes physikalischen Model entspricht dem mathematischen, da wir wissen, es gibt mathematische Modelle, die die Naturwissenschaft Physik "sprengen". Die Anwendung in Quantenmechanik verschiedener mathematische Modelle zeigt nur innere Beziehung der mathematischen Modelle.

Dein erstes Argument scheint mir einfach nur ein Spiel mit Worten — es sagt rein gar nichts.

Zum zweiten: Wer etwas modelliert (mit oder ohne Mathematik) wird i.A. zu einem Modell kommen, welches irgendwo hinkt, und somit den zu modellierenden Gegenstand eben NICHT mehr in allen Einzelheiten korrekt abbildet. Das sehe ich als Selbstverständlichkeit — es ist sogar zwangsläufig so, denn ein Modell soll ja häufig auch vereinfachen (sich also auf nur bestimmte Aspekte konzentrieren).


Irena aus 1107-20:
Zuletzt ein Beispiel, der mich auf Palme bringt, wenn es lese (bei deinen Ausführungen leider zu viel ähnliches Textes). Nehmen wir den folgenden Absatz zur Analyse:
Zitat:
Ein Vergleich der Ergebnisse von Kurt Gödel einerseits und Gerhard Gentzen sowie Paul Lorenzen andererseits zeigt meiner Ansicht nach deutlich, dass konstruktive und nicht konstruktive Modelle grundsätzlich verschiedener Natur sind. Der Unterschied besteht darin, dass viele Ergebnisse indirekter Beweisführung schon ihrer Natur nach nicht Ergebnis rein konstruktiver Mathematik sein können. Es bleibt offen, ob man auch sie als mathematische Wahrheiten sehen kann, die mehr sind als nur eine gedankliche Konstruktion.

Zitat:
...konstruktive und nicht konstruktive Modelle grundsätzlich verschiedener Natur sind.

Okay, nehmen wir es an. Es ist mir hier die Richtigkeit der Annahme von sekundäre Bedeutung ist.

Zitat:
Der Unterschied besteht darin, dass viele Ergebnisse indirekter Beweisführung schon ihrer Natur nach nicht Ergebnis rein konstruktiver Mathematik sein können.
Bitte liest dein Satz aufmerksam. Wie sprechen über grundsätzliche Unterschiede, oder? Wenn man spricht über "viele", bedeutet es nicht an "alle" anwendbar und daher nicht grundsätzlich sein kann.

Auch was du hier sagst, scheint mir nur ein Spiel mit Worten.

Dass mein Statement "... viele Ergebnisse indirekter Beweisführung schon ihrer Natur nach nicht Ergebnis rein konstruktiver Mathematik sein können" durchaus richtig ist, zeigt sich sehr schön in Beitrag 1107-22, wo wir im zweiten Kullerpunkt ja tatsächlich einen indirekten Beweis führen. Über rein konstruktive Mathematik wäre ganz sicher NICHT beweisbar, dass K lügt.


Irena aus 1107-20:
Nächste Vorwand. Mit dem Satz solltest du den ersten Satz erklären bzw. erweitern. Du machst aber Zirkelsprung und erklärst die Annahme durch sie selbst.

Ich habe keine Anhnung, wo da ein Zirkelschluss sein sollte.


Irena aus 1107-20:
Übrigens der Tisch ist nicht gleich Stuhl, gehören beide aber zu Möbel. "Die Natur" von beiden sicher verschieden ist: auf einem sitzen wir, auf anderen essen. Dennoch durch eine übergeordnete Klasse "das Möbel" zeigen sie ihre gemeinsame "Natur". Ähnlich ist mit der direkten und indirekten Beweisführung.

Da bin ich völlig deiner Meinung:
  • Beide Beweiswege sind mathematisch exakte Beweiswege (so wie Stuhl und Tisch beides Möbel sind),
  • sind aber dennoch verschieden (so wie auch Stuhl und Tisch verschieden sind).


Irena aus 1107-20:
Zitat:
Es bleibt offen, ob man auch sie als mathematische Wahrheiten sehen kann, die mehr sind als nur eine gedankliche Konstruktion.

"Sie" muss wohl indirekte Beweisführung sein. Ist direkte Beweisführung etwas anders als gedankliche Konstruktion?! Da musst du wohl erklären, da für mich es offensichtlich ist. Wenn ich auch semantischen Inhalt dieses Satzes ein Augenblick nicht beachte, dann trotzdem ist mir nicht klar, warum aus den vorhergehenden Kontext muss etwas "offen bleiben". Aus der vorigen Sätze folgt keinesfalls die Offenheit des Problems.

Nachdem man indirekt bewiesene Wahrheiten nicht konstruieren kann, könnte man sich schon fragen, ob sie wirklich einen realen Teil der Natur darstellen.
Problem nur: Wie will man den Begriff real den nun wirklich genau definieren?


Irena aus 1107-20:
u. s. w.

Schade, dass dir hier offenbar die Puste aus geht ...

Gruß, grtgrt
 

  Beitrag 1107-44
Was genau ist denn nun eigentlich Realität (reale Wirklichkeit)?

 
 
Irena aus 1107-28:
Zitat von grtgrt:
Nachdem man indirekt bewiesene Wahrheiten nicht konstruieren kann, könnte man sich schon fragen, ob sie wirklich einen realen Teil der Natur darstellen.
Problem nur: Wie will man den Begriff real den nun wirklich genau definieren?

O man, ich sehe viel mehr Begriffe, die genau definiert müssen. Vielleich müsstest du mehr Aufmerksamkeit der Philosophie schenken als Matheamtik.

Irena:

Was ich in Beitrag 1896-9 schrieb, scheint mir Beweis genug, dass ich auch der Philosophie genügend Beachtung schenke — was mich aber keineswegs daran hindert, den mehr pragmatischen Standpunkt der Physiker mehr zu schätzen (z.B. deswegen, weil ich keineswegs sicher bin, dass moderne Philosophen immer so ganz genau wissen, wovon sie denn eigentlich reden):


Grtgrt aus 1896-9:
 
Auf jeden Fall muss man zur Kenntnis nehmen, dass die Anfang des 20. Jahrhunderts entwickelte Quantentheorie eine philosophische Revolution zur Folge hatte, deren zentrale Erkenntnisse waren:

Die Natur funktioniert keineswegs voll deterministisch.

und auch alle sonstige Realität, existiert stets nur ungenau.


Einstein war der letzte große Physiker, der das nicht glauben konnte.

John Wheeler hat den Unterschied der alten und der neuen Weltsicht auf den Punkt gebracht über ein erfundenes Gesprächs zwischen Baseball Schiedsrichtern:
  • Erster (Einstein): Ich entscheide, wie es ist.
  • Zweiter (Bohr): Ich entscheide, wie ich es sehe.
  • Dritter (Messung): Solange ich nichts entscheide, war auch nichts.
Ironischerweise ergänzt die Relativitästheorie:

... und keine zwei Schiedsrichter sehen dasselbe.


Der Physiker Roger G. Newton sagt deswegen auch folgerichtig (1993):

Physiker kümmern sich nicht um so metaphysische Begriffe wie Existenz; sie versuchen nicht, die letzte Realität zu ergründen, was auch immer man darunter verstehen mag. Die Bausteine für unsere grundlegenden Vorstellungen über die Welt werden danach ausgewählt, ob sie ein kohärentes Gedankengebäude ermöglichen.

 

Gruß, grtgrt
 

  Beitrag 1896-1
Was Niels Bohr uns sagt

 
 
Niels Bohr — Nobelpreisträger und Mitbegründer der Quantenphysik — soll einmal gesagt haben:

Zitat:
Die Physik kann nicht ergründen, wie die Natur funktioniert.
Aufgabe der Physik ist lediglich, zu untersuchen, wie die Natur sich uns zeigt.

Auch Richard Feynman war der Meinung:

Zitat:
I can savely say: Nobody understands Quantum Physics.

Damit scheint klar: Physik besteht aus zwei Teilen

dem Beobachten der Natur (Experimentalphysik)
und dem Modellieren der Natur (Theoretische Physik).

Man sollte aber niemals glauben, so ein Modell sei die Natur.


Warum aber modelliert man die Natur ausschließlich über Mathematik? Und warum scheint die dafür so geeignet? Hier die Antwort darauf:

Mathematik besteht aus zweierlei: Einer Denkmethodik und der Menge aller mathematischen Gesetze.

Was wir als mathematische Methodik haben ist lediglich der beste Weg, den der Mensch bisher fand, über mathematische Gesetze nachzudenken (dieser Weg könnte austauschbar sein und wird, sobald Quantencomputer verfügbar sind, fast sicher eine Art Bruder bekommen).

Ganz anders mathematische Gesetze: Sie existieren ganz unabhängig davon, ob der Mensch sie kennt oder nicht — der Mensch kann sie entdecken, aber z.B. nicht abändern.

Damit ist klar:

Mathematische Gesetze sind Teil der Natur
und Mathematik kann als der Teil der Physik verstanden werden, der diesen Teil der Natur entdeckt
und diskutierbar macht (modelliert).


Insbesondere gilt: Die Stringtheorie ist eine rein mathematische Theorie, und so ist jeder Zusammenhang, den sie entdeckt, ein mathematisches Gesetz und damit wirklich Teil der Natur — ein Puzzlestein also, von dem aber recht lange unklar sein kann, in welches unserer (Teil-) Modelle der Natur er denn am besten passt.

Da Stringtheorie eine Art Gleichungssystem ist, das — so schätzt man — etwa 10500 Lösungen hat, deren jede Modell eines möglichen Universums ist, wird klar, wie viele Puzzlesteine es mindestens gibt und wie wenig davon wir bisher schon kennen (!).

 

  Beitrag 1896-3
Was ist real?

 
 
Bauhof aus 1896-2:
Grtgrt aus 1896-1:
 
Warum aber modelliert man die Natur ausschließlich über Mathematik? 

Hallo Grtgrt,

das sehe ich etwas anders, nämlich so:
Der Mensch erstellt ein Modell der Natur und beschreibt dieses Modell mit Hilfe der Mathematik.

Hallo Herr Bauhof,

Sie sehen das eindeutig zu eng, denn ganz grundsätzlich gilt:

Es gibt

beobachtete, vermutete, und extrapolierte physikalische Ergebnisse:

  • Beobachtete sind die durch Experimente bestätigten (das was uns Experimentalphysik und Astronomie liefern).
  • Vermutete Ergebnisse sind solche, die als Idee schon entstanden, in der Welt um uns herum aber noch nicht beobachtet werden konnten.
  • Extrapolierte sind jene vermuteten, die uns durch rein mathematische Überlegungen nahegelegt werden.

Hier zwei Beispiele extraplolierter Physik:
  • Quantenverschränkung ist ein physikalisches Phänomen, welches zunächst rein nur auf mathematischem Wege entdeckt wurde. Die entsprechende Herleitung stammt von Einstein, Podolosky und Rosen und wurde — mindestens durch Einstein — als eine Widerlegung der Korrektheit des mathematischen Modells der Quantenmechanik angesehen: Dass eine derart "spukhafte Fernwirkung" wirklich existieren könnte, haben Einstein, Podolsky und Rosen einfach nicht glauben wollen. Erst gut 40 Jahre später hat Experimentalphysik sie nachweisen können — und das auch nur deswegen, weil (etwa 1960) John Bell die sog. Bellsche Ungleichung entdeckt hatte (ihr Gegenstand sind gewisse Wahrscheinlichkeiten). Man erkennt an diesem Beispiel, wie lange es dauern kann, bis aus extrapolierter Physik tatsächlich beobachtete wird.
  • Supersymmetrie ist ein zweites wichtiges Beispiel extrapolierter Physik. Experimentalphysik konnte in der Welt um uns herum noch keinen Beweis dafür finden. Sie ist bislang einfach nur eine Entdeckung der Stringtheorie, dort aber wichtig, denn nur jene Modelle der Stringtheorie, die supersymmetrisch sind, reproduzieren alle aus dem Standardmodell der Elementarteilchenphysik bekannten Partikel. Da supersymmetrische Transformation eines supersymmetrischen Weltmodells Bosonen und Fermionen miteinander vertauscht ohne die physikalischen Aussagen des Modells zu verändern, würde — das grundsätzlich verschiedene Verhalten von Bosonen und Fermionen berücksichtigt — ohne Mathematik niemand auf die Idee kommen, unsere Welt könne supersymmetrisch sein. Dennoch haben schon 1970 zwei Gruppen von Forschern — unabhängig voneinander, und nur mit Hilfe mathematischer Überlegungen — bewiesen, dass das Standardmodell verträglich wäre mit einer Vertauschung der Rollen, die darin Bosonen und Fermionen spielen. Damit gibt es mindestens zwei grundverschiedene mathematische Argumente, die darauf hindeuten, unsere Welt könne supersymmetrische sein.

Siehe auch: Zu extrapolierender (und auch extrapolierter) Physik.

Mit besten Grüßen,
Gebhard Greiter (= grtgrt)

 

  Beitrag 1896-4
Naturgesetze

 
 
Bauhof aus 1896-2:
Grtgrt aus 1896-1:
 
Ganz anders mathematische Gesetze: Sie existieren ganz unabhängig davon, ob der Mensch sie kennt oder nicht — der Mensch kann sie entdecken, aber
z.B. nicht abändern. 

Das ist die Auffassung der ’Platoniker’ unter den Mathematikern. Aber auch das sehe ich anders. Die Mathematik gilt als menschliche Geisteswissenschaft und nicht als Naturwissenschaft. Ohne die Existenz von Menschen existiert auch keine Mathematik.

M.f.G. Eugen Bauhof

Hallo Eugen,

Als menschliche Geisteswissenschaft kann ich beim besten Willen nur mathematische Methodik sehen — niemals aber mathematische Gesetzmäßigkeiten; die nämlich haben mit absoluter Sicherheit schon immer gegolten — lange bevor es Menschen, die Erde, oder gar unser Sonnensystem gab.

Hier drei Beispiele:
  • Es gibt sicher keinen Sinn anzunehmen, dass z.B. das mathematische Gesetz, welches von Pythagoras entdeckt wurde (und etwas über einen Zusammenhang zwischen den Längen der Seiten rechtwinkliger Dreiecke aussagt), nicht schon vor ihm richtig war.
  • Ein noch überzeugenderes Beispiel: Sämtliche mathematischen Gesetze, die gleichseitige 5-Ecke betreffen (und wie man sie zur Oberfläche eines fußballartigen Körpers anordnen kann), sind durch die Natur selbst schon lange vor dem Menschen genutzt worden: so etwa beim Bau der Fullerene (das sind hoch sysmmetrische Moleküle bestehend aus je 60 Kohlenstoffatomen).
  • Auch jeder in der Natur vorkommende Kristall hat eine Struktur, die den Gesetzmäßigkeiten 3-dimensionaler Geometrie gehorcht. Wir sehen: Kristalle und komplexe Moleküle sind eine Art Buch, in dem die Natur selbst mathematische Gesetze formuliert und demonstriert (!).
Kurz: Man muss nicht Platoniker sein, um zu sehen, dass mathematische Gesetze Naturgesetze sind.

Mit besten Grüßen,
grtgrt (= Gebhard Greiter)
 


PS: Hier noch ein Beispiel für ein mathematisches Naturgesetz, das wir noch nicht kennen, dessen Auswirkung wir aber beobachten:

Zitat von Igor und Grichta Bogdanov (Inhaber des Lehrstuhls für Kosmologie an der Uni Belgrad):
Wenn Sie im Sommer an einer Wiese vorbeigehen, pflücken Sie doch mal wahllos ein paar Margeriten, und zählen Sie ihre Blütenblätter. ... Sie werden keine finden, die 7 hat, oder 16. Wieso? Weil auch die Zahl der Blütenblätter einer Blume kein Zufall ist. Tatsächlich folgt sie einem mathematischen Gesetz, das in den Tiefen der Blüte verborgen wirkt.

Und wieder ist die Frage: Woher kommt dieses Gesetz?
 

  Beitrag 1896-5
Natur und Modell

 
 
Grtgrt aus 1896-1:
Physik besteht aus zwei Teilen

dem Beobachten der Natur (Experimentalphysik)
und dem Modellieren der Natur (Theoretische Physik).

Man sollte aber niemals glauben, so ein Modell sei die Natur.

Aus was besteht eigentlich der Beobachter?
 

  Beitrag 1896-8
absolutes Axiom

 
 
Henry aus 1896-6:
Zitat von Gebhard:
Kurz: Man muss nicht Platoniker sein, um zu sehen, dass mathematische Gesetze Naturgesetze sind.

Mit besten Grüßen,

Hallo, Gebhard!

Ich denke nicht, dass die Mathematik grundlegend ist. Das System "Mathematik" beruht auf Lehrsätzen, die wiederum von Axiomen abhängen. Nimm z. B. Pi, die Kreiszahl. Sie beruht auf dem Verhältnis von Radius und Umfang. Aber nur in der Euklidischen Ebene! Lokal beschreibt Pi selbstverständlich das Verhältnis korrekt. Aber ist unser Kosmos denn mit der Euklidischen Geometrie zu beschreiben? Und wenn nicht, mit welcher Geometrie dann? Riemann? Minkowski? Ist er denn überhaupt lokal? Welche Geometrie beschreibt den Kosmos bzw. genauer gesagt die Raumzeit?
 
Hi Henry,

kann Dich verstehen; auch ich selbst dachte lange Zeit, die Mathematik sei nur ein gedankliches Gebäude. Warum aber lassen sich dann selbst so völlig unvermutete Eigenschaften der Natur (wie etwa Quantenverschränkung) mit ihrer Hilfe entdecken?

Erst seitdem mir klar wurde, dass mathematische Gesetze auch dann gelten, wenn wir sie nicht kennen, und dass sie von ganz anderer Qualität sind als mathematische Methodik, bin ich der Überzeugung, sie seien Naturgesetze.

Methodik ist austauschbar — Naturgesetze sind es nicht.

ABER: Naturgesetze sind Aussagen, die unter jeder Voraussetzung wahr sind.

Wenn ich also sage, jedes mathematische Gesetz sei ein Naturgesetz, dann meine ich damit natürlich eine Formulierung dieses Gesetzes, die alle notwendigen Voraussetzungen explizit nennt. (So weit also zu Deinen Beispielen).

Mehr zu diesem Thema findet sich auf meiner Seite Welcher Teil der Mathematik ist absolutes Axiom?.

Beste Grüße,
grtgrt = Gebhard Greiter
 

  Beitrag 1896-9
Zum Wesen physikalischer Aussagen — ein Schiedsrichter irrt

 
 
Henry aus 1896-6:
Und was beschreiben die mathematischen Modelle der Physik? Das Verhalten von physikalischen Objekten, die Stärke der Kräfte, ihre Beziehungen zueinander. Ich kenne kein Modell, dass uns sagt, was physikalische Objekte SIND. Gibt es ein mathematisches Modell, das das leistet? Was IST ein Elektron? Was IST eine elektrische Ladung?

Hi Henry,

Niels Bohr soll mal gesagt haben:

Die Physik kann nicht ergründen, wie die Natur beschaffen ist.

Aufgabe der Physik ist nur, darüber zu diskutieren, wie die Natur sich uns zeigt.


Kein Wunder also, dass du kein Modell kennst, das uns sagt, was dieses oder jenes physikalische Objekt (als Teil der Natur gemeint) denn nun wirklich ist.

Alles was wir haben, sind Modelle, mit denen wir versuchen, die Natur nachzubilden mit dem Ziel, vorhersagbar zu machen, welches Verhalten sie in dieser oder jener Situation zeigen wird.

Noch genauer: Unsere Modelle sind Mechanismen, das Verhalten der Natur nachzubilden — keineswegs aber was sie wirklich ist (und so ist es kein Wunder, dass Stringtheoretiker zunehmend mehr völlig unterschiedliche Modelle finden, die identisches physikalisches Verhalten bedeuten).


Siehe auch: Zum Wesen physikalischer Aussagen.

Beste Grüße,
grtgrt = Gebhard Greiter
 

PS: Heisenberg übringens sah das wie Bohr, als er schrieb:

Zitat von Heisenberg:
Bei Experimenten über atomares Geschehen haben wir es mit Dingen und Tatsachen zu tun, die ebenso real sind wie irgendein Phänomen im täglichen Leben. Aber die Atome und Elementarteilchen sind nicht gleichermaßen real; sie bilden eher eine Welt von Möglichkeiten als eine von Dingen oder Tatsachen.
Quelle: Werner Heisenberg: Physik und Philosophie. Stuttgart: Hirzel, 1959


 
Auf jeden Fall muss man zur Kenntnis nehmen, dass die Anfang des 20. Jahrhunderts entwickelte Quantentheorie eine philosophische Revolution zur Folge hatte, deren zentrale Erkenntnisse waren:

Die Natur funktioniert keineswegs voll deterministisch.

und auch alle sonstige Realität, existiert stets nur ungenau.


Einstein war der letzte große Physiker, der das nicht glauben konnte.

John Wheeler hat den Unterschied der alten und der neuen Weltsicht auf den Punkt gebracht über ein erfundenes Gesprächs zwischen Baseball Schiedsrichtern:
  • Erster (Einstein): Ich entscheide, wie es ist.
  • Zweiter (Bohr): Ich entscheide, wie ich es sehe.
  • Dritter (Messung): Solange ich nichts entscheide, war auch nichts.
Ironischerweise ergänzt die Relativitästheorie:

... und keine zwei Schiedsrichter sehen dasselbe.


Der Physiker Roger G. Newton sagt deswegen auch folgerichtig (1993):

Physiker kümmern sich nicht um so metaphysische Begriffe wie Existenz; sie versuchen nicht, die letzte Realität zu ergründen, was auch immer man darunter verstehen mag. Die Bausteine für unsere grundlegenden Vorstellungen über die Welt werden danach ausgewählt, ob sie ein kohärentes Gedankengebäude ermöglichen.

 

  Beitrag 1896-11
Ein physikalisches Objekt — was genau ist das?

 
 
Hi Harti,

meine Aussage auf Seite Zum Wesen physikalischer Aussagen

Zitat von grtgrt:
Physikalische Objekte sind nichts anderes als gedankliche Modelle,
die der Mensch sich macht, aus dem Wunsch heraus,
das Verhalten der Natur verstehbar und vorhersagbar zu machen.

darf natürlich nicht so verstanden werden, dass umgekehrt auch jedes gedankliche Modell ein physikalisches Objekt sei.

Gruß, grtgrt
 

  Beitrag 1896-14
Realität, Abstrakion, oder ...

 
 
Harti aus 1896-12:
Hallo Grtgrt,

dann sind wir uns ja wohl einig, dass eine Gleichsetzung von gedanklichem Modell (Vorstellung) und physikalischem Objekt (Wirklichkeit) unzweckmäßig ist.
MfG
Harti
 
Nein Harti,
wir sind uns da NICHT einig, denn:

Niels Bohr und Werner Heisenberg (siehe Beitrag 1896-9) sagen ganz klar, dass man die Natur nicht verwechseln darf mit dem Bild, das wir uns von ihr machen. Heisenberg sagt explizit, dass z.B. Atome oder Elementarteilchen mehr eine Welt von Möglichkeiten sind als eine von Dingen oder Tatsachen.

Atome, Elementarteilchen, und auch jedes andere sog. physikalische Objekt sind Subobjekte unseres Verständnisses von der Natur (des Modells also, das wir uns von der Natur machen) — sind aber nicht notwendig genau so auch Teil der Natur selbst.

Die Umgangssprache, Du und fast jeder Mensch sonst auch ignoriert diesen Unterschied.

Das ist meist tragbar, aber sicher nicht dort, wo man — wie hier in diesem Blog — den Dingen und ihrem Wesen wirklich auf den Grund gehen möchte.

Allein schon die Tatsache, dass sich die Definition wenig gut verstandener physikalischer Objekte ändert, sobald man sie besser versteht, zeigt, dass Bohr und Heisenberg recht haben.

Mfg, grtgrt
 

  Beitrag 1896-15
Wirklichkeit

 
 
Henry aus 1896-13:
... es bleibt das gravierende Problem, dass - FALLS - physikalische Objekte nur gedankliche Modelle sind, es keine Wirklichkeit außerhalb eines Bewusstseins gibt.

Hi Henry,

genau das Gegenteil ist der Fall:

Nach Bohr und Heisenberg kann die Realität sehr wohl außerhalb unseres Bewusstsein liegen. Beide betrachten das eher als den Normalfall.

Bitte beachte: Was uns bewusst ist, machen wir zu einem Teil unseres Modells der Wirklichkeit.

Gruß, grtgrt
 

  Beitrag 2039-63
Zum Unterschied von Wirklichkeit und Realität

 
 

Zum Unterschied von Wirklichkeit und Realität


In Dürr & Österreicher: Wir erleben mehr als wir begreifen liest man:

Zitat von Dürr, S. 37 und 39:
 
Wirklichkeit ist im Grunde Potentialität, nicht Realität.

Das eine, unauftrennbare, potentielle Wellengebirge2, einem Weltmeer gleich, kann sich aber so überlagern, dass das Gewoge sich nur an wenigen Stellen addiert, sich konstruktiv überlagert und sich im Übrigen, durch destruktive Interferenz, zu Null, einer glatten See, herausmittelt.

So entsteht am Ende die Realität als Ergebnis eines grandiosen ausmittelnden Überlagerungseffektes.

... das, was sozusagen überlebt bei all diesen Überlagerungsprozessen, das ist das, was wir sehen, was wir hinterher als Realität1 anfassen.

1 Realität von lat. res = das Ding, die Sache


2 Wird von mir als Summe aller Kraftpotentiale verstanden

 

  Beitrag 1896-17
Physik kann nicht entscheiden ...

 
 
Hi Henry,

es ist mir absolut unverständlich, warum mein Argument bedeuten sollte, dass wir die Wirklichkeit schaffen.

Wir schaffen Modelle, sprich: physikalische Objekte — Theorien also, die zeigen, wie wir uns die Natur vorstellen. Ob sie wirklich so beschaffen ist, steht auf einem ganz anderem Blatt und kann — so sagt Niels Bohr — durch die Physik nicht entschieden werden.

Du scheinst physikalische Objekte als Teil der Natur zu sehen, Bohr tut das nicht, und so findet die moderne Theoretische Physik es auch gar nicht mehr so verwunderlich, dass es für ein und denselben Teil der Natur deutlich unterschiedliche Modelle geben kann, die beide gleiches Naturgesetz zum Ausdruck bringen (also gleich richtig sein können).

Gruß, grtgrt
 

  Beitrag 1896-20
Ein missverständlicher Begriff: physikalisches Objekt

 
 
Grtgrt aus 1896-19:
 
Henry und Harti,

die Tatsache, dass ihr — anders als ich — den Begriff "physikalisches Objekt" als Synonym für "ein Teil der Natur" seht, zeigt mir, dass ich in Zukunft wohl besser von physikalischen Modellen (statt Objekten) sprechen sollte.

Ein Elektron, aber z.B. auch ein ganzes Universum, wären dann also stets zu verstehen als ein Teil der Natur, dessen Verhalten wir zu beschreiben suchen über ein oder mehrere — gleichwertige oder einander ergänzende — Modelle.

Danke & Gruß,
grtgrt
 

Hallo, Gebhard,
wir kommen der Sache langsam näher! Ich kann nur für mich sprechen: Mit einem "physikalischen Objekt" meine ich einen quantifizierbaren, also messbaren Teil der Natur und nicht das Modell davon. (Dass sich die Physik nun mit den messbaren Eigenschaften ihrer Objekte befasst, ist natürlich nicht meine Idee.)

Die "physikalischen Objekte " sind auch Teil der Natur, aber nicht einfach "Synonym für Teil der Natur". Ein Mensch (ein Lebewesen) ist ein physikalisches Objekt, aber wer will behaupten, es würde mit einer physikalischen Beschreibung vollständig erfasst sein? Objekte der Natur sind mehr als quantifizierbare Einheiten, sie "sind" auch und vor allen Dingen Qualität, und die lässt sich nicht messen. Ein Elektron z. B. ist ein physikalisches Objekt, über das wir nur in Modellen aufgrund von Messungen etwas aussagen können. Wenn wir auch nicht sagen können, was ein Elektron letztlich tatsächlich ist (wir also nichts über seine Qualität sagen können), so ist es aber dieses "Etwas", über das unsere Modelle Aussagen machen.

Henry
 

  Beitrag 1896-64
-

 
 
Henry in 1896-20:
Grtgrt in 1896-19:
 
Henry und Harti,

die Tatsache, dass ihr — anders als ich — den Begriff "physikalisches Objekt" als Synonym für "ein Teil der Natur" seht, zeigt mir, dass ich in Zukunft wohl besser von physikalischen Modellen (statt Objekten) sprechen sollte.

Ein Elektron, aber z.B. auch ein ganzes Universum, wären dann also stets zu verstehen als ein Teil der Natur, dessen Verhalten wir zu beschreiben suchen über ein oder mehrere — gleichwertige oder einander ergänzende — Modelle.

Danke & Gruß,
grtgrt
 

Hallo, Gebhard,
wir kommen der Sache langsam näher! Ich kann nur für mich sprechen: Mit einem "physikalischen Objekt" meine ich einen quantifizierbaren, also messbaren Teil der Natur und nicht das Modell davon. (Dass sich die Physik nun mit den messbaren Eigenschaften ihrer Objekte befasst, ist natürlich nicht meine Idee.)

Die "physikalischen Objekte " sind auch Teil der Natur, aber nicht einfach "Synonym für Teil der Natur". Ein Mensch (ein Lebewesen) ist ein physikalisches Objekt, aber wer will behaupten, es würde mit einer physikalischen Beschreibung vollständig erfasst sein? Objekte der Natur sind mehr als quantifizierbare Einheiten, sie "sind" auch und vor allen Dingen Qualität, und die lässt sich nicht messen. Ein Elektron z. B. ist ein physikalisches Objekt, über das wir nur in Modellen aufgrund von Messungen etwas aussagen können. Wenn wir auch nicht sagen können, was ein Elektron letztlich tatsächlich ist (wir also nichts über seine Qualität sagen können), so ist es aber dieses "Etwas", über das unsere Modelle Aussagen machen.

Henry


Mir wird jetzt erst klar, dass man allzu oft selbst da noch » physikalisch « sagt, wo man » physisch « meint.

Daher sei klargestellt:
    Ein physisches Objekt ist ein nicht nur in Gedanken existierender Teil unserer Welt (oder des Kosmos).
    Ein physikalisches Objekt dagegen ist ein durch Physiker diskutiertes Modell eines physischen Objekts (oder einer Klasse physischer Objekte).

 

  Beitrag 1896-65
-

 
 
Grtgrt in 1896-64:
 

Mir wird jetzt erst klar, dass man allzu oft selbst da noch » physikalisch « sagt, wo man » physisch « meint.

Daher sei klargestellt:
    Ein physisches Objekt ist ein nicht nur in Gedanken existierender Teil unserer Welt (oder des Kosmos).
    Ein physikalisches Objekt dagegen ist ein durch Physiker diskutiertes Modell eines physischen Objekts (oder einer Klasse physischer Objekte).

 

Gebhard, wenn ich "physikalisch" sage, meine ich auch "physikalisch", nämlich "zur Physik gehörend", und die Physik ist die "wissenschaftliche Beschäftigung mit den Naturerscheinungen" (Zitat aus Wikipedia).

"physich" meint: körperlich im Gegensatz zum geistigen; die Natur betreffend im Gegensatz zum Metaphysischen, also "über die Natur hinausgehend"; das Körperliche betreffend, z. B. synonym zu "somatisch"; allgemein zur Natur gehörend.
 

  Beitrag 1896-66
-

 
 
Hallo Henry,

hat man deinen Beitrag 1896-65 so zu verstehen, dass Du jetzt doch mit mir der Meinung bist, dass gilt:
  • physisch = wie das Objekt in der Natur existiert
  • physikalisch = wie der Physiker das Objekt sieht (und wie er es demnach auch modelliert)?

Gruß, grtgrt
 

  Beitrag 1896-21
Platon oder Aristoteles - wer hatte recht?

 
 
Grtgrt aus 1896-1:
 
Mathematische Gesetze sind Teil der Natur
und Mathematik kann als der Teil der Physik verstanden werden, der diesen Teil der Natur entdeckt
und diskutierbar macht (modelliert).

 


Einige Tatsachen, diese Meinung zu begründen, habe ich oben in Beitrag 1896-4 schon genannt.

Noch viel interessanter aber finde ich, dass offenbar schon Platon dieser Meinung war: Max Tegmark — Professor für Physik am MIT — schreibt in Spalte 2 auf Seite 12 eines Papiers, in dem er Ideen für potentiell existierende Parallel-Universen zusammenträgt und klassifiziert folgendes:


Zitat von Max Tegmark, MIT:
 
There are two tenable but diametrically opposed paradigms regarding physical reality and the status of mathematics, a dichotomy that arguably goes as far back as Plato and Aristotle, and the question is which one is correct.
  • ARISTOTELIAN PARADIGM: The subjectively perceived frog perspective is physically real, and the bird perspective and all its mathematical language is merely a useful approximation.
  • PLATONIC PARADIGM: The bird perspective (the mathematical structure) is physically real, and the frog perspective and all the human language we use to describe it is merely a useful approximation for describing our subjective perceptions. What is more basic — the frog perspective or the bird perspective? What is more basic — human language or mathematical language?


Nebenbei: Dass auch Aristoteles nicht falsch liegt, erkennt sofort, wer sich klar macht, dass mathematische Sprache nur Methodik darstellt und jedes Modell, auch wenn es mathematisch aufgebaut ist, ja noch keineswegs wirklich in jeder Hinsicht isomorph zur Natur sein muss.

In anderen Worten: Der von Tegmark gesehene Widerspruch zwischen den Meinungen von Platon und Aristoteles verschwindet, wenn man meiner Deutung des Wesens der Mathematik folgt: Sie ist ein Paar bestehend aus
  • Naturgesetzen einerseits
  • und des Menschen präzisester Methodik sie zu formulieren andererseits.

grtgrt
 

  Beitrag 1896-25
Beide hatten recht

 
 
Grtgrt aus 1896-21:

In anderen Worten: Der von Tegmark gesehene Widerspruch zwischen den Meinungen von Platon und Aristoteles verschwindet, wenn man meiner Deutung des Wesens der Mathematik folgt: Sie ist ein Paar bestehend aus
  • Naturgesetzen einerseits
  • und des Menschen präzisester Methodik sie zu formulieren andererseits.

Hallo Grtgrt,
ich stimme Dir im Prinzip zu, würde aber als "Mathematik" nur den zweiten Teil des von Dir genannten Paares auffassen wollen.
Wenn man die Trennung objektiv existierende Welt (Natur, Wirklichkeit, naturgesetzliche Geschehensabläufe etc) und Denken (Vorstellung, Bewußtsein, Theorien etc.) für zweckmäßig hält, ist die Mathematik unser päzisestes und widerspruchfreies System zur Beschreibung und Erfassung der Natur.
Demgegenüber sind andere Systeme wie Sprache (Bilder) mit Mängeln behaftet, die insbesondere in der Ungenauigkeit der begrifflichen Inhalte bestehen und deshalb häufig zu Missverständnissen führen. Allerdings ist die Mathematik ein Spezialsystem, das von der Mehrheit der Menschen, auch von mir, nur sehr begrenzt (teilweise) verstanden wird. (Auch Einstein soll sich für die mathematische Formulierung seiner Allgemeinen Relativitätstheorie Hilfe geholt haben).
Allerdings ist auch bei der Beschreibung der Natur mit Hilfe der Mathematik Vorsicht geboten, insbesondere wenn Unendlichkeiten auftauchen. Beispielsweise kann man mit Hilfe von Grenzwertbetrachtungen eine Momentangeschwindigkeit angeben, was nichts anderes bedeutet, als dass man die Beziehung Strecke/Zeit auf eine Strecke mit der Länge 0 und eine Zeit mit der Dauer 0 zueinander in Beziehung setzt. Eine Bewegung, die über 0 Meter erfolgt, ist aber keine Bewegung mehr, sondern bedeutet in unserer sprachlichen Beschreibung "Ruhe". Ich bin der Meinung, dass die Unschärferelation (Ort und Geschwindigkeit eines Elementarteilchens können nicht gleichzeitig mit beliebiger Genauigkeit festgestellt werden) hierin begründet ist.
Dies wird auch deutlich,wenn man die Bewegung von Teilchen im Rahmen der vierdimensionalen Raumzeit betrachtet. Hier gibt es keine "Ruhe" im herkömmlichen Sinn, allenfalls Bewegung nur in der Zeit (geometrisch auf der Zeitachse). Eine Unterscheidung zwischen Ort (Ruhe) und Geschwindigkeit (Bewegung) ist nur in dem Sinne möglich, dass sich ein Objekt mit Lichtgeschwindigkeit allein in der Zeit bewegt (im herkömmlichen Sinn räumlich ruht) oder mit Lichtgeschwindikeit sich teilweise in der Zeit und teilweise im Raum bewegt (im herkömmlichen Sinn in räumlicher Bewegung ist). Die Unschärferelation verliert deshalb nach meiner Auffassung, auf der Grundlage der vierdimensionalen Raumzeit betrachtet, ihre Bedeutung.

MfG
Harti
 

  Beitrag 1896-22
Natur und Primzahlen

 
 
Hallo Gebhard,


Grtgrt aus 1896-21:
...dass mathematische Sprache nur Methodik darstellt und jedes Modell, auch wenn es mathematisch aufgebaut ist, ja noch keineswegs wirklich in jeder Hinsicht isomorph zur Natur sein muss.

Es könnte allerdings sein, dass die universelle Sprache der Natur wirklich Mathematik ist, und reine Mathematik nichts von Menschen erdachtes (sondern "nur" erkanntes), sondern vielmehr der Bauplan des Universums ist:


http://www.3sat.de/mediathek/?display=1&mode=pl...

(Hallo Leute, dieser Link verweist auf einen Film, der ca. 45 min lang ist. Also plant entsprechend Zeit und Muße ein. Von mir allerdings kann ich behaupten, dass mich der Inhalt umgehauen hat.)

Dir, Gebhard, dürfte vieles in diesem Film nicht fremd sein, aber vielleicht lohnt er auch für dich.

Grüße
 

  Beitrag 1896-26
Begriffsverwirrung

 
 
Hi Harti,

da die Naturgesetze in dem Paar, von dem ich spreche, mathematische Gesetzmäßigkeiten sind, fällt es mir schwer, sie nicht der Mathematik zuzurechnen.

Im Prinzip aber hast du recht: Man könnte den Begriff Mathematik so definieren, dass damit nur des Menschen Methodik gemeint ist, jene Naturgesetze (mathematischer Art) zu formulieren, zu entdecken, und praktischer Anwendung zugänglich zu machen.

Gruß, grtgrt
 

  Beitrag 1896-52
Wie Bohr und Kant auf unterschiedlichem Weg zum selben Ergebnis kommen ...

 
 

Schon erstaunlich, wie Physiker und Philosphen zum selben Ergebnis kommen:

Zitat von N. Bohr:

 
Die Physik kann nicht ergründen, wie die Natur funktioniert.

Aufgabe der Physik ist lediglich, zu untersuchen, wie die Natur sich uns zeigt.

 

Zitat von I. Kant:
 
Es gibt die Dinge der Erscheinungen und die Dinge an sich.

Wir kennen die Dinge nur so, wie sie auf uns wirken.


 

Mehr zu Kants Philosophe ...

 

Interessant auch: Weit vor den beiden vertrat diese Meinung auch schon Parmenides (geboren um 530 v.Chr.). Er schrieb:

Zitat von Parmenides:

 
Die Welt, in der wir zu leben glauben, ist die vermeintliche Welt der Sinneswahrnehmungen;
die Welt ist nur Meinung ...

 


Und genau das bestätigt uns im 20. Jahrhundert die Quantenphysik.
 

  Beitrag 1896-55
Besteht unsere Welt wirklich nur aus der Raumzeit und dem darin Vorhandenen?

 
 
Hallo H...,

  • konkretes, anerkanntes (und einziges mir bekanntes) Beispiel für "unverzögert fernwirksame Phänomene" ist das Phänomen der Quantenverschränkung.
  • Was Prozesse angeht, die außerhalb der Raumzeit ablaufen, so sehe auch ich keinerlei Hinweise darauf.
  • Ich sehe aber sehr wohl Anzeichen dafür, dass unsere Welt nicht nur aus der Raumzeit bestehen könnte, sondern darüber hinaus eine bisher noch nicht identifizierte zusätzliche transzendente Dimension haben muss: Eine Art Schema, welches die Naturgesetze darstellt, die Regeln also, die bestimmen, an welche Randbedingungen sich alles andere — die Raumzeit und alles, was darin existiert — zu halten hat. Etwas mehr dazu in Beitrag 1915-7.

Beste Grüße,
grtgrt
 

  Beitrag 1904-5
Wie Ergebnisse des Delayed Choice Quantum Eraser Experiments bedeuten könnten ...

 
 
Hi C...,

ich bin sicher, dass er gar keine wissenschaftliche Karriere hat. Er scheint — sein Buch mal ausgenommen — keinerlei wissenschaftliche Veröffentlichungen produziert zu haben.

Es könnte schon sein, dass sein Buch niemals durch Unvoreingenomme gezielt einem genauen Review unterzogen wurde.

Nachdem es aber nun schon fast 5 Jahre auf dem Markt ist, dachte ich, ich fände einen in der Forschung tätigen Physiker, der es schon gelesen hat.

Auf Campbell aufmerksam wurde ich durch seine Deutung der Ergebnisse des Delayed Choice Quantum Eraser Experiments.

Sie scheint mir durchaus einen genaueren Blick wert, und so werde ich sein Buch auf jeden Fall mal ganz genau lesen (und noch vor Ende diesen Jahres hier in diesem Forum meine Meinung dazu sagen).

Dennoch bitte ich jeden Physiker, der schon eine Meinung dazu hat, sie hier kundzutun.

Gruß, grtgrt
 

  Beitrag 1904-6
Eine Meinung, die ich keineswegs teile

 
 
Hallo Grtgrt,

die Themenfrage wird immer wieder in den verschiedensten Formen gestellt. Ich habe das Gefühl, sie ist nicht eindeutig zu beantworten, weil sie verdeckt zwei widersprüchliche Antwortmöglichkeiten enthält, die nicht zu klären sind. Um dies deutlich zu machen, formuliere ich sie in etwas anderer Form, ohne dass nach meiner Meinung der Sinn verändert wird.

Existiert das objektiv, was wir subjektiv wahrnehmen ?

In der Fragestellung werden eine subjektive Wahrnehmungswelt und eine objektiv existierende Welt gegenübergestellt.

Sowohl die positive wie die negative Beantwortung der Frage können wir nicht wissen, da sie außerhalb unserer Wahrnehmung liegt.

1. Antwort: Es gibt eine real existierende Welt außerhalb unserer Wahrnehmung. Wie können wir das wissen, wenn dies außerhalb unserer Wahrnehmung liegt ?

2. Antwort: Es gibt keine real existierende Welt außerhalb unserer Wahrnehmung. Wie können wir das Wissen ( ....s.o.)

Es handelt sich deshalb um eine reine Glaubensfrage, die jeder für sich selbst beantworten kann, wie er will.

MfG
Harti
 

  Beitrag 1904-7
Real ist nicht notwendig das, was wir als real erachten!

 
 
Hi Harti,
was wir als real erachten, könnte durchaus davon abhängen, wie genau wir (bisher schon) hingesehen haben.

Mit anderen Worten: Es könnte sich lohnen, erneut hinzusehen, denn es gibt in der Geschichte der Wissenschaften ja zahlreiche Beispiele, die zeigen, dass erst die Beschäftigung mit einer neu propagierten Halbwahrheit zur Entdeckung der eigentlichen Wahrheit geführt hat.

Gruß, grtgrt
 

  Beitrag 1904-8
Realität versus Wirklichkeit: Okotombrok sieht da einen Unterschied (ich nicht)

 
 
Hallo zusammen,

ich mache für mich eine Unterscheidung zwischen Wirklichkeit und Realität. (bei Kreuzworträtseln mache ich da eine Ausnahme :lol:)
Wirklichkeit ist für mich alles, was mit meinen Sinnesorganen incl. Hilfsmitteln wie Brille und Teilchenbeschleunigern wechselwirkt. Gut, mögen unsere Sinne uns oft täuschen so lassen sie uns doch in der Wirklichkeit gut zurechtfinden.

Forumsteilnehmer Zara-t. formulierte das einmal in etwa so: "Wirklichkeit ist, wenn Wirkliches auf Wirkliches wirkt"
(ich kann es jetzt nur aus dem Gedächtnis wiedergeben und vermeide deshalb die Zitierfunktion)
womit ich nicht meine, die Wirklichkeit hänge ausschließlich vom Menschen ab. Schließlich erfahren wir Wirkungen auch aus einer Zeit als es noch keine Menschen gab.

Ob hinter Allem aber eine übergeordnete Realität existiert, etwas von den Wirkungen Unabhängiges, vermögen wir nicht zu sagen. Die Physik fragt nicht danach und Antworten können nur die Religionen geben. Diese Antworten können aber einen Physiker in seiner Arbeit nicht zufrieden stellen, entziehen sie sich doch jeglicher experimenteller Bestätigung und sind vor allem nicht falzifizierbar. Gleichwohl gibt es Physiker, die an Gott und an ein Weitergehen nach dem Ableben glauben, und das sind nicht die Dümmsten.

mfg okotombrok
 

  Beitrag 1904-13
Nachweisbare und transzendente Wirklichkeit

 
 
Okotombrok aus 1904-8:
Hallo zusammen,

ich mache für mich eine Unterscheidung zwischen Wirklichkeit und Realität.
Wirklichkeit ist für mich alles, was mit meinen Sinnesorganen incl. Hilfsmitteln wie Brille und Teilchenbeschleunigern wechselwirkt. Gut, mögen unsere Sinne uns oft täuschen so lassen sie uns doch in der Wirklichkeit gut zurechtfinden.

Auch an alle:

Für mich besteht Realität aus
  • nachweisbarer Wirklichkeit (= Wirklichkeit, die man beobachten oder beweisen kann)
  • und transzendenter Wirklichkeit (= Wirklichkeit, von deren Nicht-Existenz auszugehen zu wenig Sinn macht, obgleich man ihre Existenz weder beobachten noch beweisen kann).

Aus der Sicht formaler Logik (und als Mathematiker) sehe ich transzendente Wirklichkeit als eine Menge von Aussagen, die ich absolute Axiome nenne — als Aussagen also, deren Wahrheit ich nicht in Frage stellen möchte.

Sollte mich jetzt jemand nach einer Liste absoluter Axiome fragen, so könnte die gut nur ein einziges enthalten: Den Satz vom Widerspruch, auf dessen Bedeutung schon Aristoteles hingewiesen hat.

Gruß, grtgrt
 

  Beitrag 1904-17
Materielle und vermeintliche Wirklichkeit

 
 
Hans-m aus 1904-16:
Okotombrok aus 1904-8:
Wirklichkeit ist für mich alles, was mit meinen Sinnesorganen incl. Hilfsmitteln wie Brille und Teilchenbeschleunigern wechselwirkt.

Copperfield´s Verschwinden aus dem Käfig wechselwirkt auch mit deinen Sinnen und ebenfalls der 3D Film auf der Kino-Leinwand,
aber kannst Du das als Wirklichkeit definieren?

  • Was Okotombrok als Wirklichkeit definiert, ist materielle Wirklichkeit.
  • Copperfield's Verschwinden aber ist vermeintliche Wirklichkeit.

Der 3D Film auf der Kinoleinwand ist real als auf die Leinwand fallendes List. Er ist auch real als Geschichte, die erzählt wird.
Wer aber möchte behaupten, dass, was er erzählt, notwendigerweise Wirklichkeit darstellt?


 

  Beitrag 1904-20
Heisenbergs Antwort: auch Potentia ist existentes Ding

 
 
Okotombrok aus 1904-19:
Hallo Grtgrt,

Grtgrt aus 1904-15:
Auch Heisenberg hat sich darin versucht, zu definieren, was wirklich ist und existiert: Er spricht von Objekten und Zuständen, die

"als Potentia in einer seltsamen Wirklichkeit zwischen der Idee von einem Ding und einem wirklichen Ding existieren".

ich verstehe Heisenberg eher so, dass mit "Potentia" und "der Idee von einem Ding" die Zukunft gemeint ist. Sie bedeutet Möglichkeiten, noch nicht entschieden, unbestimmt im Rahmen von Wahrscheinlichkeiten (Unbestimmtheitsrelation), im Gegensatz zur klassischen Mechanik, bei der die Zukunft determiniert ist.
Mit dem "wirklichen Ding" meint er etwas, was wechselgewirkt hat, was zum Faktum wurde und von dem wir prinzipiell wissen können – Vergangenheit.


Diese Meinung, Okotombrok, teile ich nicht.

Heisenbergs Potentia scheinen mir ganz klar die Zustände zu sein, die eintreten k ö n n e n , also irgendwie schon vorgedacht und möglich gemacht sind.


In seinem Buch "Versteckte Wirklichkeit" (Hirzel, 2004) sieht Lothar Schäfer das ebenso und schreibt:

"Das einfachste Beispiel ist ein Wasserstoffatom. Wenn es in seinem Grundzustand ist – chemisches Symbol 1s – dann muss man annehmen, dass die Zustände höherer Energie – allgemein mit den Symbolen 2s, 3s, 3p usw. bezeichnet – in diesem Atom auch existieren, obgleich sie leer und daher nicht in einem materiellen Sinn wirklich sind.

... Sie existieren wie mathematische Formen, Ordnungsmuster oder wie Information, aber sie sind mehr als nur die I d e e einer solchen Form, weil sie wirklich betreten werden können. ...

... Zwar wird der Akt des Springens von einem Quantenzustand in einen anderen vom Zufall bestimmt, aber das gilt nicht für die Ordnung des Zustandes, der so erreicht wird:
Blinder Zufall kann zu Allem führen, ob er aber auch erschafft, wohin er führt, ist eine völlig andere Frage."


Beste Grüße,
grtgrt
 

  Beitrag 1904-21
Wirkliches kann nachweisbar oder transzendent sein

 
 
Grtgrt aus 1904-13:
Für mich besteht Realität aus
  • nachweisbarer Wirklichkeit (= Wirklichkeit, die man beobachten oder beweisen kann)
  • und transzendenter Wirklichkeit (= Wirklichkeit, von deren Nicht-Existenz auszugehen zu wenig Sinn macht, obgleich man ihre Existenz weder beobachten noch beweisen kann).

... eine "geistige" Welt, die der realen, nein materiellen Welt entgegenstehe... die aus sich heraus existiere, und eben einen beachtlichen Teil dieser Realität ausmache. Geist und Materie, es gibt sie beide!


 

  Beitrag 1904-22
Dinghafte Wirklichkeit existiert nur als Abstraktion (als Wahrnehmung also)

 
 
Hans-Peter-Dürr, schreibt in seinen Reflexionen einen Quantenphysikers:
 
Zitat:
 
Der Bruch, den die neue [mit der Quantenmechanik beginnende] Physik fordert, ist tief. Deutet diese Physik doch darauf hin, daß
die Wirklichkeit, was immer wir darunter verstehen, im Grunde keine Realität im Sinne einer dinghaften Wirklichkeit ist.

Wirklichkeit offenbart sich primär nurmehr als Potentialität, als ein "Sowohl-als-auch" ...

 

 

  Beitrag 1904-29
Real und gleichzeitig nicht real zu sein, schließen einander NICHT aus

 
 
Thomas der Große aus 1904-28:
Alles ist real


Das, Thomas, sehe ich durchaus ebenso,
denn wenn ich z.B. an Platons Höhlengleichnis denke, so sind dort ja die Schatten an der Wand (als Schatten) ebenso real, wie Gegenstände, die jene Schatten werfen.

Was wir meinen, wenn wir von mehr oder weniger Realem sprechen, ist also stets
  • entweder die Frage, welche Realität denn nur Folge welcher anderen ist (eine Ordnung auf der Menge aller Realitäten also)
  • oder einfach nur die Frage, was uns aufgrund welcher gedanklichen Fehler als real erscheinen mag (ohne dass es wirklich existiert): Ein amputiertes Bein etwa, das dem Patienten immer noch über Phantomschmerz als vorhanden erscheint, ist nun mal nicht mehr real — real ist in dem Fall nur der Schmerz.

Man kann es auch so sagen:

Wo wir uns fragen, ob ein Ding X real ist, fragen wir danach, ob der Betrachter es in der richtigen Rolle als real sieht.
Bezogen auf Platons Höhlengleichnis etwa wäre der Schatten einer Ziege, den Gefangene an der Wand der Höhle sehen, für sie
  • als REAL einzustufen, wenn ihnen bewusst wäre, dass, was sie sehen, nur ein Schatten ist,
  • als NICHT REAL aber, wenn sie eben diesen Schatten für das eigentliche Ding X halten (in diesem Fall eine Ziege).

Gruß, grtgrt

 

  Beitrag 1904-31
Realität ist nur in bestimmter Rolle wirklich Reales

 
 
Thomas der Große aus 1904-30:
Hallo Grtgrt,

Du hast oben von einer transzendenten Wirklichkeit gesprochen, die man sinnvollerweise annimmt.
Die wird man zum Masstab für alle Schattenspiele machen.

Hi Thomas,

was ich ausdrücken wollte war: Es gibt eine transzendente (d.h. durch uns unverstandene, oder noch unverstandene) Wirklichkeit.

Wo wir über sie nachdenken, kann solches Nachdenken aber nur zum Ziel haben, Teile davon zu verstehen.

Zum Maßstab aller Überlegungen darf man sie auf keinen Fall machen, denn wo man ohne Not auf noch Unverstandenem aufbaut, kann sich eigentlich keinerlei Beweis mehr ergeben. Von logischem Vorgehen hätte man sich dann verabschiedet.


In Bezug auf verschiedene Realitäten sei festgestellt:

Wo immer uns etwas begegnet, das wir versucht sind als Realität zu sehen, sollten wir uns bewusst fragen, was die ihr zugeordnete Rolle ist.
Wo die nämlich nicht richtig erkannt wird, ist man auf dem Holzweg: Der Schatten einer Ziege gibt z.B. keine Milch, und aufessen kann man ihn auch nicht.


Thomas der Große aus 1904-30:
Die Quantentheorie ist per Definition ein Schattenspiel, weil sie ihren Zufallsgenerator nicht kennt.

Mit dieser Aussage kann ich ehrlich gesagt rein gar nichts anfangen.


Gruß, grtgrt
 

  Beitrag 1913-4
Wie man erkennt, dass die Grundlage der materiellen Welt nicht-materiell ist

 
 
Wie in Beitrag 1913-3 gezeigt, wird Evolution durch zweierlei Zufall gesteuert:
  • durch einen vordergründig wirkenden (das ist der, von dem Darwin und die Beispiele aus Beitrag 1913-2 sprechen),
  • daneben aber auch durch einen hintergründig wirkenden (das ist der quantenphysikalische).

Es ist wichtig, zu sehen, dass der hintergründige den vordergründigen kanalisiert, selbst aber auch kanalisiert wird durch die den Quanten zugeordneten Wahrschein­lichkeitswellen (durch das also, was Heisenberg als die der kosmischen Ordnung zugrundeliegenden "Tendenzen und Neigungen" nennt: siehe Beitrag 1915-5).


Da nun aber Wahrscheinlichkeiten nichts anderes als dimensionslose Zahlenverhältnisse sind, kommt man nicht umhin, feststellen zu müssen:

Zitat von Lothar Schäfer in: Versteckte Wirklichkeit, S. 47:
 
An der Wurzel der Wirklichkeit finden wir Zahlenverhältnisse — nichtmaterielle Prinzipien, auf denen die Ordnung dieser Welt gegründet ist.

Die Grundlage der materiellen Welt ist somit nichtmateriell.

 


Wem das zu abstrakt ist, der sollte sich vor Augen führen, dass man — ausgehend von der Kenntnis der den Quanten zugeordneten Wahrscheinlichkeitswellen — die Struktur von Molekülen berechnen kann, letztlich also die Gesetze der Chemie.

Wie weit man da heute schon ist, weiß ich nicht genau. Auf jeden Fall gilt:
  • Schon 1998 gelang es, die Struktur von Cambrin zu berechnen. Cambrin ist mit etwa 640 Atomen ein eher kleines Protein-Molekül. Über noch größere Erfolge berichten Treppen et al., 2002, J. Phys. Chem. A. 106; 5498-5503.
  • Insbesondere weiß man: Die Struktur selbst der größten röntgenographisch erforschten Proteine kann durch quanten-chemische Berechnungen ihrer Teile vorhergesagt werden (Jiang et al., 1995, Phys. Chem. 99:10521).
Solcher Erfolg der Quantenchemie bei der zuverlässigen Berechnung der Eigenschaften von Molekülen — unabhängig von ihrer Größe — beweist ganz klar:


Alle Moleküle, ob groß oder klein, sind Quantensysteme,

und so wirkt der quantenphysikalische Zufall hinein in sämtliche chemikalischen Vorgänge (!).


Das ist für die Evolution durchaus bedeutsam, wenn man sich vor Augen führt, dass quantenphysikalischer Zufall ja nur bezogen auf das Ergebnis je eines einzelnen Ereignisses absoluter Zufall zu sein scheint, statistisch gesehen aber, wie oben gerade erklärt, durchaus kanalisierend wirkt.

grtgrt

PS: Zur Terminologie, den Zufall betreffend, siehe Beitrag 1911-1.

 

  Beitrag 2085-287
Die Natur zu beobachen bedeutet, sie zu verändern

 
 
Okotombrok in 2085-283:
Bauhof in 2085-278:
gerade darum, weil wir Teil der Realität sind, können wir nie die Realität in ihrem Wesen erkennen. Unsere Realität ganz zu überblicken, könnte nur ein Wesen, das außerhalb unserer Realität steht.

das ist wohl richtig.
Wir sind aber nicht Teil der Natur in dem Sinne, wie ein Wassertropfen Teil eines Ozeans ist. Zunächst sind wir Individuen und als solche getrennt von der Umwelt zu sehen. Das macht ein Individuum aus.
Gleichwohl können wir nicht erkennen, was die Natur an sich ist. Wir können nur eine Natur erkennen, wie sie sich unserer Fragestellung zeigt. Und die Antworten die wir erhalten verstehen wir immer nur im Vergleich mit anderen Antworten. Messen heißt Vergleichen, wie wir es in der Schule gelernt haben, und die Bedeutung dieses Satzes ist mir erst viel später bewusst geworden.

Hallo Okotombrok,

was ich da geschrieben hatte, klingt zwar vielleicht philosophisch, sollte es aber nicht sein. Ich hatte etwas anderes im Sinn als Philosophie:

Das Quanten-Geschehen ist gekennzeichnet durch die Heisenbergsche Unbestimmtheitsrelation. Wenn wir messen, kommt noch ein unbestimmbarer Eingriff durch den Messvorgang hinzu, der die vermeintliche Realität zusätzlich verschleiert. Deshalb ist für uns als Beobachter unmöglich, eine (vermeintlich) vorhandene wahre Realität zu erkennen, weil wir und unsere Messinstrumente Teil des Universums sind. Bei jedem Messvorgang ist der Beobachter mit seiner speziellen Art des Messens am Messergebnis beteiligt. Messobjekte, Messinstrumente und Beobachter sind eine untrennbare Einheit.

Und weil dem so ist, können wir nie die Realität in ihrem Wesen erkennen. Wir können nicht vom Messinstrument abstrahieren. Deshalb ist das für uns Realität, was die Messinstrumente anzeigen. Eine dahinter stehende "wahre" Realität ist uns prinzipiell unzugänglich, wenn sie denn überhaupt gegeben ist. Die Quantentheorie verneint eine dahinterstehende Realität.

M.f.G. Eugen Bauhof
 
tags: Physik1gegreit Zeitbegriff1gegreit Wesen1gegreit